You are on page 1of 406
THE BEST TEST PREPARATION FOR THE GRE=- PHYSICS Joseph J. Molitoris, Ph.D. Professor of Physics . Muhlenberg College Allentown, Pennsyivania Research and Education Association 61 Ethel Road West Piscataway, New Jersey 08854 CONTENTS INTRODUCTION About Research and Education Association. ‘About the Book About the Test About the Author About the Review Scoring the Exam .. GRE PHYSICS REVIEW 1 CLASSICAL MECHANICS Vectors Linear Motion. Two-Dimensional Newton's Laws Momentum. Energy and Work Harmonic Motion Collisions... Lagrangian Mechani 2 ELECTROMAGNETISM Electric Fields Capacitors .. Current and Resistance Magnetic Fields and Currents 3 ATOMIC PHYSICS Rutherford Scattering. Atomic Spectra The Bohr Atom The Laser ... 4 THERMODYNAMICS Temperature svcscreeseirne Coefficient of Thermal Expansion Heat Capacity, Specific Heat and Cp .. Heat of Vaporization and Heat of Fusion 53 Conduction, Convection and Radiation 54 Heat. Work and The Laws of Thermodynamics 55 Entropy -. 5 QUANTUM MECHANICS ‘Wave Functions and Equations. 57 ‘Schrédinger’s Equation... Potential Wells and Energy Level Harmonic Oscillator. Reflection and Transmission by a Barr 6 SPECIAL RELATIVITY Time Dilation and Length Contraction .. Dynamics Lorentz Transformations .. 7 optics Mechanical Waves Ray Optics Thin Lenses . Interference .. Diffraction TABLE OF INFORMATION NOTATIONS .. FOUR PRACTICE EXAMS GRE Physics Test 1 Answer Sheet Form Detailed Explanations of Answers . GRE Physics Test 2 Answer Sheet Form Test 2 Answer Key.. Detailed Explanations of Answers GRE Physics Test 3 ‘Answer Sheet Form Test 3... ‘Answer Key.. Detailed Explanations of Answers GRE Physics Test 4 Answer Sheet Form 316 Test 4 . Answer Key. Detailed Explanations o About Research and Education Association Research and Education Association (REA) is an organization of educators, scientists, and engineers who specialize in various academic fields. REA was founded in 1959 for the purpose of disseminating the most recently developed scientitic Intormation to groups in Industry, gov- ernment, high schools, and universities. Since then, REA has become a successful and highly respected publisher of study aids, test preps, hand- books, and reference works. REA’s Test Preparation series extensively prepares students and professionals for the Law School Admission Test (LSAT), the Graduate Record Examinations (GRE), the Graduate Management Admission Test (GMAT), the Scholastic Aptitude Test (SAT), as well as the Test of En- glish as a Foreign Language (TOEFL), and the Advanced Placement Exams. Whereas most test preparation books present a few practice exams which bear little resemblance to the actual exams, REA’s test preparation books usually present six or more exams which accurately depict the official tests in degree of difficulty and In types of questions. REA’s prac- tice exams are always based on the most recently administered tests, and include every type of question that can be expected on the actual tests. REA’s publications and educational materials are highly regarded for their significant contribution to the quest for excellence that characterizes today's educational goals. We continually receive an unprecedented amount of praise from professionals, instructors, librarians, parents, and students for our books. Our authors are as diverse as the subjects and fields represented in the books we publish. They are well-known in their respective fields and serve on the faculties of prestigious universities throughout the United States. GRE Physics About the Book This book provides an accurate and complete representation of the Graduate Record Examination in Physics. The four practice exams and review section are based on the most recently administered GRE Physics Exam. Each exam Is two hours and fifty minutes in lenath and includes ‘every type of question that can be expected on the actual exam. Follow- ing each exam is an answer key, complete with detailed explanations designed to clarify the material for the student. By studying the review section, completing all four exams, and studying the explanations which follow, students can discover thelr strengths and weaknesses and thereby become well prepared for the actual exam. About the Test The Graduate Record Examination in Physics Is offered four times a year by the Educational Testing Service, under the direction of the Gradu- ate Record Examinations Board. Applicants for graduate school submit GRE test results together with other undergraduate records as part of the highly competitive admission process to graduate school. The GRE tests are intended to provide the graduate school admissions committee with a means of evaluating your competence in certain subject areas. Scores on the test are intended to indicate mastery of the subject matter empha- sized in an undergraduate program. The test consists of about 100 multiple-choice questions, some of which are grouped In sets and based on such materials as diagrams, experimental data, graphs, and descriptions of physical situations. Em- phasis Is placed on the ability to grasp fundamental principles of physics as well as the ability to apply these principles. Most test questions can be answered on the basis of a mastery of the first three years of undergradu- ate physics. Emphasis Is placed on the following major areas of physics and occur in the percentages indicated. These percentages reflect the relative emphasis placed on these topics In most undergraduate curricula. 1. Fundamentals of electromagnetism, Including Maxwell's equa- tions (18%) Classical mechanics (18%) ‘Atomic physics (15%) Quantum mechanics (10%) Physical optics and wave phenomena (10%) are vi Introduction Special relativity (7%) ‘Thermodynamics and statistical mechanics (7%) Laboratory methods (5%) Advanced topics: Lagrangian and Hamiltonian mechanics, solid state physics, nuclear and particle physics and miscellaneous (10%) 9 BND About the Author Dr. Joseph Molitoris Is @ professor of Physics at Muhlenberg College in Allentown, Pennsylvania, where he teaches introductory and advanced physics. His teaching responsiblities include courses in General Physics, Modern Physics, Mechanics, Advanced Mechanics, Statistical Physics, and Nuclear Physics. After receiving his Bachelor of Science degree in Physics from Mas- sachusetts Institute of Technology, Dr. Molitoris went on to receive his Master of Science degree In Mathematics from the University of North Florida, and then to obtain his Doctor of Philosophy in Physics from Michigan State University. His post-Doctoral work was performed as a fellow of the Alexander von Humboldt Foundation in Frankfurt, Germany. About the Review The review in this book is designed to further your understanding of the test material. It includes techniques you can use to enhance your knowledge of physics and to eam higher scores on the exam. The review includes extensive discussions and examples to refresh your skills. Top- ies covered in the review are: Classical Mechanics + Vectors + Linear Motion + Two-Dimensional Motion + Newton's Laws + Momentum + Energy and Work + Harmonic Motion GRE Physics Collisions Lagrangian Mechanics Electromagnetism Electric Folds Capacttors Current and Resistance Circuits The Magnetic Field Magnetic Fields and Currents Atomic Physics Rutherford Scattering Atomic Spectra The Bohr Atom The Laser Thermodynamics ‘Temperature Coefficient of Thermal Expansion Heat capacity, Specific Heat and C, Heat of Vaporization and Heat of Fusion Conduction, Convection and Radiation Heat, Work and the Laws of Thermodynamics Entropy Quantum Mechanics vit Wave Functions and Equations Schrédingers Equation Potential Wells and Energy Levels Harmonic Oscillator Reflection and Transmission by a Barrier Introduction Special Relativity + Time Dilation and Length Contraction + Dynamics + Lorentz Transformations Optics + Mechanical Waves + Ray Opties + Thin Lenses + Interference + Diffraction Scoring the Exam ‘Two types of scores are obtained from your results on the GRE Physics examination: a raw score and a scaled score. The raw score Is determined first and is then converted into the scaled score. To determine the raw score, a number of things must be done. The following equation represents the process: R - W/4 = Raw Score (round-off If necessary) First calculate the total number of wrong (W) answers. Next, calculate the total number of right (R) answers. Unanswered questions are not counted. At this point, divide the total number of wrong answers by four and: subtract this result from the total number of right answers. This ad- justment Is made to compensate for guessing. Finally, take the last result and round It off to the nearest whole number, which will be the raw score. To determine the scaled score, find the number that corresponds, to the raw score on the table on the following page. GRE Physics GRE Physics — Total Score Raw Score | Scaled Score | Raw Score | Scaled Score 84-100 990 41-42 670 83 980 40 660 81-82 970 39 650 80 960 37-38 640 79 950 36 - 630 71-78 940 35 620 76 930 33-34 610 75 920 32 600 73-74 910 30-31 590 72 900 29 580 | 1 890 28 570 69-70 880 26-27 560 68 870 25 550 67 860 24 540 65-66 850 22-23 530 64 840 21 520 63 830 20 510 61-62 820 18-19 500 60 810 17 490 59 800 16 480 57-58 790 1445 470 56 780 13 460 55 770 12 450 53-54 760 10-11 440 52 750 9 430 51 740 8 420 49-60 730 67 410 48 720 5 400 47 710 4 390 45-46 700 2-3 380 44 690 1 370 43 680 0 360 GRE PHYSICS REVIEW CHAPTER 1 CLASSICAL MECHANICS A. VECTORS A vector is a measure of both direction and magnitude. Vector variables are usually indicated in boldface, or with an arrow, such as ¥. THE COMPONENTS OF A VECTOR 4a, and a, are the components of a vector a. The angle @ is measured counter- clockwise from the positive x-axis. The components are formed when we draw perpendicular lines to the chosen axes. ‘The Formation of Vector Components on the Positive X— Yaxis The components of a vector are given by ‘A component is equal to the product of the magnitude of vector A and co- sine of the angle between the positive axis and the vector. ‘The magnitude can be expressed in terms of the components Finally the angle @ is given by GRE Physics Review Like scalars, which are measures of magnitude, vectors can be added, subtracted and multiplied. To add or subtract vectors, simply add or subtract the prospective x and y coordinates. For example, a-b>4,-b,=¢, a,b, =6, Therefore, C is the sum vector. There are 2 forms of multiplication: the dot product and the vector, or cross product. The dot product yields a scalar value: a-b=abcos® The Cross Product of two vectors yields a vector: axb=c and lel=absin@ 3 3 (a) Dot Prodict (b) cross Product Vector Multiplication ‘The direction of the Vector Product a x b = ¢ is given by the “Right Hand Rule”: 1) With a and b tail-to-tail, draw the angle @ from a to b. 2) With your right hand, curl your fingers in the direction of the angle drawn. The extended thumb points in the direction of ¢. et B The direction of the vector product, ¢=axb (|¢| = ab sin), is into the page. Classical Mechanics Properties of the Cross Product: axb=-bxa ax(b+e)=(axb)+ (axe) (a x b) = (ca) x b = a x (cb), where c is a scalar. lax bP =A2B—(a- bY B. LINEAR MOTION ‘Any object in motion has an average and an instantaneous velocity: a) _ Average Velocity vest Boa at hh 'b)__ Instantaneous Velocity V= tim # =X = yr) ano At de Just as the average and instantaneous velocities are the rate of change of Position with respect to time, acceleration is the rate of change of velocity with respect to time, dey av iy, & dat dt From this, the following basic kinematic equations of motion can be derived: 1 vev,tat 2. vay? + 2a(x—x,) 3. xa tree, at 4 X=%, +h, (Vy + IE where y, and x, are initial values, GRE Physics Review C. TWO-DIMENSIONAL MOTION For 2 dimensional, or planar, motion, simply break the velocity and accel- eration vectors down into their x and y components. Once this is done, the preced- ing one dimensional equations can apply. x A special case of 2 dimensional motion is Uniform Circular Motion. For a particle to be held on a circular path, 4 radial acceleration must be applied. This acceleration is called centripetal acceleration. Centripetal Acceleration y’ a= where a = Acceleration v = Tangential Component of Velocity r = Radius of the Path For uniform circular motion, a can also be written as where T, The tangential component of the acceleration is the rate at which the par- ticle speed changes: Classical Mechanics avd my Ar ae ‘When dealing with circular motion, or other situations involving motion relative to a central force field, it is often appropriate to use cylindrical coordi- nates, where the position is a function of radius and angle (r, 8). In the case of three dimensions, the coordinates become (r, ®, z), where the z-coordinate is identical to the respective cartesian z-coordinate. In describing such motion, o represents angular acceleration do (@) do () If angular acceleration, o,, is constant, then equations correlating to those previously stated for linear motion can be shown to apply. and © represents angular velocity Similarity Table Rotational Motion Linear Motion Equivalent o = constant a = constant ® = tor v= vtat go = ete, ve tott, 2 2 0 = at',or x = vet Mae @ = o2+200 vos v2+2ar ©, = initial and final angular displacements ©, © = initial and final angular velocities. ‘Another type of coordinate system used is the spherical coordinate system, with components (p, 6, 8). GRE Physics Review y r=psing x=rcos® x=p sing cos @ Z= pcos yersin® y=psing sin® e=0 rez z=pcosd D. NEWTON’S LAWS First Law Every body remains in its state of rest or uniform linear motion, unless a force is applied to change that state. Second Law If the vector sum of the forces F acting on a particle of mass m is different from zero, then the particle will have an acceleration, a, directly proportional to, and in the same direction as, F, but inversely proportional to mass m. Symboli- cally (if mass is constant) Third Law For every action, there exists a comesponding equal and opposing reaction, or the mutual actions of two bodies are always equal and opposing. Newton's Laws all refer to the effects of forces on particles or bodies. ‘These forces can be represented in vector form. Force . A) _ is a push or pull that a body exerts on another. B) can be represented by a vector. C) adds and subtracts vectorially. OP=4,x+d,y+dz Unit Vector: Classical Mechanics @ = distance between 0 and F op us OF. OP Ge =F (axtdy+dz) Foran“ (ax+dy+dz) Components: pele po Pee Ga a Distance: Jalsa? sa Directional Cosines of F: 1d, a=cos? a x a 1d, a B=cos" ‘y=cos Unit Vector Expressed in Terms of Angles: u=cos ax +c0s By +cos yz GRE Physics Review Relationship Between Angles: costa. + cos? B+ cost y= 1 E.. MOMENTUM LINEAR MOMENTUM: kgm ‘sec p=my — units: where p = linear momentum of particle mass of particle velocity of particle & wun NEWTON'S SECOND LAW dp _ (mv) a a where F = thenet force on the particle, LINEAR MOMENTUM OF A SYSTEM OF PARTICLES Total Linear Momentum P=)! meri that--+Pn ist S mypy + mgd, +. + MAU where P = total linear momentum of system PM, V, = linear momentum, mass, and velocity of ith particle, respectively. Newton’s Second Law for a System of Particles (Momentum Form). F,,22 dt where F, “4, = sum of all extemal forces Momentum is conserved. The total linear momentum of the system re- mains unchanged if the sum of all forces acting on the system is zero. 10 / Classical Mechanics According to Newton's 2nd Law: i =D Fy 20 ANGULAR MOMENTUM: ‘Angular Momentum ! Vector Equation l=rxP Scalar Equation I=1P sin® where 7 = Angular momentum 7 = Radius P = Linear momentum @ = Angle formed by rand P Zp» tt ‘Angular Momentum The rotational correlation to force is Torque, which relates to angular mo- mentum by the equation. -2 To 3 Torque is simply: Paces) X (0) =T Another way of defining Angular Momentum is the Moment of Inertia times the Angular Velocity: I=l0 where 1 corresponds to mass. (@ being angular velocity) oe ae Physics Feview Determination of : Integration Method Area — General Formula: ref, 3° dA where 5 = Perpendicular distance from the axis to the area element. EXAMPLE: For a Rectangular Area | Ae i ——) x * yt dye Lop? 1y=f, by? dy= 5 bh ‘This is the moment of inertia with respect to an axis passing through the base of the rectangle. ‘Moments of Inertia of Masses: 12 Classical Mechanics In Polar Coordinates, the polar moment of Inertia is noted as J. Polar Moment of Inertia: fraa be In terms of rectangular moments of inertia: L=H+l, ‘The Radius of Gyration can be determined once the moment of Inertia and the Area are known. Polar form: Relation between rectangular component form and polar form: tek k? kaki +k, Masses Impulse and Momentum Impulse-Momentum Method — An alternate method to solving problems in which forces are expressed as a function of time. It is applicable to situations wherein forces act over a small interval of time. 13 GRE Physics Review Linear Impulse-Momentum Equation: fF F ar» imputse = mv, - mv, Ideal impulse produces an instantaneous change in momentum and velocity of the particle without producing any displacement. Mv,+ ZF At= My, Any force which is non-impulsive may be neglected, ¢.g., weight, or small forces. F. ENERGY AND WORK ‘The work done by a force F through a displacement dr dw =F - de in Joules. (SI units) Over a finite distance from point 1 to point 2: Waaf Pedr Work-Energy Principle Kinetic energy for a particle of mass M and velocity v is defined as Kinetic energy is the energy possessed by a particle by virtue of its motion. Principle of Work and Energy — Given that a particle undergoes a dis- placement under the influence of a force F, the work done by F equals the change in kinetic energy of the particle. W,_.= (KE),- (KE), Results of the Principle of Work and Ener A) Acceleration is not necessary and may not be obtained directly by this principle. B) The principle may be applied to a system of particles if each particle is considered separately. C) Those forces that do not contribute work are eliminated. 14 Classical Mechanics Kinetic Energy and Newton's Law: any a Fem G. > Gy (KE). where KE = a function of x. (This applies only in an inertial reference frame.) Power and Efficiency Power is defined as the time-rate of change of work and is denoted by dw/ dt, Power= Wap. y dt Mechanical Efficiency: Power out ‘Power NOTE: nis always <1. Potential Energy Potential Energy = The stored energy of a body or particle in a force field associated with its position from a reference frame. If PE represents potential energy, PE = mgh U,_,=PE),- PB, A negative value would indicate an increase in Potential Energy. ‘Types of Potential Energy include: Gravitational Potential Energy: Spring Potential Energy: Conservation of Energy ‘Conservative Case For a particle under the action of conservative forces: 15 GRE Physics Review (KE), + (PE), = (KE), + (PE), = E The sum of kinetic and potential energy at a given point is constant. Equation (1) can be written as: daw? > mv? + (PE) In a conservative system, if PE = £, then V = 0. non-conservative force F” d(PE+KE)=f F’- dr The Potential Energy must be less than or equal to the Total Energy. 1) The direction of F’ is opposite to that of dr. 2) Total energy E decreases with motion. 3) Friction forces are nonconservative. G. HARMONIC MOTION @ In a non-conservative system, relating potential and kinetic energy with the Simple Harmonic Motion — Linear motion of a body where the accelera- tion is proportional to the displacement from a fixed origin and is always directed towards the origin. The direction of acceleration is always opposite to that of the displacement. Equation of motion me’ + k= 0 or x74 pr=0 where p? = k/m. General Solution of Equation (2) x=¢, sin pt +c, cos pt where c, and c, may be oblained from initial conditions. 16 Q) Classical Mechanics ‘An alternate form of Equation (2) x=x, sin (r+ 9) where x, = the amplitude, and xex, sin (pt 9) Period =1 = 2n/P Frequency, f= 1/1 = P/2x For small angles of vibration, the motion of a simple pendulum can be approximated by simple harmonic motion. Mille For small angles of vibration, g=s/1 ‘Equation of Motion: ore ono The solution is: $= 4, c08 (ot +0,) where @, = /git 6, = max amplitude of oscillation a, = phase factor The period of oscillation is: 7 GRE Physics Review ‘The Spherical Pendulum refers to the simple pendulum-like arrangement, but with motion in 3-dimensions. ‘The equations of motion become: x74 (g/De=0 y+ (Q/Dy=0 with solutions: x= A.cos (or +r) y= Bcos (r+ 8) where @ = 4/g/I On the x-y plane, the motion is an ellipse. ‘Spherical Coordinates — More accurate than the previous solution. Equations of Motion: ma, = F,= Mg cos @-T ma,=F,=-Mg sing *— w7 sing cos $+ gsing =0 7“ 4 (asin? 6) = ee Forced (Driven) Harmonic Oscillator. For periodic driving force: Equation of Motion: fe. o% + 2 cose, 1 ae TH C08 @, where — @, = driving frequency F, = max. magnitude of the driving force. 18 Classical Mechanics The solution will be equal to the sum of the complementary solution and the particular solution. p Fofm X = Acos of + Bsinwt + —"— cosa,t to? f oo, oe Xe Pp Resonance occurs when «,~ co. Damped Oscillator. Acommon damping force is a pace a If an object’s motion is damped in this manner, then the equation of motion is: am, f(<) £ om (5) m The critical damping coefficient is defined as Corea = aml Now, three cases must be considered with respect t0 Corea? A) Ifc>Coyoys 4, and a, are both real, the motion is nonoscillating, and the system is overdamped. The general solution is given by xede! 4 Be? B) 19 GRE Physics Review ©) fe —~ %, and is known as the generalized force. Conservative Systems Forces expressed in terms of the potential energy function: =v F, where v is the potential energy. In terms of the generalized force, dy Ove “Ox, 99, ~~ 34, 23 GRE Physics Review LAGRANGE’S EQUATION For a system, kinetic energy KE is T=KE=$mx; §51,2,..,3M ax, | ax, Eee ee where ‘The Lagrange equation of motion using the equations above is; 4/2). oT 5 4 (a) -2 +H RAND or if the motion is conservative and if the potential energy is a function of ‘generalized coordinates, then the equation becomes 4 (7) 0 Ww pete at \Qq7,) 94, 9g, Lagrange’s Function (L) L=T~—V where T and V are in terms of generalized coordinates. Lagrange’s Equation in Terms of L 4 (3) = dt aq’, Lagrange’s equation for non-conservative generalized forces: -o. If 2,=2 %, , where Q’ is non-conservative, then Lagrange’s equation becomes fd oi 4g OL aq, 8 ** Og, and is useful, for example, when frictional forces are present, General Procedure for Obtaining the Equation of Motion: A) Choose a coordinate system. B) Write the kinetic energy equation as a function of these coordinates. C) Find the potential energy, if the system is conservative. D) Combining these terms in Lagrange’s equation results in the equation of motion. 24 Classical Mechanics Lagrange’s Equations with Constraints Holonomic Constraint — Constraints of the form oq 34, *7=° Non-holonomic Constraint — Constraints of the form h,3q,=0 Differential equations of motion by the method of undetermined multipli- ers: (The Non-Holonomic Case) ‘Multiply the equation by a constant A and add the result to the integrand of | Bq, de=0 Cle aa, Select A. such that the terms in brackets equals zero, Beek Bh oane0 at 2m Ag, =0 ‘There now exist m + 1 equations to obtain m + 1 unknowns, ode ‘This technique may be employed with moving constraints or with several constraints by having corresponding undetermined coefficients with correspond- ing A's in the Lagrangian equations, Qs ap ove ” GRE Physics Review CHAPTER 2 ELECTROMAGNETISM A. ELECTRIC FIELDS Definition of an Electric Field FE Eq where E = Electric Field F = Electric Force 4 = Positive Test Charge 4 1, COULOMB'S LAW By definition, the force between two point charges of arbitrary positive or negative strengths is given by the Coulomb's law as follows: 2,2, 7 4ned Fok where Q, and Q, = positive or negative charges on either object in coulombs. = distance separating the two point charges. the constant of proportionality (4ne,)“' = 9 x 10 newton-meter* / coul, permittivity in free space 8.854 x 10" Fm NOTE: € = ¢, €, for media other than free space, where €, is the relative permit- tivity of the media, The force F can be expressed in vector form to indicate its direction as follows: 2,2 26 Electromagnetism ‘The unit vector a, is in the direction of d =pfat Tal @ Naturally, Q, and Q, can each be either positive or negative. As a consquence of this, the resultant force can be either positive (repulsive) or negative (attrac- tive). Flux By definition, the electric flux, y (from Faraday’s experiment), is given by v=o where Q is the charge in coulombs. ‘The electric flux density D is a vector quantity. In general, at a point M of any surface S (see figure), Dds cos @ = dy, (where ay is the differential flux through the differential surface ds of M, and @ is the angle of D with respect to the normal vector from ds). (NOTE: is the case where D is normal to ds and the direction and magnitude of the electric flux density varies along the surface.) normal vector 2. GAUSS’S LAW Gauss’s law states that the net electric flux passing out of a closed surface is equal to the total charge within such surface. Hence, since dy=D-ds y=] D-as and by Gauss’s law, where Q,is the total number of charges enclosed by the surface. 27 GRE Physics Review Application of Gauss’s Law ‘The following spherical surface is chosen to enclose a given charge to be determined: Q, spherical i: closed Br*Bn surface ‘ x e Applying Gauss’s law. The charge Q, enclosed by the spherical surface is 2.=$D- ds where ds in this case is equal to 4nr* (NOTE: r is the radius of the sphere). Hence, Q,= Dan? and _ 2 Oo ar Since electric field intensity E is equal to 2, 4ne,d and d is equal to r in this case, then D = ¢,E. Some hints for choosing a special Gaussian surface: A) The surface must be closed. B) D'remains constant through the surface and normal to the surface. C) Dis either tangential or normal to the surface at any point on the surface. It is easier in solving a problem if we can choose a special Gaussian surface. In other words, this surface should be chosen to conform to the flux at any given point on the closed surface about the charge. 3. ELECTRIC POTENTIAL, ENERGY AND WORK Electric Potential Difference Wan Vem To units: Volts 28 Electromagnetism where V, = Electric Potential at Point B V, = Electric Potential at Point A Wy, = Work Done by External Force Go = Electrical Test Charge More generally: ‘The potential difference between two points p and p’, symbolized as V,,, (or $p’p) is defined as the work done in moving a unit positive charge by an external force from the initial point p to the final point p’. Vp=-f, E-d=V,-V, ‘The unit for potential difference is the Volt (V) which is Joules/coulomb. B. CAPACITORS ‘The Capacitance of two oppositely charged conductors in a uniform dielec- tric medium is Q cey, ‘The unit for capacitance is the Farad (F) c Fey where Q = the total charge in either conductor. V, = the potential difference between the two conductors. EXAMPLE: ‘Capacitance of the parallel-plate capacitor: Hl #4. onductor surface Po tt tt + tee0/ is the permittivity of the homogeneous dielectric D=p, (On lower plate: D,=D,=p, GRE Physics Review D, is the normal value of D. On upper plate: considering conductor planes of area S are of linear dimensions much greater than d. Total energy stored in the capacit Af ek Swell vy Ee ae as 1 == CV, 2 ov,= O g C x= vi- Multiple dielectric capacitors SL / nes plates: Cap ‘A parallel-plate capacitor containing two dielectrics with the dielectric interface parallel to the conducting plates; C= 1K (d,/ €,S) + (d,/ €,5) }. Electromagnetism 2,8 g=t a, V, = A potential difference between the plates = £4, +E 4, sl a) +e) C. CURRENT AND RESISTANCE DEFINITIONS Current: i= amperes where i = Electric Current q = Net Charge t = Time Current Density and Current: = Am ‘m? I*q iperes/ where j = Current Density i = Current A = Cross-sectional Area. Mean Drift Speed: vet ne where v, = Mean Drift Speed J = Current Density n = Number of atoms per unit volume. ci] GRE Physics Review Resistance: R=~% Qhms (9) where R = Resistance V = Potential Difference i = Current Resistivity: one Ohm- meters (Qm) where p = Resistivity E = Electric Field j = Current Density Power: tp 2 VW? Pevi=PR=Y waus(w) where P= Power 1 = Current V = Potential Difference R = Resistance D. CIRCUITS Electromotive Force, EMF(e) where Current in a Simple Circult where © = Electromotive Force w = Work Done on Charge q = Electric Charge >lo i = Current € = Electromotive Force R = Resistance Electromagnetism Rraa = R, +R, +R,) Q (in series) R, Let. ) (in parallel) The Loop Theorem AV, +AV, + AV, . For a complete circuit loop EXAMPLE Simple circult with resistor Vyse-iR=+ir Then e-iR-ir =0 NOTE: If a resistor is traversed in the direction of the current, the voltage change is represented as a voltage drop, ~ iR. A change in voltage while traversing the EMF (or battery) in the direction of the EMF is a voltage rise +€. Circuit With Several Loops Lin=0 EXAMPLE titind GRE Physics Review Muttiloop circuit RC CIRCUITS (RESISTORS AND CAPACITORS) RC charging and discharging Differential Equations R444 e=R 74% (Charging) O=R Bed (Discharging) An RC circuit Charge in the Capacitor a=(co(I- ) (Charging) a=(Ceye® Discharging) Current in the Resistor Je (Charging) (E)e# (Discharging) where — e = 2.71828 (Exponential Constant) Electromagnetism KIRCHOFF’S CURRENT LAW ‘The algebraic sum of all currents entering a node equals the algebraic sum of all currents leaving it. » Li,=0 = KIRCHOFF’S VOLTAGE LAW (SAME AS LOOP THEOREM) ‘The algebraic sum of all voltages around a closed loop is zero. THEVENIN’S THEOREM In any linear network, it is possible to replace everything except the load resistor by an equivalent circuit containing only a single voltage source in series with a resistor (R, Thevenin resistance), where the response measured at the load resistor will not be affected. Tinear_ -—x Active hteework y Procedures to Find Thevenin Equivalent: 1) _ Solve for the open circuit voltage V,_ across the output terminals. Vie Vu 2) — Place this voltage V,, in series with the ‘Thevenin resistance which is the resistance across the terminals found by setting all independent voltage and current sources to zero. (i.e., short circuits and open circuits, respectively.) RLC CIRCUITS AND OSCILLATIONS ‘These oscillations are analogous to, and mathematically identical to, the case of mechanical harmonic motion in its various forms. (AC current is sinu- soidal.) SIMPLE RL AND RC CIRCUITS Source Free RL Circuit GRE Physics Review Properties: Assume initially i(0) = /,, di =Ri+L Sexo, A) ty =Ritl B) (ate =e", ‘t= time constant -4 ©) Power Pye PR=12Re we, D) Total energy in terms of heat in the resistor = W,="/, Lie Source Free RC Circuit a(t) —e cavity gx Properties: Assume initially v(0) = V, wey A) C+ EHO. B) v(t)= v(O)et/¥O = Ve-t/ FC, ot 4 in dt+ (R=0 it) = 1) THE RLC CIRCUITS Parallel RLC Circuit (source free) Circuit Diagram: KCL equation for parallel RLC circuit: av dt -i(t, c= Va -iec 2 and the corresponding linear, second-order homogeneous differential equation is dv idiv_ Cate Etp Ao Electromagnetism General Solution: ss 1 2 VaAe! +Aze?, 1 1y_1 ths if.) 4 Sa" oRe (are) Lc or S\a=-at a? - a," where 0: = exponential damping coefficient neper frequency 1 2RC and @, = resonant frequency 1 * Vie eritically damped vit) \derdampea (8) COMPLETE RESPONSE OF RLC CIRCUIT ‘The general equation of a complete response of a second order system in terms of voltage for an RLC circuit is given by, St Sat v@)=V, + Ae! +Be? forced response natural response = (Le., constant for DC excitation) NOTE: A and B can be obtained by 1) Substituting vat t= 0° 2) Taking the derivative of the response, i.e., av Pu0+s,Ac¥ 45,8 0% Grothe +5 where # at 1=0° is known. GRE Physics Review E. THE MAGNETIC FIELD Force in a magnetic field Fy=qvxB where F, = Force on particle due to field Charge on particle ¥ = Velocity of particle B = Magnetic field F. MAGNETIC FIELDS AND CURRENTS THE BIOT-SAVART LAW Hol sin od ee 4n r where B = Magnetic field », = Permeability constant i = Current through a wire 1 = Length of wire 1 = Distance from assumed point charge to a point in the magnetic field © = Angle between r and the direction of the element NOTE: Hp = 4mx1077 im Integral form: ip dl’ xa, Babel, ant where primed terms refer to points along the source of the field, AMPERE’S LAW The line integral of the tangential component of B is exactly equal to the current enclosed by that path. $B-dL=I Curl of a Vector Field. The curl of any vector is defined as a vector where the direction is given by the right-hand rule and the magnitude is given by the limit of the quotient of the 38 Electromagnetism closed line integral and the area of the enclosed path as the area approaches 0. H-dL (curl H), = lim eaaae asco AS, As, is the area enclosed by the closed line integral, and n is any component; this is normal to the surface enclosed by the closed path. STOKE'S THEOREM fF dl={, (VxF)-ds F is any vector field, s is a surface bounded by /. It gives the relation between a closed line integral and surface integral. By using the Divergence Theorem and Stoke's theorem we can derive a very important identity: v-VxAs0 where A = is any vector field, MAGNETIC FLUX AND MAGNETIC FLUX DENSITY B=, H, Bis the magnetic flux density in free space. ‘Unit of B is webers per square meter (wb / m*) or Tesla (T) a new unit. = 4 mx 10” H/m (permeability of free space) His in amperes per meter (A/m). $B as=0 ‘This is Gauss’s law for the magnetic field _— V-B=0 after application of the divergence theorem. This is the fourth and last equation of Maxwell. FARADAY'S LAW Faraday’s Law can be stated as follow: GRE Physics Review ‘The minus sign is by Lenz's Law which indicates that the induced emf. is always acting against the changing magnetic fields which produce that e.m-f. Faraday's law describes the relationship between Electric and Magnetic Fields. INDUCTANCE (1) Unit of inductance is H which is equivalent to wb / A. Applications Inductance per meter length of a coaxial cable of inner radius a and outer radius b, Ho qe g Him. A toroidal coil of N turns and JA, where RK = Mean radius of the toroid. Different expressions for inductance: Wy Le Pa eae 1 Plog (7% A) dy 1 LaF lf a xtpavs {A xm a] 1 Lea [asa bath (laaae #) 34 Electromagnetism ‘Mutual inductance between circuits 1 and 2, where MAXWELL’S EQUATIONS Auniliary equations relating D and E: Maxwell's equations in integral form: Na 92 2 7, N = the number of turns tana ha (#H, Hy) dv Lye ky ‘Maxwell's equation in differential form: vee VxHes+ VvV-D=p v-B=0 D=cE D=e,E+P B=yH B=y,(H+M) J=oE J=pU F= p(B +U xB) gE. au--| 2 ds $u-ai-r+) 4 2 ds $d-ds={ pa $B-ds=0 at GRE Physics Review ‘These four integral equations enable us to find the boundary conditions on B, D, H and E which are necessary to evaluate the constants obtained in solving ‘Maxwell's equations in partial differential form. ELECTROMAGNETIC WAVES Maxwell's Equations in Phasor Form: VxH,=joe,E, VxE,=-jop,H, V-E,=0 V-#H,=0 ‘Wave equations: VxVxE,=V(V-E,)-V'E,=-jo,VxH, =o71,¢,E,=-VWE,. VE,=- 01,5, ve _ WE, FE, FE. aoe Oy =e) En For vE, FE, dy? az? ie., E, independent of x and y. This can be simplified to vE =~ OWE, Ew az? E,= E,, cos[a(t — 2 /I,e,) ] and E,.= a am cos[o(t +24/¥&)] E,= value of E, atz=0,t=0. Velocity of the travelling wave: To find the velocity U, let us keep the value of £, to be constant, therefore Electromagnetism Take differentials; we have lis at— 7 dz=0 az aU in free space. 1 s Velocity of light = U = Jen" 3x10" m/s Hoe ‘Wave length The field is moving in the ling wave. Form of the H field: direction with velocity U. It is called a travel- If E, is given, H, can be obtained from VxE,=-jou,e,H, OE, . Fem on Ey BA nr =E,(-jofiere * °° is a constant where Sot where 1) = The intrinsic impedance: It is the square root of the ratio of permeability to permittivity and is measured in Q. n, = (22 =3770 £q ‘1, = Nof free space. GRE Physics Review The term uniform plane wave is used because the Hand E fields are uni- form throughout any plane, Z = constant, and itis also called a transverse electro- magnetic (TEM) wave since both the E snd H fields are perpendicular to the di- rection of propagation. uniform TEM wave. CHAPTER 3 ATOMIC PHYSICS A. RUTHERFORD SCATTERING Alpha p >. Particle @ Is the scattering angle and b Is the Impact parameter In this collision, it is assumed that the nucleus has sufficient mass as to not be moved by the alpha particle. In this case, the energy of the alpha particle stays the same. It follows that the magnitude of the momentum remains constant P\=P,=mv ‘Therefore by the Law of sines: Solving for the change in momentum: Ap = 2m sin( 2) It can also be shown that the scattering angle @ can be determined by the equation: where = the alpha particle energy. 45 GRE Physics Review B. ATOMIC SPECTRA ‘When an atomic gas is excited, it emits radiation at certain specific wave- lengths. This produces the gas’ emission line spectrum. When white Light (all wavelengths) is passed through a gas, the gas will absorb certain specific wavelengths of the light. This produces the absorption line spectrum. ‘Wavelengths in atomic spectra fall into spectral series. There are 5 such series and 5 similar equations: Lyman Series 1 1 Ler(t nu rt Balmer Series Paschen Series Brackett Series eR (3-3) 1256.7. Pfund Series Len(h-d) avant. n=k+1, k+2,k+3, Ris known as the Rydberg constant. R=1.097x10 m" 46 ‘Atomic Physics C. THE BOHR ATOM Classical physics is not adequate in describing the atom, due to the fact that in order to resist falling into the nucleus, an electron would have to whirl rapidly around the nucleus using the pull as centripetal acceleration. This motion would ‘cause the electron to radiate electromagnetic energy, thereby rendering the system unstable. A quantum solution must therefore be sought. Hydrogen Atom (1 electron) If viewed classically, the required electron velocity for stability, not con- sidering electromagnetic generation would be rd ‘The DeBroglie wavelength for any object in motion is ss = aa < Orbital electron wavelength h ane gr MeN ‘The conditions for stable orbit can be described as the situation when the circumference of the orbit contains an integral number of DeBroglie wavelengths. ‘A condition for orbit stability rhe nr, n= 142,30 with possible radii nbc, nme? n=1,2,3,.. ‘The aforementioned spectra correspond to differing discrete energy levels, 1 level for each electron radius r,. Using the theory of the Bohr atom, an explana- tion of spectral series can be found. é Bre or, Substituting r, it can be shown that (=) my? +44) 2 r 47 GRE Physics Review Ey ” ‘The Bohr model can be used to derive accurately the value of the Rydberg constant. Note, however, that the Bohr theory is not in fact a complete and eccurate description of what is occurring, and is limited in its applications. For a more complete and accurate picture, Quantum Mechanics must be used. E, D. THE LASER Light Amplification by Stimulated Emission of Radiation Properties 1, The light is coherent. (Waves are in phase) 2, Light is nearly Monochromatic. (One wavelength) 3. Minimal divergence. 4, Highest intensity of any light source. Many atoms have excited energy levels which have relatively long life- times. (10° s instead of 10“ s). These levels are known as metastable. ‘Through a process known as population inversion, the majority of an as- sembly of atoms is brought to an excited state. Population inversion can be accomplished through a process known as optical pumping, where atoms of a specific substance, such as ruby, are exposed to a given wavelength of light. This wavelength is enough to excite the ruby atoms just above metastable level. The atoms rapidly lose energy and fall to the ‘metastable level. Ruby Transition (energy loss Laser Transmission Atomic Physics ‘Once population inversion has been obtained, induced emission can occur from photons dropping from an excited metastable state to ground state. The photons have a wavelength equal to the wavelength of photons produced by each individual atom, The radiated light waves will be exactly in phase with the incident waves, resulting in an enhanced beam of coherent light. Hence the familiar Laser effect. 49 GRE Physics Review CHAPTER 4 THERMODYNAMICS A. TEMPERATURE The average kinetic energy possessed by the molecules of a body is re- ferred to as its temperature. The zeroth law of thermodynamics states this more formally as: : There exists a scalar quantity called temperature, which is a property of all thermodynamic systems (in equilibrium states), such that tem- perature equality is a necessary and sufficient condition for thermo- dynamic equilibrium, ‘The two main scales in use for measuring temperature are the Celsius (formerly known as Centigrade) and the Kelvin scales. On the Celsius scale, water boils at 100° (1 Atm) and freezes at 0°. The 100 equal divisions between these two points determine the size of the Celsius degree, and from this, the points above 100 and below zero can be determined. When a temperature is shown followed by a degree sign, but without an abbreviation to show the scale used (e.g., 25° instead of 25°C.), it is presumed to be a Celsius temperature. The degrees on the Kelvin scale are the exact same size as those on the Celsius scale, Kelvin, however, starts at absolute zero, hence it has no negative temperatures. A temperature in Kelvin is never reported with the degree sign (e.g., 300 K). To convert Kelvin to Celsius, simply subtract 273.15 from the Kelvin temperature. Put mathematically, C=K-~-273.15. For example, 300 K would be (300 - 273.15) = 26.85° C. Converting from Celsius to Kelvin is just the opposite of this. Two older scales of temperature measurement, Fahrenheit and Reaumur, are no longer in use, and hence need not be learned. 50 Thermodynamics B. COEFFICIENT OF THERMAL EXPANSION As a substance is heated, its molecules vibrate faster and move further apart, or the material is said to expand. This phenomenom is much more pro- nounced in gases than in liquids and solids. If the material is a solid, and one of its dimensions is much more pronounced than the other two (€.g., a tod), a coefficient of linear expansion, o,, can be determined, i., a value to determine the increase of the material in that one dimension only. For all other materials, a coefficient of volume expansion, o,, is used. For solids, the o., is equal to three times the o,. For liquids and solids the o, will vary somewhat with the temperature, for gasses (at constant pressure) it is usually equal to 0.00367/°C. EXAMPLE The oy, for glycerin is 5.1 x 10+ / °C. If 25.62 liters of glycerine are heated from 25° to 125°, what will its new volume be? V=V,+(,- 0,47), the new volume the initial volume, and the change in temperature. aT AT can be calculated as 125 — 25 = 100. Putting everything into an equa- tion, we get V=V,+(Vy 0-2 = 25.62 + (25 - 62 5.1 x 10. 100) 6.93 liters. The same method is used for the linear coefficient of expansion. EXAMPLE ‘The o, for aluminum is 2.4 x 10° / °C, If an aluminum needle is 20 cm Jong at 350 K, how long will it be at 100 K? ‘The formula, much like the above, is L=L,+(l,-0, AT) or L=20 + (20) (2.4 x 104) (- 250) = 19.88 cm, Note that in this instance, the AT is negative, hence the value decreases. 51 GRE Physics Review C. HEAT CAPACITY, SPECIFIC HEAT AND C, ‘The heat gained or lost by a body can be quantisized in its ability to do work. The unit commonly used nowadays is the Joule, which is defined as the enegy required to move a force of one Newton the distance of one meter. Older, and ont-dated units include the calorie and British Thermal Unit (BTU). The ratio of the change in heat (AQ) to the temperature change (AT) in an absorbing body is called the heat capacity, of C. Mathematically C=AQ/AT. EXAMPLE A statue made of a metal alloy has a heat capacity of 55.8 Joules/°C. How many Joules of energy will have to be absorbed to raise its temperature from 25° to 100°? T= 100-25 =75°, C=AQ/A4T, or 55.8=AQ/75, or AQ = 55.8 x 75 = 4,185 Joules, or 4.185 1. A more useful value is the specific heat, or c. Specific heat is defined as heat capacity per unit mass of body, or ¢ 4Q/(mAT), where m= the mass of the object. At ordinary temperatures and ordinary temperature ranges, specific heats can be considered constant. However, it must be specified if the specific heat is measured with constant pressure (c,) or constant volume (c,). For gasses. =RIM, where R = the universal gas law constant (8.314 J/mole °C), and M = the molecular weight of the gas Sometimes specific heats are given in. J/mole °C, rather than the standard J/ gram °C, hence the units need to be inspected carefully. EXAMPLE ‘The c, for lead is 0.1277 J/gm °C, and for water it is 4.186 Hgm °C. If a lead block of 75.00 grams is placed in 200.0 grams of water at 25.00°, and the final temperature of the water and lead is 28.1 1°, what was the initial temperature of the lead? (Assuming no heat is lost to the surroundings.) 52 Thermodynamics The fact that both c,’s are given in J/gm °C saves us the trouble of doing any conversions. The Joules lost by the lead will be equal to the number of Joules gained by the water, or This can be restated as: (AT + cp: Mpg = (AT » Cp My. 9+ The c,’s and m’s for each are given, the AT’s can be calculated. For water, AT = 28.11" 25.00 = 2.11°. For lead, the initial temperature is not known, it will be assigned the value X, hence AT = X — 28.11. Putting these values into the above equation, we obtain: (X= 28.11) (0.1277) (75.00) = (3.11) (4.186) (200.0). Performing the indicated operations we get: 2,603.7 = 9.578X — 269.2, solving for X yields X = 200.95°, which was the initial temperature of the lead. D. HEAT OF VAPORIZATION AND HEAT OF FUSION When a substance changes state, e.g., from a liquid to a gas, energy is absorbed by the substance without change in temperature. For example, for water at 100° to change to steam at 100°, it must absorb 2,260 Joules per gram of water. This value is known as the Heat of Vaporization, (L,) and is unique for each substance, The melting of a solid involves a similar process, Joules are absorbed by the substance, but rather than an increase in temperature, there is a change of physical state. Ice at 0°, to melt to water at 0°, must absorb 335 Joules per gram. This value, again unique for each specific substance, is known as the Heat of Fusion, (L,). EXAMPLE For mercury the boiling and freezing points are 358° and ~39°, respec- tively. The L, and L, are 2197 3/gm and 11.7 J/gm, respectively. The c, for tiquid mercury is 0.138 J/gm °C. How much energy would have to be lost by’ 1.00 kg of Hg vapor at 358° to become solid Hg at -39°? To calculate this, one needs to find: 1) The Joules to convert one kg of Hg vapor to one kg of liquid Hg. 2) The Joules lost to cool one kg of Hg from 358° to -39°. 3) The Joules lost to convert one kg of liquid Hg to one kg of solid Hg. Put mathematically, GRE Physics Review (im. L) + (m-c,-AT]+(m- 1) = [1,000 - 297] + [1,000 - 0.138 - 398] + [1,000 - 11.7] = [297,000] + [54,924] + [11,700] = 406,848 Joules = 407 kJ lost. Had the mercury been heated to change it from a solid to a liquid and then to a as, the procedure would have been just the opposite, and we would have been adding Joules. If a material changes directly from a solid to a gas (e.g., carbon dioxide), a similar term, Heat of Sublimation, (L,) is used, just as L, and L, E. CONDUCTION, CONVECTION AND RADIATION Heat can travel from one body to another by three processes: conduction, convection and radiation. In conduction, one body is physically in contact with another, and the heat travels directly. The thermal conductivity of a material tells how easily it can transport heat, Metals are usually good at this. Steel, for ex- ample, has a thermal conductivity of 4.60 x 10 ki/sec-meter-°C, whereas asbes- tos has a value of 8.37 x 10 kI/sec-meter°C. EXAMPLE Compare how much heat is transmitted per second by a steel plate 1.00 cm thick, and a surface area of 5,000 cm?, and a temperature of 300° on one side, and 25° on the other, to how much heat is transmitted by an asbestos plate of equal dimensions under the same conditions. Heat Transfer = Thermal Conductivity - Area - (T, -7;) / thickness For the stee! plate this gives (4.6 x 10) (0.500) (275/0.0100) = 632.5 kJoules/sec. Note that cm? and cm were converted to m? and m, For the asbestos plate we get (8.37 x 10) (0.500) (275/0.0100) = 1.151 KJoules/sec. Convection simply involves heating the air molecules around an object, which then travel, via air currents, to another object where they impart their acquired energy by collisions. Radiation involves the transfer of heat by photons, which can pass through a vacuum, A black body is one that absorbs all radiant energy that falls upon it, and emits radiation perfectly. The Stefan-Boltzmann equation states that E, the walls of power radiated by a black body, is equal to the Stefan constant (6) times the fourth power of the absolute temperature, times the surface area, A, or 54 E=Aar. ‘The value of & is 5.67 x 10* Watts/m*K~, A black, cubic body, 25.0 cm on the side is heated to 325°C. At what rate is energy radiated from its surface? First the surface area of the cube must be determined. Converting cm to m, we get 0.250? 6 = 0.375 m* = A. The absolute temperature is 325 + 273 = 598 K. Putting these into the equation, E=AsT. yields E= (0.375) (5.67 x 10*) (598) = 2,719 Watts. F. HEAT, WORK AND THE LAWS OF THERMODYNAMICS THE FIRST LAW OF THERMODNYANICS Recall that work is defined as a force acting through a distance, and is measured in the same units as heat. Therefore, one might correctly conclude that a transfer of heat can be made to do work. The First Law of Thermodynamics states that there is a constant amount of energy in the universe, which can be neither created nor destroyed; it can only change its form. It is this change from ‘one form to another that we observe and call work. Expressed mathematically, we have AU=Q-W where AU = the change in energy of a system, W = the energy spent in doing useful work, and Q = any energy added to the system. 1 Q > W, the energy remains within the system, i.e., work was done on the system. If Q < W, energy was lost by the system, and work was performed. When gasses are heated at a constant volume the added energy increases the internal energy of the molecules. However, when it is heated at constant pressure, the internal energy of the molecules is increased, but in addition, work is done by expanding the gas against the walls of its container. (Hence the different values for gases of c, and c,). Under isobaric conditions (constant pressure): Q=me,(7,-7), where m = the mass of the gas and W = (density) (V,-V,), where V = the volume of the gas. Work is also defined as W =m¢,-c)(,-T) 55 GRE Physics Review in such a system. In a constant volume process (isovolumic) W = 0, hence, Q=AU=me,(T,-T,). In an isothermal process (constant temperature) AU = 0, hence, Q = W. An adi batic process is one in which heat is not transferred to or from the system. As a result. 0 = 0 = AU + W. Hence. AU =-W. THE SECOND LAW OF THERMODYNAMICS The Second Law of Thermodynamics states that itis impossible for heat to travel of its own accord from a colder to a hotter body. As a result of this, the ‘maximum efficiency of any heat engine is given by the formula @-T) 7, = the temperature (Kelvin) of the reservoir which supplies the working substance, and T,, = the temperature of the reservoir to which the working substance is exhausted. THE THIRD LAW OF THERMODNYAMICS The Third Law of Thermodynamics states that it is impossible to reduce any system to absolute zero in a finite series of operations. Hence, a reservoir of absolute zero cannot be constructed, and hence, (by the above formula), a heat engine of 100% efficiency cannot be built. G. ENTROPY Many volumes have been written about entropy. To sum it up, for the Purposes of this book, entropy can be defined as the amount of disorder in a system. By the Second Law of Thermodynamics, this means that entropy will usually increase, or at the best, remain constant. It takes energy to keep things neat! By the Third Law of Thermodynamics, at absolute zero there would be no entropy. Common units of entropy are Joules/Kelvin, CHAPTER 5 QUANTUM MECHANICS A. WAVE FUNCTIONS AND EQUATIONS ‘The wave function y has no physical interpretation, but | |? for a body at a given place and time is proportional to the location of the body at that given time. Once ¥ is determined, the momentum, angular momentum, and Energy of the body can be determined. Wave Function: Y= A+iB where A,B = real functions. Complex Conjugate: Ye eA- iB If dealing with complex wave functions: Iweapey Conditions for a wave function: 1) J” |wfav #0=Votumey 2) + must be single valued. 3 & ay oe Wa must be finite, single valued and continuous. 57 GRE Physics Review Normalization: A wave function is normalized when | ¥ P is equal to the probability density, not merely proportional to it. Caviar =1 B. SCHRODINGER’S EQUATION W is not measurable, and therefore may be complex. Specify ¥ in the x direction: wea? where @ = 2nv vel then vedere’ E = anv = 2h eS For a free particle: iigipxy w=Ae To obtain Schrédinger’s Equation therefore: ey Ev=2* ayy 2m h OV 2, = ay ee EY=- 55% and p'¥ h ax? TIME DEPENDENT ONE DIMENSIONAL SCHRODINGER’S EQUATION av (2 ) Rao ae) TY? In 3 dimensions: 58 Quantum Mechanics i wi (ay aw aw ne (Hee BT at, +V (x,y,z) P STEADY STATE FORM (3-D) ay oy ow am - Serta ort Gre- vyv=0 C. POTENTIAL WELLS AND ENERGY LEVELS PARTICLE IN A BOX A particle trapped in a box with infinitely hard walls is the simplest Quan- tum-Mechanical problem. Equation (Schrddinger's) in the box: ay | 2m EY =0 ae AsV =0, V2mE N2mE Ps Fs W = Asin x + Boos ——. As¥=Oatx =Oandx=L Ys iw i? ¥, 1,1? wy 1y,t? Wave Functions Probability Density GRE Physics Review Normalized wave function of particle in box: - evint® ne Yee Jz sn 1=1,2,3,... D. HARMONIC OSCILLATOR Thete are 3 differences from a classical oscillator: 1, Allowed energies a discrete spectrum. 2, Lowest energy not E = 0, but E = E,. 3. May go beyond +A (tunneling) The Schrédinger equation for harmonic oscillator: This leads to energy levels 1 A 1 E,=(n+5) hv with Ey= > hv. E ey Eo Evenly spaced energy levels E. REFLECTION AND TRANSMISSION BY A BARRIER TUNNEL EFFECT According to classical mechanics, when a particle of energy E approaches a potential barrier V when V > E, then the particle must bounce back. In quantum ‘mechanics, there is a chance, though usually very small, that the particle could penetrate the barrier, 60 Quantum Mechanics According to Heisenberg’s uncertainty principle, b Ax Ap2 =. ” 2 If we are 100% certain that the particle is not inside the barrier, then we are saying that Ax = 0, Therefore, AP and E would be infinite, which is impossible. ‘Therefore, there must be some positive finite change of the particle penetrating the barrier. A particle energy E > V approaches potential barrier: Ay ad MDL DIL 2 WY aor WY a | some reflect, others go on. Wave functions and probability densities of particles in finite potential wells. (Particle can be found in well). BARRIER PENETRATION vi 61 GRE Physics Review " on , 2" py =0 ax?” with solutions ay ye W,=Ae' +Be ! Bye mite ¥y=Fe' +Ge ! Outside barrier: k= Meme on OR 2% Incoming Wave: w= Ae! Reflected Wave: ‘Transmitted Wave: as Wy, = Fe! ‘Transmission probability: Approximation: EXPECTATION VALUES As Schrédinger’s equation yields probabilities, an expectation value can be described by V(x, #), is the value of x that would be obtained by measuring the position of a large quantity of particles, described by a given wave function, ata given time, and averaging the results. For position: e fl x|¥P dr ‘The expectation value of either quantity such as potential energy, that is depend- ent on x and described by , can also be found. = fl fOo|¥f a CHAPTER 6 SPECIAL RELATIVITY A. TIME DILATION AND LENGTH CONTRACTION Time Dilation f= where f, = Time passed on clock at rest relative to observer. 'f = Time interval on clock in motion relative to observer. ¥ = Speed of motion relative to observer (relative motion) ¢ = Speed of light (3 x 10* m/s) For an object or observer in motion, less time passes in a given interval (10 the observer in motion) than for objects or observers relatively at rest. Length Contraction L=L where _L, = Length of object when at rest relative to observer L = Length when approaching speed of light relative to observer. ‘To an observer relatively at rest, as an object approaches the speed of light, its length contracts. B. DYNAMICS Relativistic Mass GRE Physics Review Relativistic Momentum Relativistic Newton's 2nd Law Fs 4 E 2 Mass and Energy Total Energy rc! m myc? + k (kinetic energy) Rest Energy Eame Total Energy 2 eee ? Btamgc+pic C. LORENTZ TRANSFORMATIONS Given two reference frames (x, y, z, t) and (x’, y’, 2’, ¢), when there is mo- tion of the second frame in the x direction. Not considering relativity: x= x— vt " jon? = Galilean Transformation} ” ver vst ‘To convert velocities in the “rest” frame to the moving frame, differentiate 2, y’ and 2” with respect to time Special Relativity If we take v, to be c, then according to the intuitively correct Galilean trans- vy, which is impossible since c is an absolute speed. ‘An alternate transformation can be proposed: x ak (x-vt) which can reduce to x — vt. Xin terms of x’: xeko tor), yay az Therefore x= R (xv) thot and rone(! ‘As light has the same speed for all inertial observers: x=ct and x =ct" ‘we can evaluate k and solve for x. te -wysate+( 80 xn okt vit for x to = ct, GRE Physics Review CHAPTER 7 OPTICS A. MECHANICAL WAVES. BASIC PROPERTIES One way of transporting energy from one point to another is thtough wave motion. There are two main types of wave motion. Transverse waves — the disturbance in the medium is transverse, or at right angles, to the line of motion of the wave. ‘The shape of a transverse wave is a sine wave function, like the one shown below. where 1 = wavelength (distance between adjacent peaks or valleys). A’= amplitude (distance between the central position and an extreme position). Longitudinal waves — the disturbance in the medium acts in the same direction that the wave is travelling. A longitudinal wave appears in a spring as shown on the following page. 66 Sound waves are an example of longitudinal waves. ‘The period, T, is the amount of time to complete one oscillation in all types of waves. The frequency, v, is the number of oscillations per second. In other words, frequency is the inverse of T for all types of waves. v ae T For all waves, the waveform travels a distance A in time T. Therefore, the speed of the wave, ¢, is x roy SUPERPOSITION PRINCIPLE When two wave-forms are travelling in the opposite direction along the same line of motion, and the waveforms collide, the resultant displacement at a point along the line of motion is simply the algebraic sum of the separate wave disturbances. This is referred to as the principle of superposition. However, the two waves will leave the collision exactly as they were before the meeting. ‘The superposition of separate waveforms is known as interference. Constructive interference — The resultant displacement is greater than the individual displacements. Destructive interference — The resultant displacement is less than the individual displacements. NL 67 GRE Physics Review REFLECTION Reflection occurs when a wave meets a boundary or a place where the ‘medium propagating the wave changes. Infinitely Massive Boundary Free End —> — | nn — — — <— Right String Density Greater Optics Lett String Density Greater BEATS ‘When two tones of nearly the same pitch are sounded simultaneously, the human ear encounters a sensation known as beats, For two sinusoidal waves with frequencies v,, and v, and the same ampli- tude, the equation for displacement (the beat equation) becomes ; [2acoran (2422 }foosan (4): ‘The beat frequency, V,, is B. RAY OPTICS RECIPROCITY PRINCIPLE If a ray travels from 1 to 2, a ray will also travel from 2 to 1 by the same route. (See figure following.) GRE Physics Review _<—_ REFLECTION AND REFRACTION Law of Reflection Law of Refraction where 0’, = Angle of Reflection ®, = Angle of Incidence ©, = Angle of Refraction n,, = Index of Refraction of Medium 2 with respect to medium 1, {Normal Incident Ray t ) aig: neflectes scorch JY) ovo nce Reflection and Retraction TOTAL INTERNAL REFLECTION Critical Angle, qc n nm sind. = 7 70 0, is the critical ahgle (Total internal reflection) C. THIN LENSES ‘Thin Lens — Any lens whose thickness is small when compared with its radius. Focal Point — The point at which all rays intersect. Focal Length — The distance from focal point to the center of the lens. Real Image — Light rays actually pass through the image location. Virtual Image — The image is formed by rays which appear to come from the location of the image. RAY TRACING Procedure for locating image given object location, lens type, and focal length. EXAMPLE Ray 1: Passes through the center of the thin lens and, therefore, its path is unchanged. Ray 2: Is parallel to the lens axis and, therefore, is deviated so that the 71 GRE Physics Review ray goes through the principle focus, F,,. Ray 3: Passes through F, and, therefore, is deviated parallel to lens axis, Result: Real image is formed at distance s* from lens. Other examples of ray tracing Convex Lens with Real Image fhe 1 ,Fy Focal points f= Poca tengen Convex Lens with Virtual Image axis Graphical locations of images for three different object distances and three thin lenses The other lens equation may be used fo find f, s or s° NOTE: For concave lens f <0; for virtual images s’ < 0, ‘The magnification of the image is s meas 72 D. INTERFERENCE YOUNG'S EXPERIMENT pinhole Screen 1 or tneasterente Esser Diffraction wave pattems for Young's Double-siit experiment INTERFERENCE MAXIMA AND MINIMA Maxima where Minima dsin@=(m—/,)d where m = 1,2, ... (minima) d = Distance between slits or point sources © = Angle between the midpoint of the two slits or point sources and a point on the screen, 2 = Wavelength E. DIFFRACTION SINGLE SLIT DIFFRACTION asin @=md 73 GRE Physics Review where m = 1,2,3,...( a = Width of slit © = Angle of diffraction A = Wavelength DIFFRACTED INTENSITY where a a I, = Diffracted intensity J, = Maximum intensity a = Width of slit 1. = Wavelength © = Angle of diffraction CIRCULAR DIFFRACTION , a sin® = 1.22 4 where © = Angle of diffraction 2 = Wavelength @ = Diameter of circular aperture or lens DOUBLE SLIT DIFFRACTION 1, = Ia(cos B)* (882 B= Being » 74 sino e u Diffracted intensity Maximum intensity Distance between centers of slits = Width of slit Wavelength = Angle of diffraction orsarn 74 MULTIPLE SLIT DIFFRACTION Maxima dsin@=md where m = 0,1,2,... d = Distance between centers of. Angle of diffraction ‘Wavelength Angular Width 2 40m = Wie where A@, = Angular width of the maxima X = Wavelength . N = Number of slits d = Distance between the centers of slits ©, = Angle of diffraction for the principal maximum DIFFRACTION GRATINGS Dispersion a dcos® where _D = Dispersion (Angular Separation) m = Order of Maxima © = Angle of Diffraction Resolving Power Resolving power ‘Mean wavelength Wavelength difference ‘Number of rulings in the grating Order of maxima a zBer hauue GRE Physics Review TABLE OF INFORMATION Rest mass of the electron Magnitude of the electron charge Avogadro’s number ‘Universal gas constant Boltzmann's constant Speed of light Planck’s constant ‘Vacuum permittivity ‘Vacuum permeability ‘Universal gravitational constant Acceleration due to gravity 1 atmosphere pressure lal 1 angstrom Ba OF RP = no eae wound 1 weber/m? Notations Vectors — bold letter Unit vectors — usually the bold letters Planck’s constant Derivatives — indicated by primed symbols (Whether the derivative is with respect to time 4 or position 4 is apparent from the context of the problem.) 76 9.11 x10” kilogram 9.11 x 10 gram 1.60 x 10°? coulomb 4,80 x 10" statcoulomb (esu) 6.02 x 10” per mole 8.314 joules/ (mole: K) 1.38 x 10” joule/K 1.38 x 10'S erg/K 3.00 x 10" m/s 3,00 x 10" cm/s 6.63 x 10 joule-second = 4.14 10" eV second 8.85 x10" coulomb?/(newton-meter*) 4n x 107 weber/(ampere-meter) 6.67 x 10 meter’/(kilogram-second?) 9.80 m/s? = 980 cm/s? 1.0 x 10° newton/meter? 1,0 x 10° pascals (Pa) 1x 10 meter 1 tesla = 10" gauss The Graduate Record Examination in PHYSICS Test l GRE Physics THE GRADUATE RECORD EXAMINATION IN eX Awa wee PHYSICS TEST 1 -— ANSWER SHEET @®®DOO® D®DOO®D ®DOOO® ®OOGO ®DOOOO DOOD . O©OOD®O ®OOO® ®®OOD® ®D®OGQOH || DOOOO® . DOOO® DOOoOO | DOOO®O . DOODO . DDOOD®D . DOOD } DDOO®D . DDOOD® ®DOOOOD | DOOO®D ®DOOO®D . DOOOO DOOOOoO . DOOODOD B®OO®D | DOOO® ®®OOO®G . DDOOD®D . D®OO® - O®DOO® / D®OO® . DDOO®D 34 35. 36. 37. 38. 39. 40. Al. 42. 43, 44, 45. 46. 41. 48, 49, 50. 51 52. 53. 54. 35. 56. 57. 58. 23% RAWLS 3 ®®OO® ®DOOQD® ®OOGQ®D DOOD ®OOO®D ®DOOODGD . DDOO®D ®D®®OOO® ®OOG®D ®OOQ® ®O®OGQ®D ®O®OOGO® ®B®HOOO ®®OO® ®OOGeO ®OOO® ®DOO® |. DDOOD® ®D®OO® ®ODOO® ®OOGO® ®OODOD ®OOO® DOOOD® ®OOOD®D ®OoOO®S ®OOGDO | DOOO®D ®BOOOO ®DOOO®D ®OOO®D ®®OOOG ®DOOO® |. DOOODO ®®OO®D @®®DOOG ®®OO®O |. DOOD ®DOOO®G . DOOOOD 1 DOOO®D ®OOO® ®OOO® ®©OO® ®OOOO ®®OO® ®O©OOO ®®OO® ®POOOO ®OOGO@ DDOODO® ®®OOD ®OOOO ®O©OO®G ®®OO® ®OOO® ®OOOO@ ®OOGO ®@®OOO@ . DDOO® ®®OOG . DOOOD®D ®D®DOO® @®®OO® ®®OO® 9. ©@OOOO D®BOOO GRE PHYSICS TEST 1 TIME: 170 Minutes 100 Questions DIRECTIONS: Each of the questions or incomplete statements below is followed by five answer choices or completions. Choose the best answer to each question. It is possible that the Newtonian theory of gravitation may need to be modified at short range. Suppose that the potential energy between two masses mand m’ is given by V(r) =- SE ery. For short distances r << Ay calculate the force between m and m’. ) Gmm' |r? () F=-Gmn'afr (B) F=-Gmm! (1-a)/r7 ©) F=-Gmm! (1-a)ir? (©) F=-Gmm! (1+a)ir ‘The Stem Gerlach experiment in quantum physics demonstrates the quanti- zation of spin. Sample data is shown in the figure. The conclusion is that I (A) _ the electron is a fermion and can have spin up or down (B) the electron has no spin GRE Physics (C) the electron can only have spin up (D) the electron can only have spin down. (E) the electron is a fermion and can have spin 3/2, 1/2,~ 1/2, ot ~ 3/2 3. By the early 1900's the two major theories of physics were Maxwell's equations and Newton’s laws. The transformation from a lab reference frame to a moving reference frame was done by the Galilean transforma- tion. Which of the two major theories is invariant under these transforma- tion equations? (A) Maxwell’s equations (B) neither Maxwell's equations nor Newton's laws (C) both Maxwell’s equations and Newton's laws (D) Newton's laws (E) the invariance is not important 4, In elementary nuclear physics, we learn about the Fermi gas model of the nucleus. The Fermi energy for normal nuclear density p, is 38.4 MeV. Sup- pose that the nucleus is compressed, for example in a heavy ion collision. ‘What is the dependence of the Fermi energy on density? “A (B) itis independent of density Normal density © - Q Compressed region ® ep 5. Consider that a coin is dropped into a wishing well. You want to determine the depth of the well from the time T between releasing the coin and hearing it hit the bottom. Suppose T = 2.059 s. What is the depth fof the well. (A) 20.77m Test 1 @) 19.60m © 23,564m ©) 18.43m Uy a ©) 39.20m A 10 g bullet is fired into a 2 kg ballistic pendulum as shown in the figure. ‘The bullet remains in the block after the collision and the system rises to a maximum height of 20 cm. Find the initial speed of the bullet. (A) 28.0 m/s @) 23.8 m/s C) 3.98 m/s @) 719 m/s a ©) 398 m/s A horizontal beam of length 10 m and weight 200 N is attached to a wall as shown. The far end is supported by a cable which makes an angle of 60° with respect to the beam. A 500 N person stands 2 m from the wall. Determine the tension in the cable. x (A) ON (®) 700N © 500N it @) 231N rN ©) 808N ae ‘When a 4.0 kg mass is hung vertically on a light spring that obeys Hooke’s nw, the spring stretches 2.0 cm. How much work must an external agent do to stretch the spring 4.0 cm from its equilibrium position? (A) 1375 @®) 0393 at GRE Physics 10. 1. 12, (C) 0.207 @) 3.145 © 07s A cylinder with a moment of inertia /, rotates with angular velocity @,. A second cylinder with moment of inertia /, initially not rotating drops onto the first cylinder and the two reach the same final angular velocity a. Find or i (A) o =0, ®) e= 0,1, © o=1oft,+1) ©) o=0,1/), Au, ® o-0,0,+1), Three moles of an ideal diatomic gas occupy a volume of 20 m’ at 300 K. If the gas expands adiabatically to 40 m’, then find the final pressure. (A) 1.62 x 10°d/em? @) 1.82 x 10°d/em? (B) 1.42 x 10°d/em* ®) 3.74 10°d/em? (C) 2.84 x 10°d/em* Calculate the specific heat of a one cent copper coin using the law of Dulong and Petit. (A) 0.047 cal/g.K ) 0.27 cal/gK CB) 10cal/gK (©) 0.094 cal/g.K (©) 0.54 cal/g.K The tire of an automobile is filled with air to a gauge pressure of 35 psi at 20° C in the summer time. What is the gauge pressure in the tire when the temperature falls to 0° C in the winter time? Assume that the volume does not change and that the atmospheric pressure is a constant 14.70 psi. Test 1 13. 14, 15, (A) 49.7 psi ) 31.6 psi @) 35 psi © 46.3 psi (© 14.7 psi Consider a cone, with a light bulb at the vertex, cut by a plane making an angle of 30° with base. The area of the plane inside the cone has an image of area s on the base. If the area of the base is A and the average light inten- sity at the base is J, what is the average light intensity on the inclined plane? (A) ®) In. 8 a V3IA One of the following lists the four fundamental forces of nature in order of increasing strength. Choose the correct letter. (A) gravitational, weak, electromagnetic, nuclear (B) weak, electromagnetic, nuclear, gravitational (C) electromagnetic, weak, gravitational, nuclear (D) weak, gravitational, electromagnetic, nuclear (E) nuclear, electromagnetic, weak, gravitational A metallic chain of length L and mass M is vertically hung above a surface with one end in contact with it, The chain is then released to fall freely. If x is the distance covered by the end of the chain, how much force (exerted by the bottom surface) will the chain experience at any instance during the process? GRE Physics 17. (A) N=Mg-Mx” (8) N=3Mg (©) N=Mg-2Mx” @) N=@GM/L) gx ©) N=Mg ak A sphere of radius p and mass m is constrained to roll without slipping on the lower half of the inner surface of a hollow, stationary cylinder of inside radius R. ‘The Lagrangian Function is: (A) ‘701 (RE -2Rp+2p*)—ma(R—p)eos® @) 207 (an? —6Rp+sp?)—mg(R—p)eos® © 70 Gn? —6np +267) + matRp)sind @) J mo’? (R? —2Rp +p") + me(R- p)eos® ® 0" (an? ~6Rp+ 59%) +me(R—p)cos0 ‘Two equal masses m, = m, = m are connected by a spring having Hooke's constant k. If the ‘equilibrium separation is /, and the spring rests on a frictionless horizontal surface, then derive , the angular frequency. A) Vim @) 2Vkim ®) V2kim ® vel, © V3kim K SCOOT m, ma Test 1 24. What defines a conservative force? (A) PF-dA=0o0rV-F=0 (B) the force must be frictional (C) the force must be nuclear (D) the force must be electromagnetic © $F-dr=00rVxF=0 Consider a particle of mass m at temperature T which follows classical Maxwell-Boltzmann statistics, Find the average speed (v). A) V3erim ®) Vim = © Vite . ©) VeirTnm ©) Viti A 100 g mass attached to a spring moves on a horizontal frictionless table in simple harmonic motion with amplitude 16 cm and period 2 s. Assuming that the mass is released from rest at ¢ = 0 s and x = — 16 cm, find the dis- placement as a function of time. (A) x= 16 cos (nt) Y (B) x =-16 cos (nt+n) (©) x= 16 cos (nr +n) () x=-16cos 2nr+m) x=o! ®) x= - 16c0(5) Consider a simple laboratory experiment where the length and width of a rectangle are measured / = 5.45 + .05 cm and w = 3.86 + .02 cm. Find the uncertainty in the area A A. 85 GRE Physics 22, 23. (A) 0.05 cm @) 022cm (B) 0.02cm ©) 0.120 (©) 0.035 cm ‘Suppose that a man jumps off a building 202 m high onto cushions having a total thickness of 2 m. If the cushions are crushed to a thickness of 0.5 m, what is the man’s acceleration as he slows down? “™ s f (B) 133g ©) 5g ©) 28 ©) 266g A ball is thrown horizontally from the top of a tower 40 m high. The ball strikes the ground at a point 80 m from the bottom of the tower. Find the angle that the velocity vector makes with the horizontal just before the ball hits the ground. (A) 315° @) 90° (B) 41° ©) 82 © o A wheel 4 m in diameter rotates with a constant angular acceleration rad/s*. The wheel starts from rest at t= O s where the radius vector to point P on the rim makes an angle of 45° with the x-axis. Find the angular posi- tion of point P at arbitrary time ¢. (a) 45° (B) 45 +2P degrees (C) 45+ 114.67 degrees (D) 4# degrees ©) 229.2 degrees Test 1 2s. 26. 27. A common laboratory experiment involves the thermionic emission of electrons from metal surfaces. Use the Richardson-Dushman law to esti- mate the thermionic emission current density for a tungsten filament at 2000 K. Take $ = 4.55 eV as the work function and A, = 120 A/cm?.K? as the Richardson constant. (A) 0.00083 A/cm? @) 0.00146 A/cm? (B) 0.00104 A/cm? (E) 0.00166 A/em* (C) 0.00125 Afem? A source emits sound waves with a uniform power of 200 W. At what distance will the intensity be just below the threshold of pain? (Assume / = 1 W/m). (A) 15.92m () 7.07m @) om ©) 399m © 798m The electric field of a plane EM wave travelling along the z-axis is E=(,,x+E,, y) sin (ot—k +). Find the magnetic field B. (A) CE, x+E,, 9) 008 (tke + 4) Ie (B) &,,x+£,,y) sin (@r- kz +9)/¢ ©) CE,x+E,, 9) sin (@r— le +o)/c @) E,,x+E,, y) cos (wr- kz + $)/c © CE,x-E,,y) sin (or-iz +9) / Rays from a point object are incident on a prism of index of refraction n (shown) at near normal incidence. Calculate the de- viation angle of the rays. 87 GRE Physics 29. 31. a a @) @-05)0 ®) @-Dea ® (+050 (©) (n+ia Find the amount of horizontal deflection of a particle falling freely from height h in the Earth’s gravitational field. Let A. be the latitude and @ be the Earth's rotational frequency. A) 1/3@c08(2) 4/81 /g (D) @R,c08(A)a/2h/g (B) 2wcos(Ayh ® wcos(r)n/ 8h? /g (C) @c0s(A)q/Bhg Calculate the centripetal force required to keep a 4 kg mass moving in a horizontal circle of radius 0.8 m at a speed of 6 mys. (F is the radial vector with respect to the center). (A) 39.2N tangent to the circle (B) -30.0.N tangent to the circle (© 1440Nr @) -180Nr © 180Nr Find the tension T, in cord 2 for the system drawn below. The system is in (A) 19.6N (B) 39.2N ©) ON ©) 170N ©) 339N Test? 32. 33. 34, 35, ‘Tom has a mass of 70 kg and Susy has a mass of 60 kg. They are separated by a distance of 0.5 m at a wine and cheese party. What is the gravitational potential energy of the Tom-Susy system? (A) 112x105 @) -5.60x107F (B) 112x105 © 280x105 (C) 5.60 10-73 A wheel 4 m in diameter rotates on a fixed frictionless horizonal axis, about which its moment of inertia is 10 kg.m’. A constant tension of 40 N is maintained on a rope wrapped around the rim of the wheel. If the wheel starts from rest at ¢ = 0 s, find the length of rope unwounded at ¢= 3 s. (A) 36.0m (®) 720m T © 180m (©) 720m © 180m In Kepler's problem of planetary motion, various values of the eccentricity and hence the energy E classify the orbits according to conic sections. What value of the eccentricity € and the energy E belongs to a parabolic orbit? (A) e>LandE>0 (B) O—x © ® g Find the potential energy per ion for an infinite one dimensional ionic string of charges of magnitude e and altemating sign. Let the distance be- tween the ions be s. (A) -2ePin2y/ane,s ®) 0 (C) —e%/49 . . . © -eyAne,s ore @ ~ © esane,s A thin rod stretches along the z-axis from z= —d to z = d as shown. Let 2 he the linear charge density or charge per unit length on the rod and the points P, = (0, 0, 2d) and P, = (x, 0, 0). Find the coordinate x such that the potential at P, is equal to that at P,, a) 0 z ®) 4 d © v3 © V24 x © m2 -d A capacitor is constructed from two square metal plates of area L* separated by a distance d. One half of the space between the plates is filled with a substance of dielectric constant (x,). The other half is filled with another on GRE Physics 43. 45, substance with constant (x,). Calculate the capacitance of the device as- suming that the free space capacitance is C,. “ @) © ©) ® SCH, KY, +m) —_— (+e C, KK Coll, +) “| 2C,K,K, M(k, +) (K, +) C/2 Consider the circuit shown below. Calculate the effective resistance of the circuit and use this knowledge to find the current in the 4 Q resistor. “ (B) (©) Oy ® 0.25 A 5a 3a 12 050A Osa 12V = 10 4a 1.004 20 20 3a 125A Consider a plane wave travelling in the positive y-direction incident upon a block of glass of refractive index n = 1.6. Find the transmission coefficient. “ @) © @) ® 0.00 1.00 0.77 059 0.15 Which of the following Maxwell equations imply that there are no mag- netic monopoles? Tost? 47. (A) ®) © ©) VxB=p,5 Hee (E) magnetic monopoles have recently been found Consider a large ship of volume V, floating in a square pool of water such that a fraction f of the ship is below the water. Let the cross sectional sur- face area of the pool be A. How much does the depth h of the pool increase/ decrease if the ship sinks? (A) decrease by fV,/A (B) increase by fV,/A (©) unchanged (D) increase by V,/A ©) decrease by V,/A Consider an hourglass on a scale pictured below at times ¢ = 0, 0.001, and 1 ‘hour. What happens to the scale’s measure of weight of the hourglass plus sand combination as the sand falls? BBS t=0 t=.001 tel (A) the weight is constant (B) the weight decreases and then increases (C) the weight increases GRE Physics 49. 50. Si. (D) the weight increases and then decreases (©) the weight is unchanged ‘A wire 100 cm in length carries a current of 1.0 Amp in a region where a uniform magnetic Geld has a magnitude of 100 Tesla iu dhe a-direction, Calculate the magnetic force on the wire if @ = 45° is the angle between the wire and the x axis. (A) 70.72N @) -70.72N (8) 14142N ©) Osince is not parallel to B (© -14142N A permanent magnet alloy of samarium and cobalt has a magnetization M 50 x 10° J/T-m?. Consider two magnetized spheres of this alloy each 1 cm in radius and magnetically stuck together with unlike poles touching. ‘What force must be applied to separate them? (A) 74N @) WIN ®) 185N ©) 93N ©) 37N Recall the equation for a series RLC circuit. Compare this to the parallel resonant circuit shown and find R, if a series RLC circuit and the parallel RLC circuit are to have the same equations for the potential of capacitor while they both have the same L, C, and Q. (A) R,=R 1, l (B) R,=L oN © R=uC c 31 @) R,=LIRC (EB) R,=L/RC In a simple AC circuit involving only a resistor R = 50 Q and a voltage Test? 52. 53. 54. source V, find the linear frequency of the generator if V = 0.50 V, at time t = 1/720 s. (Assume V = Oat t=O also.) (A) 120nHz ©) 3400Hz B) 21,600 Hz ©) Onkz (©) 6OHz Use Ampere’s law to derive for the magnetic field of a toroid (Nv turns each carrying current 1) of inner radius a and outer radius b at a distance r midway between a and b. (A) Hy NI/2n(a +b) ®) p,NI/n (a+b) (©) HNI/nb da) ©) ble @+b) ©) 4H, NI/n (a+b) ‘Two parallel conductors separated by a distance r = 10 cm carry currents J, = 1.5 Amps and , = 2.0 Amps in the same direction as shown below. What is the force per unit length exerted on the second conductor by the first? (A) Hy lfm x) x B) Gy M2n)x ©) Uhhinxx @) CHA L/ex)x © Grilox x fe yy Tp Consider a series RL circuit with R = 10 Q, L = 10 pH, and V = 30 volts. ‘Suppose that / = 0 at r = 0, Find the energy stored in the inductor as t —> e. (A) 9.0x 10°F (B) 905 GRE Physics 55. 56. 57. © 455 @) 45x10°F ©) 15x10°5 Consider a circuit that consists of four resistors (each with R = 1 MQ), aca- pacitor (C= 1 WF), and a battery (V = 10 MV) as shown. If the capacitor is fully charged and then the battery is removed, find the current at ¢ = 0.5 $ as the capacitor discharges. (A) 40a y ®) 2A eB () 243A a Loe @) 147A Ly z ® 54a Two wires are bent into semicircles of radius a as shown. If the upper half has resistance 2R Q and the lower half has resistance R Q, then find the magnetic field at the center of the circle in terms of the current I. (A) - Gg M2a)z B) (Uyf/2a)z —- ©) -Wf/6a)z I ©) Hf /4a)z © ~(ugl/4a)z RQ ‘What is the magnetic field at the center of a circular ring of radius r that carries current /? A) gl 2r ®) pyf/2nr u © plier Test 1 61. ©) Bgl fer ©) itis equal to zero Consider a Fermi gas of electrons at low but non-zero temperature T. How does the thermal energy vary with the temperature? (A) proportional to T (D) proportional to T? (B) proportional to pT ©) _ proportional to 7/E, (P) (©) proportional to T?/E, (p) How fast must a 2 m stick be moving if its length is observed to be 1 m from the laboratory frame? . (A) length is the same in every reference frame (B) 0.866 cm/s ©) 1.50x10%cm/s D) 2.60 x 10%cm/s © 2.12 10'em/s In the classic Fizeau experiment to verify the special theory of relativity the speed of light in a moving liquid of refractive index n is measured. If the speed of the liquid is v, then what is the measured speed of light in the Inboratory frame? (A) cint+yv @) (int vl -vien) ®) cin @®e ©) (in+vy((1 + v/en) ‘An inertial system X’ is moving with a velocity 0.8 c relative to the inertial system K along the xx’ axes. A particle in K has velocity v= 0.5,V3/2)¢. Find the velocity of the particle in K’. 97 GRE Physics 62. 6. (A) v= at @= 120° (B) vw =catO= 60° (©) v’=0.92°%c at @ = 109° @) v'=092%cat@=71° © wv =05°% at @ = 30° x Light of wavelength 4500 A is incident on a Na surface for which the threshold wavelength of the photoelectrons is 5420 A. Calculate the work function of sodium. (A) 2.76eV @) 4.76eV @) 2.29ev ©) 1.00ev (©) 047eV Which of the following would be a true statement about the Frank-Hertz experiment? (A) the value of Planck's constant was first measured (B) the charge to mass ratio of the electron was measured (C) it was proved that atomic energy states are quantized (D) it was proved that the electron has spin (©) _ the quantization of photon energy was discovered In a Rutherford scattering experiment, 10 MeV a. particles are scattered by a gold foil 0.1 jim thick into a detector whose sensitive area is 10 cm? which is placed 50 cm from the target and makes an angle of 45° with the Test 1 65. 67. incident beam. Calculate the differential cross section in the center of mass system in barns (b) per steradians (SR). (1 millibam = 10 square meters.) (A) 2.65 bist @) 15.1 b/sr (B) 1.04 bjsr ©) 4.08 b/sr © 3.78 dsr ‘What was achieved by the recent discovery of the W and Z intermediate vector bosons? (A) the nuclear force was discovered (B) the gravitational and nuclear forces were interlinked (C) the proton will decay in 10" years (D) there are particles called rishons inside the quarks ©) the electroweak unification was verified Consider a mechanical model of the proton where the spin is due to its rotation, Assume the proton to be a uniform solid sphere and derive the equatorial velocity. (Assume m, = 1.67 x 10% gr, v, = 10"? cm) (A) 1.58 x 10" cm/s @) 1.87 x10" cm/s B) 3.00 x 10" cm/s ©) 7.88 x 10° cm/s (C) 3.9410? cm/s In quantum mechanics, one may picture a wave function in either momen- tum space or configuration space. If the wave function in momentum space is 6(p) = N/(p? + 02), then calculate the wave function in configuration space (aside from a multiplicative constant). Oat @ en (B) cos (Px/h) © © sin@x/*) 99 GRE Physics n. 100 Calculate the coefficient of reflection for a particle incident on a step potential with E> V,,. Let k= -V2mE |h, k= 4/2m(E-V,) /& A) R=0 Vix) (B) (C) « 5 Vo @) © Consider the hydrogen molecule H, as a rigid diatomic rotor of separation r = 1.0 A between two protons. Calculate the energy of the ¢ = 3 level in the rotational spectrum. (mm, = 938.280 x 10° eV/c? r= 1973.5 eV.Alc (A) 0.10ev ) 0.005 ev B) 005ev ©) 0.10eV (C) 0.15eV ‘An object of mass 0.4 kg is attached to a spring of Hooke’s constant k = 200 ‘N/m and subject to a resistive force — bx” where v = x’ is the velocity of the ‘object in m/s. If the damped frequency is 0.995 of the undamped fre- quency, find the value of b. (A) 3.20 kg/s (@D) 0.895 kg/s (B) 1.79 kg/s © 000kg/s (©) 1.60 kgs In the atom Na, the two levels *P,, and *P,,, are separated by 5.97 A in wavelength. Transition from these levels involves the emission of light of Test 1 B. 14. wavelengths 2, = 5889.95 A and A, = 5895.92 A. Calculate the value of the constant in the expression for the spin-orbit coupling. (A) O1ev @) 0.0001 ev @®) 001ev © 10ev (C) 0.001ev According to classical mechanics the atom will decay in a very short time. Roughly how long does it take for the electron to spiral into the nucleus as it emits electromagnetic radiation? (A) 10s @) 10*s ®) 10's “® 1078 © 10*s ‘The 1-meson has the same charge as the electron, but a greater mass m, = 207 m,, Use Bohr theory to find the radius of a p-mesonic atom with nucleus of charge Ze orbited by the - as compared to the radius of the hy- drogen-like atom. A) 3%y B) 1, =207 ry © +,=20Pr, D) 1,= 74/207 ® 1,214/207 X-rays with an energy of 50 keV undergo Compton scattering from a target. If the scattered rays are detected at 45° relative to the incident rays, find the energy of the scattered X-ray. (A) 514 keV @) 47.2keV B) 48.6 keV ©) 500kev (© 528kev 101 GRE Physics 15. 16. 71. TB. For collisions between identical particles, what is the relationship between the CM and the laboratory scattering angles? v; (A) e=y vt Kew Obab ®) 0="%,-W N\ © o=%+y Initial and final lab situation IS Uy @) e=y2 1" Mi Uy =Wy/2 »\ Oem © 0-2 Initial and final CM situation In the Mossbauer effect, the absorption by the absorber may be destroyed by moving the source. For the case of ‘Fe in a lattice where the gamma ray has energy 14.4 keV and lifetime 9.8 x 10% s, find the minimum source speed necessary to destroy the resonant absorption, (A) 0.028 cm/s @) 0.014 cm/s ®) 0.010 cm/s ) 0.007 cm/s (C) 0.100 cm/s Calculate the speed of a proton of kinetic energy 1 TeV in the Tevatron at Fermilab in Batavia, Illinois. (Use a Taylor expansion.) (A) B=0.999 999 6 @) B=09999 ®) p=10 () B=0.999 999 94 © B=0999 Calculate the Q value in MeV for the nuclear reaction” Al(d.p)*Al given that m("Al) = 26.98154, m, = 2.01473, m, = 1.00794, and m(*Al) = 27.98154, all inamu, (A) ~6.32 Mev @) 6.83 MeV (B) 0.0Mev (©) -6.83 MeV (©) 632 Mev Test! 79. 80. al. ‘The nuclear charge density as found from electron scattering is given by (7) = py(l + e*-), What is the meaning of the fit parameter b? (A) b= 1.12.4" gives the half density radius (B) itis merely a fit parameter and has no physical meaning (©) _ itis the classical electron radius b = ¢/m,c* (D) generally b =o andr < aso that p(r) =P, ©) 5 =0.6 fis related to the surface thickness ‘A simple wave function for the deuteron is given by $(r) = A sin[k(r— a)]/r fora a + b. Find the probability that the neutron and the proton are between r = @ and r = a + 6 in separa- tion, re(r) @) JT ogra B) A*(H/2-sincakbyak (C) A*/2 @) BYE" Y—e™) a a+b (©) 1.0 since the neutron and proton are bound together ‘Using the Boltzmann factor, calculate the percentage of hydrogen mole- cules in the first rotational level relative to the ground state at T = 300 K. Assume that r = 1.06 A is the appropriate molecular distance. (A) 75% @) 0% (B) 50% ©) 9% ©) 25% ‘According to the nuclear shell model, what is the proton configuration for the nuclideZ AI? (A) 1s*2s*2p'3st3p! GRE Physics @B) 1223s CO) As y_P 25 Pop)" B5ya)* Paa)? @D) (1s, MOP a VBdyg)® ©) 1s Py PP Adyg)* According to Bose-Binstein statistics, there exists a Bose condensate for collections of bosons. What does this mean? (A) as T—> © all particles reside in excited states (B) for T <7, all particles reside in the ground state {C) bosons are not physically meaningful particles (D) bosons are like fermions ©) for T pV’ = constant, thus Po =P,¥, Py=Po (7) where the exponent a Y= (Cy + RIC, = ER +R) / GRp-F since a diatomic gas has 2 extra degrees of freedom. 2a" = . Py = (3.741 X10 (21a 113 Detailed Explanations of Answers = 1.42 x 10'd/cm? 11. (E) The law of Dulong and Petit states that the molar heat capacity is cy=9R =3(8.314 x 10" —7_y—_al___ mol — k 4, 184 x 10 gre _ cal =5.96 mole Basically there are six degress of freedom that contribute to this molar heat capacity. To get the specific heat, we must use the atomic mass of copper 1mol) Ate) cal wol-k Cy =Cyx =5.96 =.094£2 ek Note that in the above equation, A is the atomic mass of C,. 12, (0) The absolute pressure p is the atmospheric pressure plus the gauge pressure. P=P4tPo = 35+ 14.7 =49.7 psi= ps Now use the ideal gas law Pw _ Tw PV =nRT , B= F Pe=. 13, (C) The inclined plane cuts out an area given by: Area= = —S__ = 28, ‘cos 6 cos 30° Let the average intensity at the inci Be x. Then: eo} ( =z) = (/A) =(constant output power of bulb). 114 Test 1 mn (ED 14. (A) ‘There are four known forces. In order of increasing strength, these are the gravitational force, the weak force, the electromagnetic force, and the nuclear or strong force. ‘Also, since Einstein, physicists have been trying to verify the forces in a unified field theory. 15, (D) The net total force exerted on the chain (by both the surface and gravita- mass density .-M L| %= near of the chain a. kK To find the equation of motion of the center of mass, according to the figure, we can write (all the distances are evaluated with respect to the hanging point): L- Emme MEHL ons + N: the normal force of the surface But x” = g since the chain is falling freely and also we have: xt=2gx (equation of motion with constant acceleration). So we have M = 3M N= Tags 2ery= AT x 115 Detailed Explanations of Answers 16. (E) Let T be the kinetic energy of the sphere. Let U be potential energy. Then the Lagrangian is given by: L=T-U 1 1 T= 5 mV +510" ‘on =(R- po = 48 dt 2 no? zm U = —mg(R -p)cos® Ls Fmr -pyo7+ 4 Se’ p74 mg(R - p)cos @ 2 L 7S car” —6Rp +5p") + mg(R- pcos @ 17. (B) In general 4 rod wos yo % x, and x, are the separation of the two masses from their equilibrium positions along the respective axis parallel to their path of motion. myx; =— ke where x= m,x5 = — ke Note: (x, the total compression or stretching of the spring, is equal to the alge- braic difference of x, and x,, Subtract the two equations to get, (xy 15) == Dee since m,=m me +2ee= 0, x74 hy =0 18. () A conservative force is a force such that QF dr=0 or VxF=0 116 Test! these are equivalent conditions since VxF=0 JVxF-da=QF-dr by Stoke's theorem, GF -dr=0 Stoke’s theorem relates the surface integral of the curl to a line imtegral of the original vector field. 19. () ‘The Maxwell-Boltzmann differential probability for speed is fv) dy = 4notem 7 ay then =f 40) av] ro dv We also have: nr) =f veeray Using integration by parts, we can write: u 1 du =(n = 1)v""*dv => v dyavestay) ya-sbes" av na? | 1 ow? p-2 ahs -f mae (n-1v"ay : a 1, =(0- n= [0-014 Le 2a Hence we can write: 17 Detailed Explanations of Answers Finally, ” 2 2a. e 2h 3h To 20. (C) ‘The basic simple harmonic motion equation is x= A.cos (wt + 8) A=16cm,T=25, ‘The linear frequency is then vel or v='/,Hz. Hence @=2nv = rrad/s is the angular frequency. Hence, at r= 0: =16 = 16cos (+8) 8 = mrad Therefore: x = 16cos (nt +m) 21, (D) From the theory of propagation of error: Az=lw HAY (Aa Y AA= (441) +(Har) = VowAy + amy TE = (5.45) (3.86) =0.22cm 22. (B) Use basic kinematics. vi=v,.=2a(x- x4) v=V2ax 118 Test 1 = (2(9.8)200 = 62.61 m/s vy? v9? = 2a(x—49) 0? - 62.61? = 2a(0.5-2) a= 1307 m/s? a=133g using g=9.8m/s? 23. (A) With standard kinematics, we get yetat?, t= a/2y/e = V80/9.8 = 2.865 xey,t, v= = B= 28.0 m/s Vz ~ V5 = 2a(y - Yo) V 207 =~ 4/29.8) (40) =- 28.0 m/s vy o= aim (%2) =a 24. (C) 6,=45° P = 2 5 180° a= 4 rad/s? x Oe A\ = 720/n deg/s* Now 0-0, +a¢+Lar is one of the basic rotational kinematics equations. 119 Detailed Explanations of Answers = 4504 3002 = 45°4 114.6997 25. (E) 74.55 0 AT = (1.381 x 107") (2000) Le 1.602 x 10°" erg = 0.1724 eV The Richardson-Dushman Law states that seat = (120) (2000 )%¢#s8/0.12 # = 1.66 x 107°A/ em? where A, = 120 A/em?. has been used. 26. (E) The power of the sound waves remains constant as they spread over the region. Since intensity is proportional to the inverse of the area which receives the power, we can write: Pp = 200W \ J T=PIA = P/4nr r Taking the area as the surface area of a sphere Lo -~ r=VP/4nt = 4/200 742) =3.99 m 27. (C) ‘The given electric field vector is E = €,x+E,y)sin (@—kz+$) QE, 9E, (= 3) VxE=| o> a tlar Taking the curl = E,,k cos(ot - kt + 6)x - E,, k cos(wor — kt +6)y __ 2B = Or 120 Test 1 By Faraday’s Law B=-J (VxE) dt integrating °y = (~Ey,x+E,,y) Esin(or — kz +6) 28. (B) and Taking the index of refraction Using the small angle ap- proximation, we get Use Snell’s law to get: sin , =” sin @, nsin @, =sin 0, of the prism to be 1. 0, =, and 10, ~ 0, From geometry & = 0,40, and 8 = @,-0,)+@,-8,) Note that 6 is the internal angle of the small upper triangle. = 10,-0,4+70,- 0, =(@-la 29. (A) Breaking @ down into its x and z components © = -@coshx+osinkz Use v= —gtz to find a= g-2@xv As the acceleration = g—2@cos(A) gtxxz and a, = 2@grcos (Ay is the Coriolis acceleration then v, = Ja, dt =cgrcos d y = Jy, de=, ge cosh i kai Fsinor — ee + o)x +E, Esin(or- + oy Figure not drawn to scale 124 Detailed Explanations of Answers ho Yj gh@t=2h/e thus yy =, @cos Asa? /g is the eastward deflection. 30. (D) Use Newton's 2nd Law Fe and the centripetal acceleration toget F= 2 46) =e 7180 F=-180N r 60° 31. (E) Ts Using the free body diagram and ZF = 0, ty we get 7 T, = mg=2(9.8)=19.6N i T, =mg =2(9.8) = 19.6N A second free body diagram is drawn where the strings meet. Ty T,, = T,=19.6N T, = T,,/sin 30°) = 39.2N T, = T,,=T, cos (30°) =33.9N Wa=mg Free body diagram 32. (D) Newton's universal law of gravitation states that Gm,m, r Fe-—! .~_____-. 7 my, m, Now integrate the force to get the potential energy. Gmim, s-|F drps-—12 (6.67 x 10 10) (60. -7 y= GX DY) ) 5 60x 1077 05 33. (B) ‘Use the diameter to find the radius Test 1 Now, Newton’s 2nd law for rotation gives Seer =rT sla rT__ (2) (40) + ae T 10 = 1 0=0,+0,14+ 5 sar0= Fore z 2G) =72.0m 34. (E) The various curves are shown in the figure below. ‘The parabolic orbit has € = 1 and total energy E=T+U=0 35. (A) Let d be the diameter of the pinhole. When the pinhole is large, then the object is imaged as a disc of diameter d’ = d. When the pinhole is small, then the Rayleigh criterion gives the image disc diameter in the following way: sin@ = 1.22Nd @=> X between light ray to the circumference of the image and the maximum intensity of the image. Now d/D =20 and for @<<1 rad sin @ ~0=1.22Nd thus d/D= 2x122xd/d or = d= V2.440D 123 Detailed Explanations of Answers 36. (C) ‘We would like a lens that brings objects from 25 cm to 100 cm. Note that 5’ is negative because the image is supposed to be located on the same side of the Jens as the object, s=+25em, s=- 100 cm dette a la 37. (D) ay Ay = [sin@ is the distance between the maxima. dsin@=) a Ay=l= 2 dad d 10" 92 =4py=!2 10 mee =0.5x10* m=S000A 38. (B) E=al/petemte® ho wal key + (me)? = aficy + (met /a by” vp Ewe + (met by” Sen) 1+ (meth ky Test 1 ‘The group velocity v= ae Ok is equal to the physical speed. 39. (E) V-E , According to superposition principle, we can evaluate the electric field produced by each sheet separately and then add them up, be- cause the presence of each sheet does not have any effect on the charge of the other. Therefore, to find the electric field of one of the sheets we can write: o o $e dane > 2 Now reduce the series resistor: Viti 3. 2 57. (A) Use apa to ldixe an the Biot-Savart Law where r is the unit radial vector. Only the z component of B is non-zero at the origin. (dl xr), =rd0 2 -(44) ¢ ae po LA 133 Detailed Explanations of Answers 58. (C) For a low temperature Fermi gas, it is true that St Ee E,@) is the thermal energy where n? (an? ’ £y(0)= #- (2 is the electron Fermi energy. 59. (D) ‘The Lorentz transformation equation is, x = ¥Q-v) or Ox’= yAx with Or =0. ‘The Lorentz contraction formula is l=], a y y 2=n v —- x x = 2.60 x 10cm /s 60. (C) ‘The liquid is at rest in K’ where the speed of light is v’ = c / n, In the lab frame we must add the velocity v’ to the liquid speed v, relativistically v,+V, 2 1+V,V,/c? = clint. 14% viet = tinty T+ v/a 61. (A) v= 0.5C, v= B=0.8 > v=0.8c 2 -¥ 0.5-0.8 —_ is 10.8) (0.5) ms -0.5e 134 Test 1 0-08) @5» e O= Aten vs = 180 - 60°= 120° Note that ? ved/vityy since the speed of light is invariant. 62. (B) By conservation of energy hy = O+T At threshold 1 hy, = gandv, = clr, e may be used to get ae 4 o = hes, = 12,400 eV.A /5420 A as = 2.29eV 63. (C) ‘The Franck-Hertz experiment definitively established the quantization of atomic energy states. In particular, the data looks like I 49 98 147 V (volts) 135 Detailed Explanations of Answers for Hg when one plots the beam current versus the accelerating voltage. The first excited state of mercury has energy 4.9 eV above the ground state, A decrease in electron current is observed every time the potential is increased by 4.9 volts, 64. kinetic energy in C.M. system T,, is kinetic energy in Lab system 6 is angle in C.M. system ‘wis angle in Lab system. Given: y= 45° and T, = 10 MeV Since the mass of an o particle is much smaller than the mass of a gold nucleus, #5-(g.) A$)" 2 - eon) sin-*(22.5°) 4q) fm? . 10mb = 1,509 Sp x one = 15,090 SP ~ 15.09 2 6s. (E) ‘Carlos Rubbia and his collaborators at CERN verified the theory which unifies electricity, magnetism, and the weak force. The mass of the W* or Z° is about 90 GeVic?. Electricity and magnetism were connected by James Maxwell into electromagnetism. The new unification is called the electroweak theory. 66. (E) ‘The moment of inertia of a sphere is Test 1 a> 3 4 ) / 0.673 x 10) (107) = 7.88 x 10° cm/s 67. (D) O@)=N/G' +a") ‘Take the Fourier transform to get via) = Jee [dp pete” ents N Trl pro para ?* De Neth geen = —_—- Ff Tins S Greer? ‘Use Euler's theorem e* = cos0 + isin® = c0s(px/h) worn Jf Baa {elena -N [24% 7h 20 be eaune N where we have used some knowledge of the Fourier cosine transform. Thus y(x) «ent 137 Detailed Explanations of Answers 68. (C) Vix) The Schrédinger equation is Hy=Ey where H = T + U is the Hamiltonian operator and £ is the energy eigenvalue. ‘The solution to this equation is et tre, x<0 k= V2mE/h VO=y te, x>0 k= /2m(E-V,) in ‘w(0) continuous => 1+r=for r=t—1 W‘(0) continuous => ik—ikr = ik’ t, Now r+ S(k-k)/(k +k) is the amplitude of the reflected wave and (k-k) (e+e) is the reflection coefficient. Rerts 69. (B) ‘The rotational energy eigenvalue is BE, = (6 4+1)/2pr where /is the angular momentum quantum number and r is the relative distance, h = 1973.5eV-A/c m, = 938.280 x 10° eV / =m, =m, —a ‘The reduced mass is 7 H = m,m,/(m, +m,) = m,]2-=469.140 x 10 eV /c? 138 Test 1 E, = (1973.5)? 3(4) / 2 (469.140 x 10°) (1)? = 0.05 ev 70. (B) By Newton’s 2nd law, EF = -ke- bx’ = mx” The equation of motion is then bey k 2 x x + xs 0 where w gives the square of the rational frequency Ae™P*cos(or +9) isa solution. b= -V0.040 fan = 4/0.040(200) (0.4) = 1.789 kg/s n. (©) Spin orbit coupling results in an energy change AE =aA(\-s) The total angular momentum is j= +s. fats al +st+2-s Now the eigenvalues of any angular momentum operator follow the rule Py=GU+Dy thus = 1-s=(j(j+1)-1(1+1)-s(s+1))/2 ‘The two states have: Gh) Hence, 2) respectively 139 Detailed Explanations of Answers (2-2-iy2 [2h lise 3 a {? G-2-52 40+ s)=G +1)= _ Ok ols (i-%) @) Using E=hv=hch = (12,400) (3) (sabor = 0.001 eV 72. (A) Larmor’s formula states that an electron circling a nucleus with centripetal acceleration a emits energy with rate e.2. ee aw P Gm, 30 dr From this it may be shown that the electron will spiral into the nucleus in about 10°", 73. (D) Using F = ma with a = v /r as the centripetal acceleration and F = fg, qo 7°48 the Couloumb force, we have my? _ kZe? rs? Fe 140 Test 1 r Woln= al my 207 # where r,, is the radius of the Hydrogen-like atom in Bobr theory. 74. (B) ‘The Compton shift is AR a NAD N-h = A,(1-cos 8) where A, = helm is the Compton wavelength = 12,400 / 511,000 . final = .02426A E = 50x10’ eV =hc/r_ A = he/ E= 12,400 / 50,000 = 0.2480 A A = A+ Ad= 2480 A + 02426 A (1- Fe) = .2551A Thus E’ = he/N = 12,400 / 2551 = 48.6 keV is the scattered photon energy. Note that the photon loses energy to the electron. 75. (E) ‘Consider the initial and final states in the two frames: — uy ey /2 Now u,=¥,/2 =¥,-¥,/2 hence, u,’=v,’—v,/2 141 Detailed Explanations of Answers as in the below figure. “Vy /2 O=yra by geometry and also any 0=2y since Iu’,! =a making the triangle isosceles. 76. (D) The Mossbauer effect is the recoilless resonance emission/absorption of nuclear radiation. For the "Fe case E, = 144keV, 1=9.8x104s is given. The energy width is T= b/e= (1.055 x10) / (9.8 x 10") (1.602x 10" erg/eV) = 6.72107 eV ‘One destroys resonant absorption in the lattice via the Doppler shift AE2T, $£,20 (6.72 x 10”) (2.998 x 10"") vy 20 ¢c/E, = ————____,— 14.4 x 10 = 0.014 cm/s 71. TeV = 10° Gev m, = 0.9038 GeV E=m,+T =10° + 0,938 GeV = 1000.938 GeV = E/m = 1000.938/.938 = 1067 However. Vitx =f0)+f/ Ox =14+ hx 142 using a Taylor expansion. Hence e1-4.21-4x107 z 2y' = 0.999996 Itis incredible how close to the speed of light one can get! 78. (C) Consider the reaction 1+TE+R Where I = incident particle, T = target, E = emitted particle, and R = residue which is also written TULE)R. The Q-value is Q = m,+m,—m,—m, ee (2.01473 + 26.98154 ~ 1.00794 — 27.98154) amu 931.502 MeV / amu 6.32 MeV 79. (€) ‘The parameter b is a measure of the surface thickness. The fall off distance is approximately the same for all nuclei. R. Hofstadter of Stanford University was awarded the Nobel Prize for these experiments with electron scattering. 80. (B) Asink(r-a)/r acra+b P=f-"yyra’r = J 09% dr J YY q*dQ where @ is the radial part of the wave function and ¥ the spherical harmonic an- gular pa = J A’ sintk(r -a)dr aafaet - = Af (Z - Foos2k(r - a)ydr aay sin 2k ja+b FAGrH gpsin2kr - a) 7 a HAUG Zpsin 2k) 143 Detailed Explanations of Answers 81. (A) ‘The rotational energy eigenvalues of the Schrédinger equation are: E, = jG+ DF /2ur The hydrogen atom consists of a proton and a neutron: my, = 938.280 + 511 = 938.791 MeV ‘The reduced mass is therefore H = mm, / (+m, =m, /2 = 469.396 MeV and r= 106A is the given relative distance r=1r,—F, |. AT = (1.38 x 10) (300) / (1.602 x 10°) = E,=0 E, = 2 /pr = (1973.50? / (469.396 x 10°) (1.06)? = 00739 eV NIN = 6o = 0.75 82. (E) The proton configuration is (18,)* (1 Py) (1 PyaY? (Ady, since there are 13 protons while the neutron configuration is (15,29 (1 Pya)* Cl Pia)? da since there are 14 neutrons. Only the open proton shell contributes to the nuclear spin. Hence, the spin of the 7A nuclide may also be deduced to be j = 5/2. 83. (B) The Bose condensation phenomenon occurs for low temperatures T less than a critical temperature T_ where all particles reside in the lowest state. 84, (C) Moseley discovered that the frequency of x-ray emission v is proportional to the atomic number Z of the emitting system. This follows quite naturally from quantum theory since E,=-13.6 eV Z4/nt is the energy eigenvalue for a Hydrogen-like atom and thus the transition energy 144 Test 1 E,-E,=hv « Z produces a photon of frequency v. 85. (C) ‘ In the Zecusan effcut, the frequency is shifted by an angular frequency Aw = £eB/2me = tp, B/h thus the energy shift is AE = hho = ty,8 86. (A) By definition of the exchange operator, P,Wd,2)= ye.) vid.2) a (WL,2) + WD) then P,, y'(1,2) = (11,2) 1 or Py W'(,2) = 87. (E) ‘The potential energy function is 2 x<-a/2 V(a)=4 0 -a/2a/2 ‘The Schrésdinger equation HY, = EW. with Hamiltonian H = T + U bas eigenvalues given by Wala) = cos(nmx/a) (WC, 2) + y(2,1)) 2 am _ enon? since ote 2m “de?” ~2mat 2 a'r’ Eyank = 5 for a single boson. In the ground state, n = 1. For N such bosons, E,=NE,, Detailed Explanations of Answers 88. (C) In the WKB approximation, the SchrOdinger equation a a + res -Vviyinn=0 has a solution yneetor where kin) =f dra/2m(E -Vin). This may be used to find the probability for an 0. particle to tunnel through the ‘Coulomb barrier. 89. (B) The law of Dulong and Petit statues that C, = 3 nR where n is the number of moles and R = NV,k is the ideal gas constant. In the Debye theory ome finds that C, increases from zero at T = 0 K to approach 3 nf at high temperature. There- fore the law of Dulong and Petit is valid only for high temperatures where C, ~ 3 aR. 90. (©) One may find from J foyav Tf@av using the Maxwell-Boltzmann distribution lv). [2a = a/200.381 x 107) (300) / (28/6.022 x 10”) = 4,22 x 10° cm/s The escape speed from the surface of the earth is equal to the velocity of a particle whose kinetic energy is equal to its gravitational potential energy at the surface of the earth. Hence: v= V/2GMIR - [69x 10°) (5.98 x 10”) 6.38 x 10° = 1.12 10° cm/s thus / v= .038 which explains the large presence of N, in the atmosphere. 146 Test 1 m =3.87x 10g .87 kg In this convenient gravitational system of units, one could take G = 1. 92. (A) \E = 120MeV THhIAE Using the uncertainty principle = 197.35/120 fm 10°" =1.64 10 om _ 3x10 cm/s 125.510 93. (E) The u quark has charge *, and the d quark has charge ~ '/,, Hence the com- bination uud has charge 2,2 1 3*3° 37 the charge of the proton. 94. (D) velar, yj=0" are given LM otW, ax by the normalization condition ficeta 1 Now we know that 3 1 4 Cams 147 Detailed Explanations of Answers using the standard Gaussian probability density function. Thus 2 Te’ Village a1 Va Vik C = @a/ny" 95. (C) If the two particles are in a spin singlet state, then the is symmetric. Thus 45-1 @)+700-Of 96. (A) 4 BE te(r+ 2%) is one equation of the Lorentz transformation. Hence Ara a/1-B Ata Ary using Ax=0 Now since B << 1, we may use a Taylor expansion, Viveniede and get Area Spar converting 600 km 90050 = (1-206 "i. 1000 nf _ast 1 ) -(: 2(50 ba sarap) (008) =(-4x10"") 5 a small but measurable difference! 97. (E) ‘The adiabatic gas law is pv =pVy. pV =nRT is the ideal gas equation of state for the final situation 148 Test! Vo = MRT, and the initial situation. Taking y= '/, for a monatomic ideal gas with 3 degrees of freedom. Thus 2/3 E-(Q)"-00 98. (C) ‘The particle area density is related to the normal density by P.=P, where x is the physical target thickness. (300 ¥ x= pyfp= ee SLee 265 (11.35 g/cm?) ‘The kinetic energy loss is exponential: Ar =T, = 106728553 = 6.07 MeV 99. (B) B= 2.04 x 10°Pa p =1g/cm’® = 10° kg/m? > = [2- | 2.04% 10° = 1430 m/s 10 2 = 1880 25.45 m 0” 262 100. (C) A ray through the center of the lens is not bent at all. 149 PHYSICS Test 2 GRE Physics THE GRADUATE RECORD EXAMINATION IN PHYSICS TEST 2 —- ANSWER SHEET 1 @®OO® 4. OODOOO 8 DOOO® 2 @©OO®D 35. DOOOO 9. DHOOO 3 O©OO®D % DOHOO®D 0. OOHOO® 4 D@OO®O 37. DOOOO 1. BOHOOO 5. DOOO® 8B. ®OOOO 2. @®©OGO 6 DOOO®O 9. DOOOO B. DOOOO 1. O®DOO®O 0. DOOGQO 4. DOOOO & DOOOO 4. @@OOO 5. DOOOO 9%. DOOOO 2. QDOOO®O 6. ®®@OOO 0. DHOO® B32 ODOOO®D 7. B©OO®D 1. DOHOOO@ 4. DOGO B. Q®OOOO®O 2. ODOO® 45. QDOOOQD D. DOOOO 3. DOOOO 46. DOOOO 0. DOHOOO 4. DOOOO 41. @©OOO 31. POOOO 15. DOOOO 8 DOOGO 2 ®@QOO® 6 DOHOO®D 0 DOOOQO 3. DOOOO 1. ODOOOD 0. ODHOOQO 4 DOOOO 8B. OOOO 5. OOOO 5. DOOO® 0. OOHOO®D 2 ODOOO 6 DOOGO 0. DDOOO 3. DOHOOO 7. DOOOO 2. ©D©OOO® 4 ®@OOG® 8. D®DOO® 2 DOOOO 5. DDOO® 9% DDOOO 2B. DOOO®O 6. DBOOO® HN. ®OOO®O 4. @OQOGDO 5. B©OOOO 1. DOOO®D 3. DOOO® 8. DOOO® 2 DADOO® 6 ODDOOO ». DOOO® 3. DHOO® 21. @@OOO 0. DHOGOO 4. DOOO® BQ®OOO® 6. DOHOO®D 5. ODOOO® 9. DDOO®D 2. Q®OOGOO %. OODOOO 0. DOHOOO 8. DOOO® 7. DOOO®D 31. DOOO®D 4. DHOOO 8. DOOOO 32. DOOOO 65. DOOOO®O % B®OO®O 3B. ®®DOOD 6. HOODOO 10. DDOO®D 61. DOOOO 152 GRE PHYSICS TEST 2 TIME: 170 Minutes 100 Questions DIRECTIONS: Each of the questions or incomplete statements below is followed by five answer choices or completions. Choose the best answer to each question. 1, The displacement vector r of a point mass may be expressed in cylindrical coordinates. In that representation, determine the velocity vector v = dr/dt. (Ay vaste yea? +004 22 ® ¥ (©) v=p6+pesined +27 @) v=? +p66+rpsineg © veart+rdb+rdo 2. Consider the total energy E of a particle. Let T be the kinetic energy and U the potential energy. What is the total time derivative of that energy if the force acting on the particle is conservative? (A) aT /a @) F-v F > © vw : @) asd ©) w/a GRE Physics In the realistic fall of a spherical object in fluid air, calculate the magnitude of the viscous force. Given that the Reynold’s number is R, = 0.5, the kine- matic viscosity is 0.149 cm/s, the radius of the sphere is 005 cm, and p,,= 1.22 x 10° giec. (A) 1.28 md D) 3.22md B) 5.22md ©) 215 md (C) 1.66 ma Determine the corrected value for the time of flight of a projectile near the Earth’s surface (in two dimensions) subject to a resistive force F, =—bv. Let y=bIm. A r= 2y/8 D) 1 2y/e (1+27¥4/38) @®) t=2v,Je—7y,/39) (B) t= 2v,Je (1 +7¥,/8) © t=2v/e (1-1/3) In the photoelectric effect, the threshold wavelength is 2756 A. If light of wavelength 1700 A is incident on a metal substance, determine the kinetic energy of the photoelectrons. A) 450eV (B) 2.25ev v 7 (©) 3.60ev “ey / ©) 2.79eV ee | ‘X-rays of wavelength 3.00 A are incident on a substance. The scattered X- rays observed at 45.0° have a different wavelength due to the Compton effect. Find the scattered wavelength. (A) 3.024 @) 293A ®) 298A ©) 307A © 301A Tost 2 In statistical physics, the counting factor NI is very important. Approxi- mate this for large N. (A) (Wier @) W/in2y ®) © Nw-1) © N ‘The N-step random walk in two dimensions (with step length 1) looks very much like the famous Brownian motion which supported the kinetic the- ory. What is the root mean square distance from the origin? a) VN @) 2N /3 © Vw ©) Vn ® VWwa ° Identify the following elementary particle scattering cross section curve. {sis the the total center of mass scattering energy. o is zero below 2.014 Gev. 18-280 88 4.8 Vs Gev (A) pp inelastic cross section (D) _ pp direction cross section (B) pp total cross section (E) mp total cross section (©) np total cross section GRE Physics 10. ul. Determine the laboratory threshold kinetic energy T, for the reaction P+p—p +p+R. ‘The target is at rest and the projectile is accelerated to have kinetic energy Ty Let m, = m, = 938 GeV and m, = .140 GeV. Ty P P (A) 1.038 GeV @) 140Mev ®) 1876Gev ©) 2.016 Gev (C) 290Mev ‘What is the correct relativistic Lagrangian which yields the Lorentz. force law dpidt = yq(E +.u xB)? Let + = (yu, ye) be the 4-velocity and A* = (A>) be the 4-potential where y=Uv1-B. (A) Jom +f q(E+0xB)-dp @) Lop? J yy(B+0xB)-dp © prt, tally @D) Frwy, +19AMy, © J muy, — J xq(B+v%B)-dp Find the correct 4-dimensional Lorentz transformation matrix for a boost in the y-direction K — K’. Let x* = (r,ct) be the 4-distance. A (fy 0 0 -py ® 0 0 oOo o 10 0 o1 0 0 o 01 0 00 y -py -By 0 0 0 00 fy Y Test 2 15. © ro yo fy © f(y 0 0 py fy 00 ¥ 0 1:0 Oy oo1 0 oo1 0 o oo 1 by 0 0 ¥ ® (1 0 0 0 o y 0 -fy oo 1 0 o -Bby OY In the Rutherford scattering of p + 3; U ,the differential cross section at angle @ is measured to be 10 barns. The kinetic energy of the incident proton is 7.6 MeV. Find 8. (A) 30.0° D detector @) 60.0° © 90.0° f @) 436 beam target © 218° Calculate the vector force due to a potential energy U = kr". (A) - kare ©) +t 8) - kare ® +mrr ©) +hrtr Study the problem of a rocket in a con- stant gravitational field g = 9.8 m/s*. If the initial velocity is 0.40 km/s, the burn time is 100 5, the exhaust velocity is 2.0 km/s, and the mass decreases by a factor of three, find the final velocity. (A) 0.81 km/s B) 0.98 km/s GRE Physics 18. (©) 2.40 km/s ©) 1.62 km/s © 142km/s The spherical region a 0. Let the charge density be o and the x-direction be to the right. (a) @) © @) ® -ox/e, +0x/2e, — ox /2e, +0x/€, -26/¢, ° The electric potential of a grounded conducting sphere or radius a in a uniform electric field is iven as @(r, 0) = —Ey{1 — (a/r)*] cos @. Find the surface charge distribution on the sphere. “ @) © ©) ® cE, sin ® e008 0 2e4E, cos 0 —e 3e,£, cos @ 2e,£, sin @ Test 2 90. What is the distance at which the eye can in principle resolve two truck 91. 93. headlights separated by d = 2.0 m? Take the pupil of the eye to be a circu- lar sper ire of diameter D = 3 mm and the relevant wavelength to be 1 = 6000 A. ® 414m W) 16ekm @B) 8.2km ©) 20km (© 12km Light is incident at near normal incidence angle on a prism of apex angle .. The prism is made of a substance of refractive index 1, and the light comes from and returns to medium n,, Find the angle of deviation using the small angle approximation. A @ntda ®) @/n-)a (a\ © (/n,-Ne ©) O/n+o 1 ny ® @-na Which of the following is NOT a correct statement about the optical proc- es of fluorescence? (A) _Itis the emission of electromagnetic radiation. (B) The process takes place in about 10-* s. (C) Often fluorescence is quenched by collisions. () Ultraviolet photons may thereby be converted to visible photons. (E) A quantum of light is scattered elastically. Find the output of the circuit (shown on the following page) using only NAND gates. 181 GRE Physics 95. A) @) © ANANDB @) AORB AXORB (©) ANORB AANDB Adding negative feedback to an amplifier results in which of the follow- ing? (A) @®) © @) ® decreased gain, increased distortion, increased input impedance, and decreased output impedance, increased gain, increased distortion, increased input impedance, and decreased output impedance. increased gain, decreased distortion, decreased input impedance, and increased output impedance. decreased gain, decreased distortion, increased input impedance, and decreased output impedance. decreased gain, decreased distortion, decreased input impedance, and increased output impedance. What waveform appears on the oscilloscope for the following circuit? The zener diode has a breakdown voltage of 5 volts. Test 2 Scope 96. Which of the following best describes the way in which a lock-in amplifier improves signal to noise ratio. The amp operates with a reference fre- quency @ and a time constant RC. (A) _Itrejects all noise at frequencies <@+1/RC. (B) Itacts like a narrow band amplifier at frequency « with a bandpass of about 1 /RC. (©) Itrejects all noise at frequencies > w+ 1/RC. (D) It amplifies signals at frequency @ and rejects noise at frequencies > 1/RC. (E) It amplifies signals at frequency @ and rejects noise at frequencies < 1/RC. R, 97. R VA O- = Yo Vp o—VWW- + Ry Re 183 GRE Physics 98. ‘What is the output of the OP AMP circuit shown above? A) RV, +¥,)/R, @) R,V,-V,)/R, (B) RV, +V)/R, ©) R,V,/R,-R,V,/R, © R,WV,-V)/R, An electon is projected at = 0 at an angle of 30° with respect to the x-axis with a speed of 4 x 10° m/s. The electron moves in a constant electric field E= 100 NIC y. At what time after t = 0 will the electron recross the x-axis? (A) 10ns @) 18ns @B) 12ns © ns ©) 15ns ‘Which of the following is NOT a true statement about the optical device known as a retarder? (A) A retarder is used to change the incident wave polarization, (B) The produced phase difference is 0.1/2 ua’ (C) A full wave plate has 2m rad retardance. (D) A half-wave plate has mrad retardance. (©) A given wave plate has continuously adjustable retardance, ). What must the speed of the sliding bar be when the current in the resistor is 0.5 amp? Given that B=1T, R=20, and w=.5m. (A) 2m/s x @) 4m/s © ims eo O}/9 R v iw @) 3mis ORTON EO} © Sms c / B Test 2 TEST 2 ANSWER KEY @) ® (A) @®) @) ® (A) (©) (A) © © ©) «@) (A) () (A) «) @) ©) (A) @) (A) @) (2) (B) 26. 27. 28. 29. 30. 31. 32. 33. 34. 35. 36. 37. 38. 39. 40. 41. 42. 43. 44, 45. 46. 47. 48. 49. 50. ® @) @) @) ® @) @) © @) (A) (A) ®) @) © @) © @) () ® @®) «© @) (D) «© © S51. 52. 53. 54, 55. 56. 57. 58. 59. 60. 61. 62. 63. 64. 65. 66. 67. 68. 69. 70. 71. 72. 73. 74, 75. © © @) @) @) ©) © (@) © (A) (A) (A) @) () (A) (A) ® © «®) () @) (A) (B) () (c) © (A) @) (A) (®) @) (A) © (@) @®) (A) © © @) @) @) (E) @) @) ® @) (@) (E) (E) (A) 185 DETAILED EXPLANATIONS OF ANSWERS TEST 2 1 (8) This problem involves vectors: r=xt+y9 +22 = pcos@t +psinO) +22 The tangent vectors are: ar_, az ar ines , 5g 7 Pain ee + poses and a cose sinds The unit vectors are: 2=(ar/d2) / 1ar/acl=2 and similarly in OF + cos OF 02 +siney amd j= Inverting, one finds =cos@p—sindd 0p +cos86 +2 Finally v= Ex ppp Biz pp.pdbee2 2 (E) E=T + Uis the total mechanical energy. Differentiate it to get: ae at dv dtodt dt wy wu aRvs yy UW vee Te ae =F vu a Test2 2 since F =-VU 3. (A) ‘The viscous force is Ym p, vivic,(”) =Fy. ‘The drag coefficient is c,(”) = 24/R, = 2400.5 = 48 here. For a sphere, R, = 2ivir sv v = R,/2r= (5) (149)/ 2 (.005) = 7.45 cm/s = ' mr pyc, (v) J, m (.005)? (1.22 x 10°) (7.45) (48) = 1.28 md by integration: Similarly, vy, = B+ (+ BEF y = -8t/y+ VY, + 8) -e) by integrating the second equation. At y = 0 for positive ¢ = T we find the time of flight: 187 ‘Where we have used yee 5. @) ‘The work function is = hy=he/ 2, = 12,400/2576 = 4.50eV and hence Cu is the substance, The light energy is Es hvahe/d = 12,400 / 1700 = 7.29 eV by conservation of energy t+ = hy ‘Thus the kinetic energy of the photoelectrons is T=W-6 = 2.79eV 6 (E) ‘This is a standard Compton scattering problem. With 4 = 3.00A and y = 45.0° Y 5 given. The Compton shift is WI © n= 2h, sin? Lan’—r ‘The Compton wavelength is h 6.626 x10™ mc 9.109x10™")3x10°) =2.42x107? m= .0242A Hence AA = 2(.0242)(sin* 22.5°)= .071A, Finally /=A+A4=3,07A de 7. (A) Stirling's theorem (Ist order approximation) InNI= InN +...+1n1 x =2 yj it afl inxdx Test 2 =xinx-. =NinN-N soley * aan displacement, we sum up: xy = Ix= Ecos @ Yy = By=Esin @ P= x+y? a, ee IM dle Oy : -ay! [(cos0,+...40080,)*(sin®,+...48in0y)*]d0,...d0y ay! [ccose, +0)? +(sin®, +¥)?] 49, ...d8y, ay! [cos? 0,ssin? 0, 40/74¥? +20 008 0,42V sin s8,]d8,...dy 7) -ayl [f"a+u7 +v2)49,] ®,...dBy, 1 2 a Gs in®,)? "aartl [(cos0, +...+c086,)? +(sin8, +... +8in 8) +1) 40, ...d0y a14..41 =N os Rug #VEP > VN 9. (A) 50 a(mb) 5 W ev Detailed Explanations of Answers ‘The pp inelastic cross section rises at the pion production threshold 5 =2.014 MeV and approaches a constant 30 mb at high energy. 10. (C) P+p—p+p +7’ is the given reaction, The total u-vector momentum squared is (2, +P, =f omic? —m,2c? 3 EE where each p = (p, iE/c). Now E,=T,+m, om fh? +m? +(T, +, my +m, =/4my? + 2myT, ty 2.938) +.140 = +/4(.938)? +2(.938)Ty. Solve for T, = .290 GeV = 290 MeV and Vs= Using m, as the answer, i. (C) ‘The correct relativistic Langrangian is 1 L=Smu"u, +g" Dei ty tA ty To see this, use 4(2)-#- dt\Qu") ax" which is the relativistic Lagrange/Euler equation eg aoa dt ot a! aa! “ co an ng] 38-0 +100) ae 7 E+ UXB) 12, (E) ‘The 4-distance is 190 Test 2 = (ct) and Y= we is the transformation equation. The = 1 and = 3 components are ex! since there is no change in the transverse di- 8 rections. For the parallel direction and time component, we have xteya Bat) xfs y(x* Bx?) x?ey oa ‘The standard forward Lorentz. transformation for y and t. Hence 1 0 0 O° oy 0 -By aa 0 0 1 0 o -by 0 Y is the tranformation matrix and x’ = yx is the matrix equation. 13. (D) p + 7% U is the elastic Rutherford scattering reaction. do. (22¢) sin ® dQ 4T 2 is the differential cross section, 10x 1000-mb ,. Lt” — 1000 fin? & 10mb =| 200.44) | ine 2 1000 [ ce 1s sind = 0.37 = q=43.6° 14. (A) ‘The given central potential is Usk" Hence 29,219 5 1 9 =-vu =-K A 2 46t 2) = (erin) 191 Detailed Explanations of Answers Writing the gradient in spherical coordinates. wt P= knee ar ar since $5 and 35 are zero (curvilinear orthogonal coordinates). Also reaxtyy+m in spherical coordinates. 15. (D) A rocket in a gravitational field may be understood from mg = mv’ + um’ by Newton’s Second Law =0.4-9.8x10" (100)+2.01n 3 =1.62 km/s 16. (A) Use § y-da=—4nGm,, the integral form of Gauss’ law for gravitation. Evaluate the integral in the different regions r g=0 r>b, § y-da=—4nGM =-g4nr? ee eee A plot of g versus r is shown in the following figure. 192 Test 2 9 a b y 7. (©) ‘The gravitational field of a disk may’be found from fundamentals e=-GJ C Ui where dm = oda = 2nasds. ex-o] St =-2noGf (sds/r?)(x/r)% 2s s 2noGx(* wa x = -2r0q{) | 7 N 18. (8) Fermat's principle requires that the time be a minimum. The time is given by the action integral ds A=j S Using n=£ or vat/n we get ants (x,y) dx? +dy* Since ds= Jade +ay* thus Aa? iy ?ntx.y)ae 19. (C) ‘The nuclear reaction is HAM (a,p) RSi or $47 Al — {p+ Si with Q-value. Q = mt my— My My 193 Detailed Explanations of Answers = (4.0026 + 26.9815 — 29.9738 — 1.0078) x 931.502 = 2.3 MeV For the reverse reaction Q = -2.3MeV and Tas (+) ics =2.7MeV is the minimum kinetic energy needed. 20. (A) Sound waves in a gas may be modeled as adiabatic. Hence pV’ = constant where y = 4, for three degrees of freedom. Differentiate to get a - Lop y=pyt+vt 2a wr) w° and hence doubling p changes the sound speed c, by a factor of +/2. 21. (B) ‘The Maxwell-Boltzman distribution is given by LY" ans dn=gN | s (Sena) ? where the degeneracy g = 1 for classical particles. One can always find the factor (1/2 % mkT)* by normalization fran =N. Clearly & = pv) dv 194 Tost 2 defines a probability density function p(v). Thus =f vpody "omneh ery? gmtvm dv 32 =4(—% nat 32-1 dt m -4(5 ) fee J where amTe 2 3 Em ve EE (ge) stm was used. 22, (A) ‘The situation is governed by the binomial distribution N oA P(n)= (” )era -p)" N Nt where ( )- aay is the standard binomial coefficient, The probability parameter is 8 2 fo . 107 em’ 26 pea 10 210 V 1x 10° em? ‘The desired probability is p(0) N! oo 26 1027 y= p10 PO) oi? © ) =e" =4,5x10% where we have used (-xe(eyae" since x<< 1. 23. (D) ‘The standard rule for the propagation of error is, the density of a sphere is 195 Detailed Explanations of Answers a ed payamian Gs) “(&) for the solid Al sphere’s relative uncertainty. 24, (E) For the entire cycle ADCBA AU = 0=4Q-AW Hence AQ = Q.-Q,=AW since Q._is absorbed and Q,, is ejected. The efficiency is then since Q,,>Q.. Finally, one needs to use the fact that Q,,/Q.=Ty/ T-to get Ti e= E 1 where Ty>Te, 2s, (B) A single particle with one degree of freedom may be described in a space (x, p) of two dimensions, The cumulative number of states is TE)= hf LEH pl de dp L L ph oe Seca? 35 Lets? =tV2mE / nh ‘The density of states is B= WimJE/20h 196 Test 2 26. (E) For the two state system with degeneracies (g,, g,) and energies (E,, E,) = (0.) the partition function is Z=Ege Ml =e, +H. “The average energy is thea 3 1% = a2 nz=-t op Zap = g;ee* /(g, +220) and the total energy for NV particles is y=N = g:Nee™ /(g, + 8,6) =a:Nel (ae +2) 27. (0) Lissajous figures are generated from the coupled harmonic equations 40220 fos =0 with solution x = A cos(of+o) y = Bos (ws +B) ‘The figure is thus a parametric plot (x(¢), y(f)). Let 8 = B - ot be the phase differ- ence. Then A=B,0,=20, 5= 7/2 gives the “butterfly.” Also b=4n/2,0,=0, gives an ellipse, and 5=0 or x witho,=o, isaline, 28. (D) ‘The light damping or underdamped motion solution corresponds to coy? > 7/4. ‘The differential equation is 197 Detailed Explanations of Answers a bx? x 4p 40x =0, ymb/m, Om Vk/m A solution would be x = e”e* with auxiliary equation p* + yp + ©, 0. This im- plies Xsio where om fo,?-77/4 me’ Ps ‘One can write XSxtx Ae? (geet) 4 g-lersd) = 2Ae™*”? cos(ar +8) = Be” cos(cor +8). with Bm 2A, 29. (B) R = 109,677.6 cm is given as the Rydberg constant for hydrogen. The Lyman, Balmer, Paschen, Brackett, and Pfund series result from electronic transitions from level m to n= 1, 2, 3, 4, 5 respectively. For the Paschen series n = 3 and so tan(t-1) x nm? 1 1 =A(j m Clearly m —+ 2 gives 1 = 8206A the lower limit. m = 4 gives the upper limit 1. = 18,7604. 30. () It is desired to find the average energy for a photon gas, i, black body radiation. Proceed using Boltzmann factors: E,=jhw, j=0,1,2,...00 = Lee (Ze shold je*/D e*,x=hop d i shod iy//D y/, yme™ =ho(y/0-y)?)/A/0-y)) =ho/(e*-1) Test 2 ‘We have used the infinite geometric series results lty+y'+...=-y) and y+ 2y4 3y?+...) d g_yie—? fAa-yyt= IB I-N =a HF 31. (D) A voltage applied at the metal gate does indeed act as a switch and allows current flow from source to drain. Doping with impurity atoms such as phospho- rus and boron determines the formation of the n-source and p-substrate in silicon material, The extra P electrons or missing B electrons (holes) can conduct a current, Current flow from the n-source to the n-drain is usually blocked by the p-substrate. A positive voltage applied at the metal gate allows current flow. 32. (D) “The electric field and electric potential are related by: E=-vp=-@ or ¢=-[ B-dr ‘Another fundamental formula is that s=b = +2nko (Wb? +2? - Val), 0<0 Graphically, this looks like lel 199 Detailed Explanations of Answers 33. (C) ‘The black body distribution is given by . u@)=22,_1 we? (ei) For high temperatures or low frequencies, one can make a Taylor expansion of e* 2 x alert t. Hence ho’ oer pte eon =ouT YO)= 7S Tahep Ic’ If one tried to now calculate the total energy density u=f U@)do, one gets the ultraviolet catastrophe. 34, (D) The Thomas-Reiche-Kuhn sum rule states that x keyol (E,-E,)= 8" [2m 7 Since b b bh (-b)_ ate os s2( = tseB2, tata tp)-2(2)= 2 # ®? Also (0}-—| 0) = =(0|[x,Hx—xH] |0) m ™ using a wave function y, and expanding the commutator a m= (02x - x°H - Hx?) Further <0|xHx|0>=) E) = L(x) E; j and <0|Hx?|0>=<0|x7H|0> =Y <0b j>£5 =D |rjol Zo Finally we get i 2 Iw EPL E-Fd 35. (A) A general rule for commutators in quantum physics is (A,B) = ATA, B+ (A, BY A. Test2 ‘This problem involves the operator He F_+V(s). Obviously, (V(x), x] = 0. Then consider let a] = gp (ole. 2141.21) 36. (A) A typical experimental set-up is pictured below. 928 inlet a Hg vapor at low pressure variable av voltage V o ‘The heated filament supplies electrons which are accelerated by the posi- tive voltage of the grid. The accelerated electrons gain eney so that (eV, = '/, mv*) and experience inelastic collisions with Hg atoms. The back voltage AV deters some electrons from reaching the plate. The current-voltage curve peaks at the quantum energy levels. 37. (B) For any spherical harmonic, the normalization condition is S¥in Vin * AQE1 ii where Q is the solid angle. For the given problem, we have ema Detailed Explanations of Answers f%y tan. JN? sin? ee“ d(cos®) dd where we have used th fact that dQ sind do db , Sin? @d(cos®) "ap Use the identity sin* = 1 — cos* 0 to get 2nN* f'(1—cos? ®)d(cos®)= anna—22) 1-Svtane 78x 38. (B) ‘The hydrogen radial wave functions come from the associated Laguerre polynomials. The ground state wave function is found from the quantum numbers n=1,1=0,m,=0. 312 Rg(¥)= Ne = (2) eel, 0 ‘The radial probability density is p(r) = RR* r* and this peaks where r=%, Zz One may also show that 2 eo ao proving that the extremum is a maximum, 39. and / = 1 radial wave function is under consideration -oRa, R,(r)=Nre Use the normalization condition J RR* r? dr=1 Test 2 =NPE rhe2'™ dr =NTE rte Alder =N°T(5)/(Z /d9)° =N?41xa,5 /Z> ts wa 2 Ba, aS N= 7 aa? Va 40. (B) ‘The hydrogen-like atom energy is a function of Z, 1, and n E,=—k'Z"pet/2w n? 2 Zw 2+ Foe This is most easily derived from Bohr theory rab we r 7 baprenay/Z Lepvr=hh with the Bobr radius as a, =h* / ke. Then 41. (C) Experimentally, the nuclear binding energy is BE = 931.50 A, 938.28 Z- 939.57 N. neglecting the electron masses. According to the semi-empirical mass formula, we get 2 BE=-C,A+C,Z(Z~—1)A™? +C,A*? + Cy az with parameters C, = 15.6 MeV, C, = 0.7 MeV, C, = 17.2 MeV, C, = 23.3 MeV. Detailed Explanations of Answers 42. (D) In the nuclear Fermi gas model, the density of particles in momentum space is constant: = ‘= gV /(2rh)* = constant # Pp= a ‘aml’? a Vv or 2 Teer 3” Finally, rr=(30°) a a 13 =(3x'014) 197.35* =252MeV/c lfm/c 10"? m/(3 x10"m/s)) A _ 6.63x10™ a 2n =197.35 MeV fm/c th 43, (D) yok L By Kirchhoff's current law L=I+l>1=1,-1, By Kirchhoff’s voltage laws: -A-heo Y ~Hle-Ll', wh /e-L, 41% =0 Differentiate and let K = 1/LC and x = 1/Ly. Then we obtain = -1K+G,-1)« 1 = -1K-G-1)« Add and subtract the equations letting y get +1, and z = I, ~ 1, respectively, to yt mo ky and 2” =— kz 2kz, Hence, the ratio ©, 1, = (EF 2K)/ = 14 Test 2 4. (E) ‘This is the standard twin paradox problem. We are given v = 1.50 x 10* m/s. Hence 1 2 Ec ip 3 ‘The time dilation equation is t = ,y where t,is spaceship time and t is Earth time. 45. (B) In relativity dp Poa just as classically. However, the relativistic momentum is p = my ¥. If v changes only in direction, then £ (pnvy) = my B= mo at dt + If v changes only in magnitude, then a 46. (C) The Biot-Savart Law must be used gato ldxr 4x r Hol ("dl _ Hol B, = Ho. Hol an f Po or The current is 205 Detailed Explanations of Answers = e Lew t Qnr/v ~ 2ar and the velocity comes from (in Bohr theory) my? /r = ke?/r? Vien = 2.19 x 10° m/s Thus f= (1.60 x 10%) (2.19 x 10°) / 2n(.529 x 10") = 00105 A B, = (4m x 10-7) (,00105) / 2.529 x 10) = 125 Tesla 47, (D) Maxwell’s equations are V-D=0 —-V-B=0 a ap vxE=-2 vxn-4 2? where D = eE and B = pH. Now suppose E=Ey, and H=Hz, since E,= since H, = 0. Then we may rewrite the above aH__aD .. aB__ aD ox a a ae Hence, we obtain 26 2B yg 28, a ar oe ar I exece” and =pye”, then we get OE _-OB_d/,aD\)_. oD aD. Qe Drax Or p Dyan oe et ar 48. (D) Because of Lorentz contraction, the circle becomes an ellipse as seen in the lab frame shown on the following page. 206 Test 2 With that proviso, we can find V teh 1 a= 2b i favi-8. yh yh ! bi Let k be the lab frame and a X the rest frame of the ob- x ject. seen in k seen ink” Hla? + yb? = 1 eeyeh 49. (C) ‘The initial picture is T OO and the final situation is 41 By symmetry, the kinetic energy is evenly split after the collision. Now: E=T +m and p= give pt=T" +2mP. Similarly T+m and p'=VE?-nt yield p’?=T? /44mT. By conservation of momentum p = 2p’ cos @ or cosO= VT? +2mP / VT? +4mT. Plug in m = 938 MeV and T= 1876 MeV = 2 m to get @= A cos (7/,) = 35.3 > 20 =70.6°. 207 Detailed Explanations of Answers 50. (C) ‘There is a wide range of temperatures in the physical universe, many le in the laboratory. acces OK __ isthe absolute zero or coldest temperature. 20K _ is the normal boiling point of hydrogen. 77K _ is the normal boiling point of nitrogen. 1234 Kis the normal melting point of silver. 6000 _ K is the sun’s surface temperature, 10° K is the sun’s interior or core temperature, 10" Kis the temperature achieved in a nuclear collision. si. (C) ‘The hydrogen-like atom wave functions have the following functional de- pendence . Vieo He Wow ot (I~ Zr / 2a, ye" -2 ew lay Way o&reos de Won arsinde%e2/ Way-1 @rsinOe*e- 77” ‘The orbital shown, plotted in 3D at y = 0, is that of the 210 state: PO, 2) = Wyt (= 0). 52. (C) In order to get from the usual wave equation to the Schrédinger equation, one uses the De Broglie wavelength concept Ah Ah he 0” TmE-0) Start with the 1-D wave equation ; Fy LEY ysnegane™ at FoF” Separating variables & a Gtr Hono where @? = 4my? = 4n°v? /2? = 4n?2m(E-U)v? / 1? Substituting 208 Test 2 ah, 2m(E-U) Sh IME =D ux =0 Rearranging h? d? tm at tube FO or finally # 6 = £6 in operator form. 53. (D) ‘The damped one-dimensional harmonic oscillator differential equation is x+y axed where E b oe fE at yo! In the light damping situation w,? > */ 4 and the solution is x= Boe”? cos(or+8) where w= V0," 7/4. Ify<< 1, then x= ~Bywe*” sin(we +5) ested? and B= m4 = Late (ma sin? (ar +8) + keos"(cr+8)) =hee™ using @~ @. 54, (D) For E < V,, the two needed wave numbers are k=WVimE a and k’= 4/2m(V,—E)/h. ‘The wave function is ma {ere x<0 @yat® 7 tet x0 ‘The boundary conditions give 0)=14r=r $0) =ik-ikr= kt Solving simultaneously gives t = — 2ik / (k’ — ik) and the transmission coefficient is T = 1 Detailed Explanations of Answers = GK + 4 kD) (+ RP = 4B (B+ / (B+ = 48 / (8+ k?) 33. (B) The given wave function is 42) =-) cos Va“ 2a and we want to find the value of . #0 but <> = 0. 36, (E) ‘The physics laboratory methods used in medicine and law are quite impres- sive. Doppler Vibration Imaging is based on the Doppler effect. Since malignant tumors are more rigid than nearby healthy tissue, they can be detected by this non-invasive technique. Ultrasound uses about 20 KHz sound waves, whereas Doppler vibration imaging uses 200 Hz sound waves. Positron emission tomography is commonly used to study brain tissue, the positon is actually anti- matter, Magnetic resonance imaging or Nuclear Magnetic Resonance makes use of the flipping of nuclear spins. The Massbauer Effect is the recoil-loss resonance emission or absorption of nuclear radiation (y-rays). 37. (C) For an RLC circuit, Kirchoff's law gives 210 Test 2 2 -RI-L'- 5 =0 or siterenining and defining r = Te met T+ +071 =0 for the critical dumping case, aga Pla and the solution is T+ Boe If] =/, and J’ = 0 at r= 0 then the desired solution is T=1+ t nent!?, 58. (D) An object propelied horizontally at distance r from the center of the Earth into a circular orbit feels a force GmM _ my? vat and v=] or r eo GM,” interesting to note that this is the same as the period of an object dropped from distance r =r, and falling through a hole in the Earth (see above figure) to execute simple harmonic motion. 59. (C) By conservation of energy mgh= 3m? or v= 2gh is the pendulum bob velocity Detailed Explanations of Answers Just before it hits the spring. The conservative force is F=-ke— be? so that Us-| Fdc=t t+ + ost, 2 4 Again by conservation of energy a Lyte dnt 7 hE he? +L be Srv? =mgh= Fhe? +t Rearranging 2 ky _ Amgh , OR ee famgh Pk)" =| [amen ek “ -( oe i) 60, (A) This is Johann Bernoulli’s famous brachistochrone problem. By conserva- tion of energy AT=-AV Sow’ mgy=v=2ay- ‘The action integral Asj& v =fyity? de / \2ey must be minimized. The second form of Euler's equation x Alyy x) oe yy is useful since Laity? Vy 4f@ a Hence E(r-v Z)=0 ¥ o =f -y¥ se ad After some work, the cycloid equation is obtained. 212 Test 2 61. (A) For one-dimensional vertical motion the basic classical mechanics equation is mv’ =mg—bv ow vag, Thm Integrate ~e~ [so y a-w by using the u-substitution u = g—y to get din8=P at of v=£(I-e*), Y 8 Y Integrate once again to get 8B ew yr re) Note that dy / dt= v. 62. (A) Gauss’ law for gravitation is V-g=-4nGp in differential form or — it) -_—— § g-da=-4nGm, “Y in integral form. For a single infinite sheet of mass density o =m / A use a Gaussian pillbox as in the figure. Then — | —- | —_ §g-da=—4nGm,, —gA-gA=—-4nGoA —_—| |= 2gh=4nGoA g=2nGo I ¢ u.« W For two sheets, the field will be 4x Go in magnitude in regions I and II and zero in region Il. 63. (D) The first condition of equilibrium, that of translational equilibrium, says IF=0. Hence Detailed Explanations of Answers EF,=R,-T,=R,- F,=R,+T,-W = Tr Ry + Spe men 0 ‘The second equilibrium condition involves rotation specifically Zr = 0. So RO)-wE +T1sin(180-0)=0-9 7 = Ww =¥2500Nn R= 500 Thus —R, = 1000-500=500N In polar coordinate rotation, R = (707N, 45.0°) 64. (D) The moment of inertia T=[r dm 1 =ym for a right circular cylinder. Then by conservation of energy a1? +tie?, v= mgh= mv" +> 10 + vero For normal translational motion v= V3gh Hence 1 2 x. iat Sis Sls 214 Test2 65. (A) Kepler's Second Law is that the area swept out per unit time by a radius ‘vector from the sun to a planet is constant. By the usual triangle area rule Since the angular momentum is L = Ia = mr’, Thus L = constant my,2a = mya My lVq = 2 66. (A) The given mass density is N= M/ (nR/2) The differential force dF between differ- ential mass element dM = Ads = ARdO and m has components 7 dF, = GAM ogg R and dF, = Gm dM sin@/R” oFy oF 6 Gm yp{”, Thus F,= F, oral, cos 6 d@ A -_ x A _ 2GmM dF, =Gm X= 2GmM x RnR’ and Fy= = agi sine a8 = 2GmM. in polar coordinates. 215 Detailed Explanations of Answers 67. (E) By Newton's universal law of gravitation and the definition of weight GMm 2 = me " h Thus ,- GM. is the gravitational field. R 2M 1 yak R (l+xy’ R Use a Taylor expansion to get hy fy = g|1-2443(% Z a R (ey to 2nd order. 68. (C) The hotter fluid loses heat energy to the cooler one: Heat lost = heat gained (T-T,) em, 1) = en Assuming equal specific heats c, = = 2m, we get 2m, (2T, -T) = em,(T -T;) 47, - 27 -T +1, =0 =c and with the masses related as given m, ST, =3T 5 r=5r1, 32 5 ==(30°)C 36 a = 50°C 69. (E) The tide-raising accelera- Earth tion the moon produces is a dif- ferential acceleration between points A and B A8= 84-8 a a ‘Moon =Gm__Gm a@ (atry m=my 216 Test 2 (1-745) -F(-(-29)) using a series expansion, thus Ag = 2Gmria? ‘Tidal acceleration is hence inversely proportional to the distance cubed. 70. (B) ‘Measuring the distance between the two stars in a binary system and know- ing the period of revolution allows one to determine the total mass. Kepler’s general law (3rd law) is T? = Anta? / (m,+m,) For the solar system m, = m, aNd Mt, = Myo Also T = sidereal period. For a bi- nary star system, the period assigned to the system applies to each star. Hence m, = (m,+m,) = 4ra? |T? If m, =m, = m, then m= 2a /T? gives the mass of one star. 1. (0) The 3-D harmonic oscillator can be used to develop a basic nuclear shell model. Use the Schrédinger equation Hy = EW a FEV tuyezy with potential energy U = '/, ia?, Because Pextyte, we get eneray eigenvalues E=(n, +n, +1, + Sho ‘The nucleon has g = 4 since we have p, n, and f and J spin. For e=ino, we get 4 states; for e=Sho, 217 Detailed Explanations of Answers 24 states. Thus 4, 4+ 12 = 16, 4+ 12 +24 = 40 are magic numbers. ‘He, "0, and Ca are very stable. 72. (A) We are given information on the classic Young double slit interference experiment. Plane waves of monochromatic light are incident on the 2 slits. The condition for constructive interference is dsinO=nh. » x (6000 x107"°)(15) (0.15x107) 006 m is the location of the 1st maximum. 73. (B) In the Mayer and Jensen nuclear shell model, the spin-orbit interaction H, splits levels with the same ! but different al-s jes+l For example the P,, and P,, states both have / = 1 and s =", but the different j values ('/, and */,) produce different degeneracies (2 and 4). For the nuclide "10, the proton configuration is 218 Test 2 sy.F Oy) Py? and the neutron configuration is (Usa? CP ya)* Pia? (dy). The ground state nucleus spin is thus /= 5/2 from the unpaired neutron. 74. (E) For a spherical shell, the potential energy is dU =- g 4 rr ) Ganrtarp) =- 16n’Gp'r'dr/3 The stars total gravitational potential energy is then. U =-16n°G 9/3 [ira = - (16n°Gp7/15)R° Since the volume V and Mass M are vader and Ma4nn’p we may rewrite this as being us 3 3 (4n apts u=-2oMR= 3() om’v Finally, the gravitational pressure is then 1(4n " ‘omy 3 4n\” i 413 = pM” 0 V" 3) ° 75. (C) there are six leptons: 6.05 HY and t,o, The electron, mu, and tau increase in mass from 0.511 MeV/c? to 105.6 MeV/c? to 1784 Mev/c?; each has charge — ¢ where ¢ = 1.6 x 10-C is the fundamental electronic charge. The neutrinos v,, V,, and v, are thought to have no mass and and also have zero charge. In nuclear reactions, electron lepton number L,, mu- lepton number Ly and tau-lepton number L, are conserved quantities. The an- tileptons have opposite charge and lepton number: e.g., ¢ has q = —e and L, but et has q= +e andL,=-1. 76. (C) ‘There are six quarks up, down, strange, charm, bottom, and top or u, d, s, c, b, and t. The up, charm, and top quarks have charges ’/, ¢ and masses 350, 1800, 219 Detailed Explanations of Answers and ~ 20,000 MeV/c. (The top quark has yet to be found.) The down, strange, and bottom quarks have charges —'/, e and masses 350, 550, and 4500 MeV/c?. ‘The strange quark has strangeness S = — 1, the charm quark has charm quantum number C = 1, the bottom quark bottomness B = — 1, and presumably the top quark has top quantum number T = 1. Hadrons like the proton and pion are built of quarks and anti-quarks. 71. (A) The theory of radioactive decay proceeds as follows. Let P = probability, then P(1decay)=2dt, A= decay constant The differential number of particles decaying is aN =—NP, =-ANdt he, i", N ‘The mean life is calculated as an expectation value acroa fl te™ de) [ e™ de - [Ade aN =Nye™ 0 where e** plays the role of a probability function. 78. (B) ‘The Roentgen is the standard unit of radiation exposure. 1 R is the amount of radiation which releases in 1 cc of dry air at STP (.001293 g) one esu of charge (4,803 x 10™ esu = 1,602 x 10"? C), The REM or Roentgen-equivalent- man is the amount of radiation which provides the same effect in humans as 1 R of x- or y-rays. The statements in the problem are in millirems or 10-7 REM. Note that a coast-to-coast flight, the natural radioactivity of the body, and living in Denver, Colorado, are natural radiation sources whereas TV (1 mrem per year), a dental x-ray, and a G.I. tract exam (7500 mrem) are obviously human-made radiation sources. 79. (A) The Méssbauer effect, discovered by R. Méssbauer involves the resonance emission or absorption of nuclear radiation without recoil. For the usual emission O Oo ww O oO Test 2 case A* — A + the initial and final states are shown in the preceding figure. By conservation of momentum the recoil and photon momentum are equal Pas Py Hence the recoil energy is E, = Pe/2m, = (129 x 10°) /2 (191) (931.5 x 105) = .0468 eV 80. (E) ‘One must use a fat lens formula to solve this kind of problem: nin aon "452 —15 cm. Thus s’ = 85 sos oR ‘We are given n’ = 1.7 and n= 1.2, AlsoR = 5 cm and cm, 0.5 1.217 95.5 9 285m 81. (B) Consider transitions between two molecular states i and j. We are usually given that N’=N,B, Uo) and N’p N/A, + B, Uo). Hence Ma (4,+8,01@))/ 8,010) E,-E,=ho Detailed Explanations of Answers But from the Boltzmann factor Nae Ny, Thus B,e*PU(@) = A, +B,U(@) U(@) =A, | (Bye"*? - B;) where Ay 7p and B, are the Einstein transition coefficients, 82. (A) The B fields of a single circular current loop may be calculated from the Biot-Savart law apate ldtxs iss 4 tobe ™ = pe 207? +27) along the z-axis, For Helmholtz coils, we have two such loops, each with N tums, and the field at z = R/2 is 2 B=2n bol {a+®) 4 $y? = NpoIR? | (3) R 4 =8Npol / (5°7R) 83. (C) Consider Faraday’s law — Maxwell's 2nd Equation aB vxE=-B A vector identity is Vx(VxE)=V(V-E)-VE Now V-E=0in vacuum =-ZvxBe-VeE. Now use Maxwell's 4th equation 135 VxBe 5 5r Test 2 to get “ vee LE Far ‘A plane wave solution is Ex yE., cos (cot kx +) +2 008 (cor ke +B) E,, I o=Pande,=£,=£, then ER+E=E and we get circular polarization. 84, (0) ‘The potential is that of a real charge q at z’ = A and an image charge q at yerh 1 1 ° elector ir Note the boundary condition $(z = 0) = 0 is satisfied. The electric field is then 8 tg] og te E,=-3,7 4 Gaaar py @ tatty, =0)2-—! 2 py i EG = Oye ze aghl(s? +H)" using, k= and where s?= x? + y* infinite V=0 conducting plane 85. (B) ‘The Fraunhofer diffraction intensity pattern is run (9)(9} [Ltdy=2f dy yey N 223 Detailed Explanations of Answers ue uN £ Idy =215 (sin?()/2?) ae = (2pM / nd) (m/2) =hN/d To do this problem, one needs to know “sing 4. p"sin@x 4 [3R dew [Bae E The physics student should recognize sin (x) / mx as one representation of the Dirac delta function 8(x). 86. (A) ‘The electric potential is Vek pd’r/r = fr? [rpd’r = /rp by definition of the dipole moment. Continuing Vak-p/r? =k pcos0/r?- for = The electric field is E=-W 2p M6 or Using spherical coordinates for V = kp((2.0080/ r°)F +(sin®/r?)8) =(k/ 1?) -p)F-p) and this is the usual dipole field. 87. (C) For a particle on the surface of a sphere in spherical coordinates x =r sin@cosg y=rsin@sing 2 = rcos® and x+y? 4 7= eRe, ‘The potential energy is 24 Test2 U = mgz=mgRcos® and = = R474 RG sin? O ; ve A since r'= 4, (R)=0. ‘Hence the Lagrangian function is L=T—U='/, m(R? 07 +R? 67 sin? 8) —mg R cos 0. 88. (C) ‘The electric field of an in- finite sheet of surface charge may be found from Gauss’ law. V-B=p/€> $ B-da=qu/€ Recognizing that E - da is only non-zero on the left and right side faces of the Gaussian pill box, we get EA+EA=0A/€y oe pn {Ol 2eox, #90 6 /2egx, x <0 ‘The electric potential must be such that Be-W=-%x Va-JE-dr=-[E dx =-ox /2€5 for x> 0 as desired. 89. (0) ‘The electric potential of a grounded conducting sphere in a uniform electric field is easily found and given as (1,0) =~ Ey(1- (4) cose The electric field has two components eya-bDa-2(1-($) )ane (Asr—a, clearly E, +0.) and g-- Bees +2,]cos0 the surface charge is thus Detaled Explanations of Answers 0, =£,E( = a)=3€,£, cos 0, 90. (B) ‘The limiting angle of resolution of a circular aperture is 2d ee where 2 is the light wavelength and D is the lens diameter. We are given 4 = 6000 A and D = 3 mm. Hence 22, $000 x 1 3x 10 By the definition of radian angle @=a/l then = J = d/@, 2.0 /2.44 x 107 = 82x10m = 82km 2.44 x 10™* rad Because of smog and haziness, one can rarely see this distance. 91. (B) The deviation angle of a prism at nearly normal incidence is found from Snell’s law and geometry n, sin®,, = n,sin®,,, or 18, =7,8,, and 7,0,,=7,8,, in the small angle approximation, ‘The total deviation angle has 2 parts: 226 Test2 8= (On - 8,1) +82 - 82) = 0,-0,, 4% 02-02 i n; -(&-rJles +82) ‘Observe that o= 0,487 H( for ®,, small at near normal incidence. Thus 92. (E) Fluorescence is an emission of electromagnetic radiation which takes place rapidly in t ~ 10-8. This is the natural lifetime of the molecular or atomic excited state, At STP, a molecule undergoes 100 or more collisions in this time and hence the fluorescence is usually quenched. Ultraviolet photons are commonly used to generate lower energy quanta via the Stokes transition. When there is an appreciable delay t >> 10+ s, the emission is called phosphorescence. A quantum of light is scattered in the Compton effect off of electrons, but no such scattering ‘occurs in fluorescence. 93. (B) ‘The truth tables for circuits are very much related to standard mathematical logic truth tables, Let T = 1 and F = 0. Then, there are four cases AorB A andB output A B AvVB AAB ANANDB AXORB 1. 1 1 0 0 1 0 1 0 1 1 o1 1 0 1 1 00 0 0 1 0 where we have shown A v B and A B as examples. 94. (D) Adding negative feedback to an amplifier results in decreased gain, de- creased distortion, increased input impedance, and decreased output impedance. 227 Detailed Explanations of Answers 95. (E) The oscilloscope measures the voltage across the resistor R and hence the current through R. For positive input voltage, the diode conducts. For negative input voltage, the diode only conducts when the voltage across the diode exceeds 5 volts. Thus the input AC voltage is chopped as shown, 96. (B) A lock-in amplifier is a phase sensitive detector. It detects signals at the same frequency and phase as the reference signal. This produces a DC signal which then goes through a low pass filter with time constant RC. The effect is to reject noise at frequencies more than 1/RC away from «. That is to say, the lock- in amplifier acts like a narrow band amplifier at frequency @ with bandpass of about 1/RC. 97. (D) This is a simple difference amplifier. 98. (E) This problem is very similar to motion in a constant gravitational field. By the definition of electric field: Fe => a 0 0 = (1.6 x 10°") (100) / (9.1 x 10°), 1.76 x 10" m/s? Then from kinematics v, =v, tat 0 =», sin@+at 0 1 = -v,sin@/a T = -2y,sin@ /a 2 (4x 10°) (sin 30°) / 1.76 x 10") 2.3 x 104s = 23ns since the time of flight is twice the time to reach the apex. 99. (E) A retarder is an optical element used to change the polarization of an incident wave. The phase difference produced AQ is called the retardance, The optical device is a full wave plate when Ad = 2m rad. The optical device is a half- wave plate when the retardance is r rad, Similarly, the quarter-wave plate causes Test 2 a phase shift of 1/2 rad. A compensator impresses a controllable retardance on a wave; this retardance can be varied continuously. 100. (A) By Faraday’s law or Maxwell's 2nd equation: ‘Thus the induced voltage is dad ve] Beda a! =4axw a = Bwy By Kirchhoff’s law V=Rl. Thus Bwy = RI v = RI/Bw = (2:9) (0.5 A)/(1T) (0.5 m) = 2m/s. The Graduate Record Examination in PHYSICS Test 3 GRE Physics THE GRADUATE RECORD EXAMINATION IN PHYSICS TEST 3— ANSWER SHEET 1 @®OO® 4. O@©OODO 8. DDHOOO 2. ®®OO® 8. @OOOO © ®@DOGOO 3 O©OOO 6 @DDHOO® 0. DOOO® 4 ®©OO®D 3. DOOGO® 1. @OOO® 5. ©OHOO® 8B. ®@OQOO®D 2. @OOOO 6 ®OOO® 9%. DOOOO B. DDOOD® 1. ©@©OO®D 0. DOOOO 4. BOOOO 8 ODOOO® 4. DOOOO 5. DOOOO 9». DOHOGO 2 @OHOG®O 6. HOODOO 0. O©DOHOO® 8B ®DOOG® 7. B©OOGO ll. DO©OOO 4. @OOO® B. ®BOOOO 2 OD©OOO®O 45. @OOOO® Dd. DOOOO B. ©@Q@OOO 6 HDOOO® 0. DOOOO 4. @©OOO 41. @®DOO® 31. DOOOO 15. D©OOO 8 DOOO® 2 ®DHOO® 6. ©DDHOOO 9. DOOO® 3. DOOO® 1. ©@©©O®D 0. DOOOO 4. DOOGO B DDOO® 51. DOOOO 5. D®HOO® D0. DOOOD® 2. @QDOO® 6 DOOOD® 0. DOOO@ 3. DOOOO 7%. @OHOOO 1. DOOOO 4 @®@®OO®O 8 DDOOO® 2. BOOOO 5. DOOO® 9 DOHOO® 3B. ODQDOOD® 46. @DOO® HN. ODGHOO® 4 O@OOO® 5. @©OOO 1. DOODO 35. DOHOO® 8 QDOOOO 2 DDOOD® 6. ADOOO®O 92. DOOO® 3. DDOOD® 27. DOOOO 2 DOOOO 4. @DOO® 8 D@OOGOO 61. ODOOO® S. OD OO® 9. @DOO@ 2 ®OOOO 6. ODODOO® 0. ©@DO@DO 83. DOOOO® 7. DOOOO 31. @®OO® 4. DOOD 8 ®@OQOOO 2. @DOO® 5. DOOOO 9%. ODDOO® 3. @D®OO® 6 ®DOOOO 10. ©@®O©O® 5. DOOOO 232 GRE PHYSICS TEST 3 TIME: 170 Minutes 100 Questions DIRECTIONS: Each of the questions or incomplete statements below Is followed by five answer cholces or completions. Choose the best answer to each question. Let the point of application of a force F = (5, 3, -2) N be at position r = (-2, 1,-3)m, Calculate the torque t due to this force. (A) Tx-19y-11zN-m @) -1ix-1ly-izN-m (B) lix+ily+1zN-m (©) -7x+19y + 1izN-m (©) -10x+3y+62N—m Julie, a physics student, stands on top of a 50 m cliff. She releases one stone with a downward speed of 1.0 m/s. With what speed must she project a second stone 0.5 $ later at a 35° angle if both stones are to hit the bottom. at the same time? (A) 3.28 m/s 5s later @) 2.00 ms oe ve (©) 4.00 ms ° @) 1.51 m/s nase © 8.60 m/s A cylindrical bucket of cross sectional area A, initial mass m,, and initial speed v, moves through space, picks up space debris of density p, and slows down. Use Newtonian mechanics to find the velocity of the bucket as a 233 GRE Physics function of time ¢. (A) vey, B) v=rgerMol (©) vev9/fi+2pAvet iy D) verge terre ) vv / {1+ pAvot [img 4. Consider two reference frames K and K’, where K’ moves relative to K with speed v = c(1 - 8) along the xx’ axes. Consider a particle moving with speed u’, = c(1 — 5) in X”. Find this particle's speed in the lab frame K to order 8%, Note 8 << 1. (A) 2c(1-8) K K y y (B) (1-8/2) © cd-8 . @) e(1-8/2) x x ® a 5. Consider two equal masses m, = m, =m that are attracted gravitationally. ‘Suppose that the masses are initially a distance r, apart and that one mass is given a velocity v, perpendicular to r,. For what values of v, will the masses be bound in elliptical motion? (A) %> 24/Gmir, @) %<2a/Gmir, (8) %>4/Gmir, ©) %<4/Gmir, ©) %=fGnir, 6. According to special relativity, a clock at the North pole must measure a longer time interval than a clock at the equator of the Earth. Suppose that the polar clock reads T = 100 years. By how many seconds does the clock at the equator differ? (r,= 6.4 x 10°m) Test 3 (A) 1.90x 107s (@) 7.58x 10°s ®) 3.79x10?s (BE) 9.48 x 107s ( 5.70x 107s Two balls are thrown vertically upward at the same time. Suppose that the balls have initial velocities v, = 20 m/s and v, = 24 m/s, re- spectively. Find the distance between the two balls when ball one is at its maximum height. (A) 20.40 m () 814m B) 28.56m ©) 14.28m © 16.28m Consider two masses m, and m, moving on a frictionless surface as shown. Find the distance x of maximum compression of the spring. M4 Ye ——— LOTT TTT my m2 Vy > V2 A) af milky, D) fC, + myTE(, +) ®) Smsky, ® Vmm,i(m, + mkv, - ¥,) © Sim + mph, -vy) Imagine a type of cylindrical brake as pictured. Initially a solid cylinder of mass M and radius R is rotating with angu- lar velocity «,; then the brake, a cylindrical shell of mass m, collapses onto the cylinder. What is the final angular ve- locity of the system? (A) Ma/(M + m) (B) ma/(M +m) GRE Physics 10. 12, (C) MoM + 2m) ©) mos © Mom ‘A mass m is subject to the gravitational force and attached by a string to a Second mass m”. Supposing that the pulicy has a finite moment of inertia /, find the acceleration of mass m. (A) mgh(IIR? + m+ m’) m R ®) 8 (C) mgim’ @) mglm+m) m ©) (mm )gi(UiR*) A skier leaves a ski jump ramp at an angle of 14° with an initial speed of 11 ms. Later he lands down the slope a distance ! from where he started the jump. If the slope is inclined at 45°, then find J. (A) 20.5m ®) 411m © 820m ©) 615m © 102m Consider the problem of a spherical pendulum of length J and mass m subject to grav- ity as shown, Derive the La- grangian for this problem in spherical coordinates. The pendulum is free to move both in the x and y directions and would hang (at rest) parallel to the z-axis. Test 3 14, (A) 1/2 mP(07 + 7 sin’®) - mg I (1 +.cos @) ®) 1/2mP07~mgi (1 +c0s@) C) 12m? (67 +97 sin’®) - mg I (1 - cos 6) @) 12 mF 6? sin @- mg I(1 +08 8) ©) 12mP O7+4%)—mg tos 0 Picture a particle of mass m that is constrained to move on the surface of a cylinder x? + y* = R? and subject to a force F = — kr. Find the Hamiltonian in the appropriate cylindrical coordinate system, (A) He l2m:7+12kz? B) H=12mR07+27)+ 12k +2) (©) H= 12 mR (07+ $7 sin? 0) + 1/2? ©) H=12mR*97sin°+ 172kR* © A= 12mR07+12kR Suppose that the disk of radius R shown is in equilibrium, Note that the incline has a coefficient of static friction 1, = y. Find the tension in the cord. (A) T=mgsin® (B) T=mgcos® fn (C) T= mg sin @/(1 + cos 0) CG). @) T=mg cos @/(1+sin @) 7°) ©) T= mg (1 +cos y/sin @ Imagine that an object of mass m (m = 2kg) has position vector r = (3¢ + St*)x. Calculate the work done on the particle over the time interval from 0 tols. (A) 783 @®) 1573 GRE Physics 18, © 2355 @) 3153 ©) 393) Derive Kepler's 3rd law from the assumption that the Earth moves in a circular orbit about the sun of radius r = 1.90 x 10” cm. Use us intorma- tion to calculate the mass of the sun. (A) 25x10" g @) 10x10", @®) 50x10", © 20x10", © 35x10", A block of mass m moving at speed v collides with a spring of restoring force F =—kx— kon frictionless surface. Find the maximum compression of the spring. m WS Za A) Vik be (1+ mEe - 1) © Sidhe, (f1+ mee -1) ©) klk + 2mv?k JK, -1) © Site, (f1+ ame 1) 2 A dumbbell type molecule is modelled as two spheres of ra- dius r and mass m separated by distance 21 as pictured. Sup- pose that the angular frequency = @y + wz. Find the angu- Jar momentum of the molecule, 2 Test 3 20. 2. (A) L,=0,L,=4/5 mea, L,= 4/5 mea, (B) L,=0,L,=2 me, L, = 4/5 mro, ©) L.=2/5 mr'o,,L, = 4/5 mrw,,L,=0 D) L,=0,L,= (4/5 mr? + ImPyon,, L, = 4/5 mr, B®) L,=0,L,= 2/5 mr + Ib yo, L, = 2/5 mre, The executive toy in its simplest form is made of two identical masses hanging from a pivoting rod as shown. If each mass is m and the lengths are L for each arm and / for the pivot, derive the condition under which the toy is stable. (A) Leos @>1 (B) e<4s° © Leoso>! @) e<30° ® L>1 Consider the flow of an incompressible fluid through a horizontal pipe as shown. Determine the pressure difference p, —p, in terms of the cross sec- tional areas and the flow velocity v, at the left. (A) 1/2p v2 QA.) ®) W2p¥ Ae (© 1/2p vt a +AV/A2) © 1/2p vt AA, A © 1/2p v2 @-A3/A)) Consider the problem of four infinite charged planes situated as shown. Find the electric field in the region !x | < a/2. (See figure) GRE Physics (A) o/2e,x yh (B) -0/2e,x ; (©) 26/e,x @) -26/e,x ® 0 e ° -o -6 22. A wedge capacitor has potential © (@ = 0) = 0 and ® (= a) = V,. Assume Sat ‘two plates are infinite. Find the electric field between the capacitor (A) E=-Vv, 9/09} ®) E=-ve/o$ © E=-V,/a§ @) E=-V,/op$ ® E=-v,/p8 x 23. A neutral hydrogen atom may be thought of as a proton orbited by an electron. Supposing that the electron’s charge density is p(r) = ¢ (8 (r) =(@? / x) e-™), calculate the radial electric field E,. (A) E=e/4ne,7 ®) E,=¢/4n,re* (©) E,=6/4ne, Qo/r + ry @) E,=¢/4ne, (20? +2o/r) ©) E,=6 / Ame, (20? +20/r + rt) em 24. A charged pith ball of mass 2 g is suspended on a massless string in an electric field E = (3x + 4y) x 10° N/C. If the ball is in equilibrium at @ = 57°, then find the tension in the string. Test 3 27. (A) .0500N @) .0250N © 0125N @) .0063N © .0032N ‘A rod 20 cm long has a total charge q = ~75 j1C. Find the electric field along the axis of the rod 10 cm from one end. SSO = 20m 10om 0 (A) -5.50x10°N/Cx @) 225x10°N/Cx (B) -2.25x 10° N/C x ©) 5.50x10° N/Cx ©) ONC A capacitor is constructed from two rectangular metal plates of area A separated by a distance d. Suppose that one half of the space between the plates is filled by a dielectric x, and the other half by a dielectric x, Find the Capacitance in terms of the free space capacitance C,. (A) 2x, «, C/O, +) B) (K+K)C, © c @) (,+K)C,/2 ® «,«C,/(K,+K) Find the electric field of a uniformly charged disk of radius a situated in the ‘yz plane at point P along the x-axis. Let the surface charge density of the disk be o. (See figure) (A) 0/2€5 @@/ Vx? +a? -1)x B) 6/2e,x GRE Physics 29. 30. ©) o/regx/\x +a? x @) 0a /4eyx 28 | ©) o/2e, (Vx? +a" -x)x P (x,0,0) Pe 0.0) A resistor is made from a hollow cylinder of length /, inner radius a, and outer radius b. The region a O and an infinite barrier for x < 0. In the WKB approxi- mation, what equation must be solved to find the energy eigenvalues? Let x = a specify the classical turning point. A) § pdx/h=n/2, 3n/2, 52/2,... ®) ff pax sh=3n/4, 7/4, 11/4, © ff pacsh=n, 2n, 3m... @) ff pdx sh=n/4, 30/4, Sx/4,... ©) ff pdcsh=n/4, x/2, 3n/4, In the Zeeman effect, it is found that a sample of Na placed in a magnetic field B has its spectral D line split into three lines. Find the amount of the shift 50, in cgs units, where cis the angular frequency of the spectral line. (A) bo=4eB/8mc (B) Sm=+ eB /4me 247 GRE Physics ©) bo=4eB/3me ©) 8o=4eB/2me ©) bo=teB/me 48, Each hydrogenic spectral series has an upper and a lower limit. Which of the following spectral series has an upper limit A= 13,700 A? (A) Balmer series (D) Pfund series (B) Lyman series (E) Paschen series (©) Brackett series 49. In the Thomson model of the atom, the electrons are distributed as plums through a positive atomic pudding. What single wavelength of light would a Thomson hydrogen atom emit? (A) 35504 @) 23704 © 184A RatA @) 4740A ©) 7100A 50. Which of the following statements is not true about the Franck-Hertz ex- periment? Specifically, consider the case where the Franck-Hertz tube is filled with Hg vapor at low pressure. (A) An electron loses most of its kinetic energy in an elastic collision with an atom. (B) Electrons raise Hig atoms to the first excited state. (©) The kinetic energy of the electrons may be changed simply by alter- ing the voltage on the grid. @) The collected current peaks at multiples of 4.9 volts. (©) Any monoatomic gas at low pressure may be used. Test 51. 52. 53. 54. In the photoelectric effect, electromagnetic radiation is incident upon the surface of a metal. Which of the following is not a true statement about the photoelectric effect? (A) There is no photocurrent unless v > v, (B) _v, is characteristic of the cathode material. (©) Above the threshold frequency, the flux of electrons per second in- creases as the intensity of incident light. (D) The stopping potential V, is proportional to v2. () The stopping potential is independent of the intensity. A beam of tritons is incident on an‘Au foil 1pm thick and scattered through an angle of 37° solely through the Coulomb interaction. Find the differential scattering cross section in b/sr. (A) 5.156 (B) 6.23b © 7340 @) 8.95b ©) 10.306 tam Consider a cubical container of volume V containing a photon gas in equi- librium. Calculate the differential number of allowable normal modes of frequency ©. (A) Vo? deo/ 2x? ©) Vordo/ne B) Wer donc? ©) 8Vardw/rc ©) 4Va'do/rc ‘The black body energy density u(«) has a maximum as a function of w. Find the equation for this maximum, also known as Wien’s law. Let k be the Boltzmann constant. GRE Physics 55. 56. 57. 58. A) Aggy Tm he / 225k @) Ay T=he/3.50k ®) A, .T=he/2.82k ©) AgyT=he /4.82k (©) gy T= he | 3.00k Photoelectrons are found to be ejected from a metal surface when the wavelength of incident light is below 2300 A. If the wavelength of incident photons is 1500 A, then what must be the stopping potential V, to stop the photoelectrons? (A) 827V @) 232V @®) 539V ©) 1.56V © 288V ‘X-rays of wavelength 1.50 A are scattered by a metal through an angle of 90°. What is the kinetic energy of the recoil electrons? (A) 132ev ©) 7360V (B) 264eV ©) 822ev ©) 368ev. A particle is bound in a potential well given by V(x) = 0 for x <0 and V(x) = cx for x > 0, Estimate the ground state energy of this system. A) (he/2V2my"? v ®) W/2m+c YY (© thee? sm? (D) (Re/-V2myP!? + (4? c? (2m)! &) the / 22m)? + (H? c? Jy!" x A ball with a mass of 2 g and a kinetic energy of 10* erg is incident upon a hill 20 cm in height and 2 cm in width. What is the probability that the ball will quantum-mechanically tunnel through the hill and appear on the other side? Test 3 59. 60. 61. 29 (ay 1/10" x0 13x10 ®) 1/10 29 © 1/10 ° = en ©) 1/10° 2 ©) 1/10°°*” > =< 2em Which of the following is a false statement? (A) Avwave function can be found that is a simultaneous eigenfunction of HL, and L,,. (B) [L, 1] =0 where L, is the raising operator, ©) LY,,(@, 6) is an eigenfunction of L? (D) A wave function can't be constructed that is a simultaneous eigenfunction of H, L,, and L,, (E) Exact energy eigenfunctions are obtainable for the case of three iden- tical fermions interacting with an external potential well, but not with each other, ‘What is the transmission probability due to the tunnel effect of a 1 eV electron incident on a barrier .5 nm wide and 5 eV high. (A) 114x104 @) 45x10" @) 36x10 © 32x10 © 86x10" Calculate the Fermi energy for relativistic electrons in a neutron star. The density of the electrons is p = 0.01 / fm’, (A) 16.9GeV @) 65.77 Mev (®) 845GeV (E) 131.54 Mev (©) 38.4MeV 251 GRE Physics 63. ‘Two particles of mass m move in a 3-dimensional cubical box of side a. If the particles also repel each other via a weak short range force V(r, —r,) = V, (r,—1,), then calculate the ground state energy using perturbation theory. (A) 3h? /ma*+(3/2a)V, (D)b? n?/ ma? + (3/2.a)V, (B) 3h? / mat © #r/ ma © @G/2ayv, An atom has three valence electrons in a p shell. Determine the total number of states in this configuration. That is, how many distinct three electron states can be constructed from the orbits in a p shell? (A) 8 @) 4 @) 2 © 3 ©) 12 Let the potential energy of the NaCI molecule be described by Vira e/reke ? where r is the internuclear separation. If the equilibrium separation is r+ = 2.50 A and the dissociation energy is V(r*) = 3.60 eV, then find the con- stante r, and k. (A) 1,=2.50A,k=3.60eV @) 1,=1.25 Ak=7.20ev O——O ©) 120.944, k= 30.94 eV Na Pa @) 1,=1.88A,k=15.47eV ® 7, =2.50A,k =5.76eV Scientists use the Lennard-Jones 6 - 12 potential V(r) = A/r'? — B/r® to de- scribe the interaction between the atoms in a diatomic molecule. For small departures from the equilibrium separation r,, find the angular frequency of oscillation. Let m be the mass of each atom. Test 3 67. 68. (ay ©=(B2ATe “BEB Tek (B) @=YA/ mri +B/ mg (© = Al mr" = Bim @) w= 312A) mn +848) mig ©) w= i56A/mr* = 428) me An object of mass 6.0 kg oscillates harmonically with negligible damping with a frequency of 1.0 Hz. With a small magnetic damping, the amplitude decreases from 0.25 m to 0.125 m after 10 seconds. Find the angular frequency for the damped system. (A) 6.28 rad/s @) 4.21 rad/s (B) 3.14 rad/s ©) 5.28 rad/s (©) 1.07 rads ‘The bobs of two simple pendula each have mass m and are attached to a string of length / as shown. If the two pendula are coupled by a massless spring of constant &, then find the higher frequency of oscillation of the system, A) onn/g/ltkin ®) a+ fightin Y © waf2g/T+ skim 1 ©) w=Sg/t+2kim ® w=JSgii+skim The ionosphere may be viewed as a dielectric medium of refractive index n = n(@,), where «, is the plasma frequency. Calculate the group velocity of a radio wave of frequency © = /20,. (A) c/V2 ®) c/v3 GRE Physics 70. 1. © ¢/2 @) c/4 ©) v2 In a laboratory experiment, two quamtites x and y are measured, Then the formula faenfzly is used to calculate a third quantity f. If Ax and Ay are the uncertainties in x and y, respectively, then what is the uncertainty in f? A) afar B) ofa A Ge/22F Hayy? ©) Af=f/2y(ax/ x)" +(dy/y* @) ape sY(ax/ay (dy /2y © ap fyax/xF yy A point charge initially at rest at the origin experiences an acceleration a for a very short time and then proceeds to move with speed u = a Ar. What is the magnitude of the Poynting flux of this accelerated charge at distance r and angle 6? (A) qa cos? 0/ 16n*e, #2? z B) asin o/argrc / (C) @atcos? 6 /8nte, rc u ©) ga'sin’ 0 /16n er? © Parsin' Ocost o/4rere The nitrogen molecules in the atmosphere have an ultraviolet transition at 1, = 750 A. The density of N, there is p, = 1.68 x 10” particles / m?, Find the fraction of blue sunlight A = 4500 A scattered out of the atmosphere. (A) 43% B) 86% Test 3 n. 73. 4. 15. (C) 1.29% @) 1.72% ® 2.15% Consider the field of a point charge 7 moving with constant velocity. What must the speed Bi of the charge be for its field in the lab frame at 6” = 90° to be twice the normal non-relativistic value? (A) c/4 @) o/V2 ®) ¢/3 ® V3el2 © cl2 A particle of mass m follows the Maxwell-Boltzmann distribution at tem- perature T. Find the most probable speed for this particle. A) Vurim ©) V3 jm ®) ViFim ©) V3er/nm ©) V8 /xm Consider the validity of the classical theory of the ideal gas. Let p be the number of particles per unit volume, m the particle mass, and T the tem- perature. Derive a condition for the classical Maxwell-Boltzmann distribu- tion to be valid, (A) hp! /f2mkT <<1 @) p<<1A B) hp? /VmkT >>1 © np'3/-V mk << () p@or1A Consider a Helmholtz resonator of neck length L and area A as shown. Find GRE Physics 16. 71. 78. the frequency of oscillation if the volume of the container is V,, Let v, be the speed of sound at the ambient temperature and pressure and ¥ be the adia- batic coefficient. O Sump AleVeL D) WnVfAlVoL B) Jw Al Vol ® van 1V © vitq/VLTA ‘Consider the oscillations of an electron in a plasma of electrons. Let N be the ‘number of electrons per cubic meter. Calculate the oscillation frequency v, in Hz. @) 17.96-VN ©) 4.49-VN @) 1347-VN ©) 3.9%VN © 225-VN A square wave f(t) = a for 0 <1 @ fe Pb (A) 7.87 fm ©) 1064 fm (8) 15.74 fm ® 13.20fm (C) 532 fm ‘The problem of n identical harmonic oscillators with negligible interactions in a microcanonical ensemble of energy E is solved by considering an n-di- mensional sphere, What is the volume of such a sphere? A) 4/3nr @©) e/nir @) xr © esi ©) ra /aIr Consider an ensemble of systems consisting of N harmonic oscillators of total energy E = 1/2 # @N +h @ M subject to the constraint Test 3 93. x E nam. Find the number of microstates Q(E) and from that the entropy in Stirling's approximation. (A) S=(4+N) in +N) @®) S=MinM (©) S=NinN @) S=(M+N)in(M +N)-MinM-NinN (B) S=(+2N)in (4 +2N)-M in M—2N in 2N A solid at absolute temperature 7 contains N negative impurity ions per cm? with lattice spacing a. An equal number of positive ions are free to propa- gate throughout the solid. If a small electric field E is applied along the x direction, then find the electric polarization P,. (A) Nea/2 tanh (¢Ea / 2kT) és (8) Nea r Qe _O @ (©) 2Nea sinh (2eEa/ KT) ] @) Nea/2 (BE) Nea tanh (eEa / kT) An ion of mass m and electric charge ¢ is moving in a dilute gas of mol- ecules experiencing collisions. Suppose that the mean time between colli- sions is + and that a uniform electric field E is applied in the x direction. What is the mean distance that the ion travels between collisions? (A) 1/2¢eEt/m ©) ct /-V2m ) ct /m © &?/V3m © V2eEV/m GRE Physics 95. 96. A hollow cube has conducting walls defined by six planes: x = 0. y= 0,2 =0, x= a,y=a,andz= a. The walls at z =0 and z = a are held at constant po- tential V, whereas the other sides have = 0. (See Diagram) Find the po- tential at the center 6,, (A) V,/6 ®) V,/4 © ¥, Shaded walls, ¢=V, ° x othersg=0 ©) v,/2 ® V,/3 If the Boltzmann equation, which describes the scattering of two particles with initial momenta p and p, to final momenta p’,, and p’, is integrated with a weight of the mass, then what is the result? Let p be the local den- sity, w the local mean velocity, P the pressure tensor, Q the general energy flux, and F the external force. (A) pits-v.P+2r at m p a (B) one finds that momentum is not conserved locally VE © B2+v-Q=—-F/m ©) 2% 69. (ouy= OF +V- (pu) =0 Pp PB (E) ome finds that mass is not con- served locally Consider a sphere of radius R whose surface is held at a potential given by WR, ©, ) =B sin @ sin 9. Find the charge density on the sphere. (A) Be,/R ) Be,/RcosOsing ®) 3Be,/RsinOsing ©) Be,/RsinOsing (C) 28e,/RsinO sing Test 3 ‘A beam of singly ionized boron is accelerated through potential differ- ence of 4 kilovolts and then passed through a mass spectrometer with magnetic field B = 0.5 Tesla. What is the radius R through which the boron is bent? Note A = 10.0129 amu for boron. (A) 5.76cm @) 1152cm @) 2.88cm ©) 1440cm (CC) 864cm ‘A nuclear magnetic resonance experiment is performed with protons. The frequency may be adjusted so as to resonate when the sweeping field crosses its zero value, What resonance frequency is expected for a mag- netic field of 6642 gauss? (A) 7.02 MHz “@) 21.1 MHz (B) 28.1 MHz ©) 3.51 MHz (© 14.0MHz . Radiation safety is an important laboratory issue. Consider a whole body exposure of 10 rem received in a few hours. What is the average effect on the human body? (A) results in quick death (B) causes injury (C) causes detectable blood changes @) results in a 50% death probability in 30 days ) causes no damage Answer Key TEST 3 1 (A) 26. (A) 51. @) 76. (B) 2 ® 27. (A) 32. ®) 7. © 3 © 2. © 53. (D) 2. © 4B) 2» © 54. (B) 79. (A) 5 @) 30. (B) 55. © 80. (B) 6. @) 31. @) 56. (A) 81. (A) 7. @) 32. ® 37. ® 82. (B) 8. ® 33. (D) 58. (A) 83. (©) % © 34. ©) 59. (B) 84. (D) 10. (A) 35. (@) 60. (D) 85. ©) 11. @) 36. (A) 61. (&) 6. © 12. (A) 37. ® 62. (A) 87. (@) 13. (B) 38. («© 63. (B) 88. (C) 4 © 39. (A) 64. ©) 89. (B) 15. @) 40. (D) 65. (A) 90. (A) 16. (&) 41. (A) 66. (A) 1. © 7. @®) 42. @) 67.) 92. (D) 18. (D) 4. (©) 68. (A) 93. (A) 19. (©) 44. (D) 69. © 4. @B) 20. (A) 45. (6) 70. (D) 95. (B) a. © 46. (B) 7. @) 96. (D) 22. () 47. (D) 2 ©® 97. (B) 23. (B) 4. © 73. (A) 98. (A) “a © 49. (© 74. (A) 99. (B) 25. (B) 50. (A) 75. (D) 100. (C) DETAILED EXPLANATIONS OF ANSWERS TEST 3 1 A) r= -2x+y-3am F = Sx+3y-22N ‘One must evaluate the cross product as a determinant, x yulx oy terxF=|-2 1 -3]-2 1 5.3 2|5 3 = x(-24+9)+y(-15—4)+2(-6-5) =7x-19y-222 N-m At=058 - Using basic translational kinematics, we have = ttt Yatevyet gia t+ 4.92 thus hh = 44.97 for the first stone, 50 = 14498 B +.204¢ = 10.204 Or ve ‘Completing the square, we get (t+ 102 = 10.204 + (.102)? So 1 = 3.0945 is the time for the first stone. 265 Detailed Explanations of Answers ‘The key here is to find the time from the first stone and then use that information. Again, we apply the basic 1-D kinematics formula with x, now there is a time delay Ar = 0.5: Yy = Vy(t= 5) +4.9 (F-5)? 50 = v, (2.594) + 4.9 (2.594)? Vy = 17.028 [2.594 Vy = 6.56 m/s ‘The initial velocity of the second stone in the y-direction must therefore be 6.56 m/s. Vy =6.56m/8 6.56 = con® ‘2 = 6:56 __ 6.56 cos@ cos(35°) vy = 8,60 m/s 3. (C) Conservation of momen- tum means that mv = m, Vy. Yet there is a mass increase of thus a Game amt = 2mpAv = 2pAmv = 2pAm,v, Ly acm? )=f) 2pAmavedt, Integrating, we obtain m? = mo? +2pAmgy,t / 1+ 204%, My 4 (8) ‘The velocity of the particle in the rest frame K’ is w= c(1-8), d<<1. Test 3 K « L. oa v x x ‘The Lorentz addition of velocities formula is y= + = 61-8) +e1-8) _ 2c(l-8) areas 140-8" «140-87 2 to order 8. 5. (D) ‘The motion is elliptical when the mechanical energy is negative: B=K+U 242-2 <0 2 % Now the reduced mass is t= myn, / (m, + m,) = m/2. Thus La Gm E=im?-“* <0 4 % = Gn? [1,> I, my 9S 24 Gril 6. (B) ‘The clock at the North pole is at rest in an approximate inertial system. The equatorial clock moves with speed v = aR, Now T= T,yis the time at the pole and T= tony 3554y 85.4005 _ aid dy ar=T-Tyi-B =T -T(-4+8*) .15 x10" s v To =Tt Detailed Explanations of Answers Now the angular frequency of the Earth’s rotation is © = 2ny = 2n/ ty = 2m /86,400 = 7.27 x10 rad /s Hence AT = $(3.15 x10”) (7.2710)? (6.4 x10°)* /(3x10°)*) =3.79x10%s 7. (0) ‘The two balls have velocities ¥,= 20 mys and v, = 24 m/s at time = 0, For the first object: Womvi+ 2a (y—y) 0 =20'-2(9,8)h Since the velocity reduces to zero at the maximum height h = 20/28) = 20.41 m= y, Also v =y,tat 0 =20-9.8 t =2.045 For the second object: Y set vet, ae = 241-498 = 24(2.04)- 4.9(2.04) = 28.55 m=y* Ay =y,-y,=8.14m & (E) a 7 o——> LOOCTTTTTIIT P—_$< = ™ me Vy > Ve We must use both conservation of momentum and energy. By the law of conservation of energy: dey? +4 mv: =} m, +m, Vv? +h? By the law of conservation of momentum: +m,v, = (mm, +m,)v= y= EE, My, + MV, = (Mm, +m, )V=> Vv a Test 3 p (miv, +6 oy? my + my, mem im, +m, J} et = em tm") om +.) — Mav? — m,*y,* = 2myMVY, m, +m, = mm — v2)? Hm, +m) xsafmm, /[(m, +™)k] (4-2) = Ho. —%2) where = mm, | (m, +m) 9% (Ce) ‘The cylindrical shell has momeat of inertial /, = mR? while the solid cylin- der has I, = ', MR? . ® @ We must use the conservation of angular momentum. HL =1,0,= (+1) 0, Since initially only the cylinder is rotating while finally the cylinder and shell rotate with the same angular velocity. ©, =1,0,/(, +1) =", MP a |), Me + mr) =Ma, | (M+ 2m) Note that energy is not conserved in this inelastic interaction. 10. (A) This is a basic dynamics problem. Applying Newton’s 2nd law to the hanging mass m and then the second mass m” we get: Et =(7,-T,)R=la, mg —T,= ma Detailed Explanations of Answers T,=ma Applying Newton’s 2nd law for rota- mm R tion, we obtain: I v=Ra=>a=Ro (,-T,)R=1a/R m(g-a)—m‘a= Ia | R mg -(m+m‘)a=1a/R a (UR? + m+ m’)=mg => a= mg | (UR? + m+ m’) 1. (B) Vor = %— 008 @ = 11 cos 14° = 10.67 m/s =v, sin @ = 11 sin 14° = 2.66 m/s x =f Icos 45° = 10.676 t =1/15.09 Y = t% t+ tha, e —1sin 45° = 2.661 — 4.97 = .1763 1 - 02152 F .02152 F - 0.88341 =0 1 =0,8834 /.02152 = 41.05 m fy 12, (A) Consider the usual spherical coordinates (r. @. 6). The velocity then has two components 270 Test 3 Hence the kinetic energy is K ='/nP 07+ Psi’ 097. Also the potential energy comes solely from the z coordinate 2 =rcox@=Icos@ U_=mgi(1 +c0s 8), Note that @ =180° => U=0 Hence the Lagrangian is L=K-U='/, mP(Q + 67 sin? 0) —mg I(1 +cos 6) 13. (B) ‘The potential is U =4F-dr=firdr="/,o* "h, MIR? + 23) In cylindrical coordinates the velocity is var P+ O'b422 Thus K =';,me =), m (72+ P82 + 27) Since r = R = constant ="), m(R? 07 +27) Hence Ho = K+ Um 'ym(R? 07 +27) +R? + 2) 14, (C) ‘This is a static equilibrium problem. Px 271 Detailed Explanations of Answers ‘The first condition of equilibrium gives: IF, =T,+uN-W,=0 T cos © + p(mg cos @ + T sin @) — mg sin @=0 ad = «EF, =N- T,-W,=0 Thus N =mgcos@+Tsin® which we already used above. The second condition of equilibrium is: Et =RT-pNR=0=>pN=T => mg cos +7 sin 0) =T Tcos0+T=mg sin ® Finally TT =mgsin@/(1 +cos @). 1s. (D) ‘The given position vector is: r=Gtt5e)x or x=3r450 de 2 = Baa 415¢ ae a-Se 01 => F = ma=60t it since m = 2kg is given. The power is P =F-v=1801+900° The work is then w=f Par=[ (1801 +9089) de =9017 422544] | 31S 16. (E) If the motion is circular, the gravitational force causes centripetal accelera- tion so that Fo=my'lr IF =ma Gmmjr = marr using v= or = 4ntnev'r since @ = 2mv = 4m /P sincev = 1/T Gmmje = 4emr/ T* 272 Test 3 P = (4n/Gm)r Hence m, = (4n4G) A/T = (4n6.673 x 10°) (1.50 x 10%) / (x10 =2.0x 10" g 17, (E) ‘The given spring force is nonlinear but conservative. Fo=-kx-kx Using the work-energy theorem: W=hK We-[F de= "katt, kx ="), mv m v 18. (D) ‘The products of inertia or off diagonal elements of the inertia tensor vanish for a symmetric body, €.g., 1, = —Zmxy = 0. Thus L,=1,0,L,=1, @, and L,=1, ©, 1, just comes from the moment of inertia of the two spheres: 1, = 4, m2 43), mr = 4, me 1, comes from the parallel axis theorem: 273 Detailed Explanations of Answers oy 1, =2(_,+md?) where d = I has been used = 2C/,mr?+ mf) =4/,mr + ImP thus -L, =0,L,=(‘/, m+ 2mP) 0, and L,=“/, m0, 19. (©) Consider that the executive toy is tilted at angle & U_ = mg({.cos o- Lcos (c+ 6) + mg (I.c0s aL cos (8—0)) = 2mg (I~ L cos ®) cos a. Equilibrium occurs at HE ~-amg(t-Leostysina=0 As we expect, => o.= 0 s0 that stability at equilibrium requires 2, ey =—2mg(I- Leos) cosa.> 0 a=0 ‘Thus L cos @ > / which means that the masses must hang below the pivot. 274 Test 3 20. (A) Bemnoulli’s equation is 1), pv? + p+ pg h= constant Here we may take h = 0 so that 1 ‘yD Vy? +P, = "ly PVy + Pye Ae Now the equation of continuity implies that AV, =A,Vy Hence P,P, tom Note that 4, >A, => p,0 0 s=a s=0 Test 3 Now the electric field is the negative gradient of the electric potential. B=-W= 2 teat xx 2e9 Ox oa = (x? +a? 2x — Dx a(x 2&9 Vea 28. (C) Now the current density is j=llA=1/p@-a) and by Ohm’s law Bold p pl where s is the conductivity and r is the resistivity. Hence 1v 1 pl x@ a) Finally R=V/T=pl/n@-a. j=0E 29. (E) The bottom two resistors are in parallel 10a He Thus 279 Detailed Explanations of Answers ‘The last two resistors now add up since they are in series: R=R A+R, = 1043.75 = 13.752 ‘The current in the reduced circut ie then 1=V/R=12/13.75 = 873A Applying Kirchhoff's voltage law to the second circuit, we get: 12V = 51, + 101= 51, +8.73 thus 1,=3.27/5=0.654 A. 30. (B) If a magnetic field is applied to a current-carrying conductor perpendicu- larly, an electrical potential difference is generated. This physical phenomena was discovered by E. H. Hall in 1879 and, consequently, is known as the Hall effect. 31. (D) The Biot-Savat law states that: Solr? /2(r? +27)? ‘Where we have used the theorem of Pythagoras Saree 32. (E) Ampere’s law states that VxB=y,j. Since A-(AXB)=0,V-(VxB)=0=5V-j=0. But really vj 280 Test 3 is the continuity equation, Now V-E=6,9, is Gauss’ Law, Hence v uae (B/ey)-0 1 aE. Vda no 33. (0) Ina vacuum V x B = pj. But really j =j,_.. In the presence of matter J Ian tase Hence VxB = 1,6,.+i) V XB ~ We banes = aban Defining the magnetization vector by Vx ~ M)=Hy daw where VXM = jy the problem is solved. 34, (C) ‘The electron drifts in a different direction depending on the direction of the electric field. When the electron drifts upward: mg = cE-6n RNY, ‘When the electron drifts downward: mg = -eE+6nRQy_ Subtracting the two equations we obtain: 0 = 2eE-6 RN, +) 3nRn or es Sot E E ee 6xRnv_ mg 6rRnvy mg ee 281 Detailed Explanations of Answers 35. (B) Derivation of Coulomb's law from Gauss’s law: V-E=p/e, fV-Edv=$E-da by the divergence theorem. fE-da=fpdv je, For a point charge E-4nre = q/e E = q/aney? for a charge q = q, E, = q,/4ne,7 F = 4,E,=49,9,/4n&,r= for the force on another charge q, due to 4,. Using Ampere’s law: VxB= HJ JVxB- da = $B-dl by Stoke's theorem. JB-da = p,Jj-aa For a line current QnrB = p,t B= wl /20r B, = p,1,/2nr For acurrent I= 1, The Lorentz force is F = qv xB=1,1xB, F/L= yi ,/2mr=k ihr G1G/P . = Ho Hence: =_— bly = 5 ane, = 21,€, 36. (A) We must use Faraday's law: VxE = -dB/at a Ve ‘diz -—2 feed a whee =JB-da=Blx V = —BlvandV=RI=1=V/R= BW /R 282 Test 3 Re12Q @ |———va4s iatsm » " VI (BY /R (6-15-4y/12=75 Furthermore, J must be counterclockwise by Lenz's law to counteract the in- crease of magnetic flux due to the motion of the bar. 37. (E) By Kirchoff’s voltage law, EV = 0. So 2 -RI-u'-£=0 c Royton a+ oe *TE2 0 @p = VILE »y=R/L,@= Yo? — 7" /4 c 2" +7 Q'+072=0 ‘Then for the light damping case, the solution is, 2=Q,e7""' cos(ar +8) =O," cosae if 8 = 0; R = 109,L=10°H, C= 10°F = VIC = 1-10" = 31,620 rad/s ot LR oa 5 =_ fs1,6207- 4 0 10 4 c08(31,220 x 2 x 1074 ® = 31,220 rad/s = 0,366 283 Detailed Explanations of Answers 38. (C) ‘Maxwell's equations in vacuum are: V-E=0 V-B=0 VxXE = -aB/a VXxB = p,¢,0E/at Now use a vector identity: Vx(VxB) = V(V-B)-VB Vx (i,€,3E / 24) = —V*Basince V-B=0 2B Hof 5,5, = 7B a VB = Hoe oP is the general wave equation. For B=B,(x, yz) ei 7 # a, west Gatyr tsa )B, =Hete St 39. (A) Start with Gauss’ law in integral form: fE-da=q/e, E2npl = q/e, E = (A/2npe)p ‘Now use Ampere’s law: fBeda=pt 2npB =n! B = ,1/2np)o By the Lorentz transformation, E’,= 1(E,~ By) ao A -ytel (a “(bs ve) Gmp lee! Hence a a Eyno ig Wome YO 40. (D) The altitude is 264 Aga Tost 3 x = 60km Also x = ot 60x10" sothat t= =2x10%, 3x10" Every half-life, one loses one-half of the particles Gy to ty= 3X 15 x 10° =4.5 x 10s three half-lives. Using time 2x10 fet YF Sige eM 1 er B= - =0.98875 1~B=0.00025 tion: 41. (A) ‘The kinetic energy of the electron K = 700 MeV. Hence the total relativistic energy is E = K+m=700511 MeV ‘The y parameter is thus y = E/m=700.511/5.11= 1371 1, =0.9999997 f,_-4 ae Y ar Now we must use the index of refraction: n = clv= 1.00029 ‘The Cerenkov radiation formula is needed B’ = 1/n=0.9997100 cos @ = B’/B = 0.9997100/0.9999997 = 0.997104 @ = 138°, 42. (B) ‘The nuclear reaction is tH+7H + IH+]H or dtd > ptt 285 Detailed Explanations of Answers qe — 4 x T, = 1.808 MeV and T, =3.467 MeV ‘The relativistic 4-momentum must be conserved (P)o=(P),=> (P, i Ele), = (P, i Ele), The 4th component of the 4-momentum is the total energy: E,+m,=E, +E, ‘The momenta components are assumed to be in the xy plane. Pap = Pay Py= Psy m2 > E2-pz=(E,+m,E,-p,'-Py? + E2-p,2+E}—p,2-2E, Ey +m + 2m, (E,-E,) = 2m, +m? + Im, (E,-E,)-2E, E, We are given the mass of the deuteron and the mass of the proton: m, = 1876.140 MeV, m, = 938.791 MeV Hence E, = T, + m,= 187.948 MeV and E, = T, +m,= 942.258 MeV m, = 2 (1876.140)' + (938.791)? + 2(1876,140) (187.948 — 942.258) ~2 (1877.948) (942.258) Finally m, = 2809.462 MeV 43. (C) Classically 20 = 90°. In a relativistic equation, rest energy (mass) must be considered as part of the total energy of the system. The total initial energy is thus: E=T+2m, By symmetry, the (otal final energy of each particle is: EaLt+m. 2 286 Test 3 we m m o— 0 T we Initial Final ‘The relativistic momentum must then be: p=VEt—m* paleo jh 4 = tre +mT Using the 3-momentum triangle, 2p’cos0= p. T? +m? +mT-m? 2 thus, cos?@=—2 a = (1? +2mT)/ (1? +4mT) Using a trigonometric identity. $t+00820)= (T+2m)/(T+4m). c0s(20) = (2T +4m-T~-4m)/(T +4m) =T/(T+4m) 44, (D) ‘Where the Schridinger equation is HY =EY H=T+U =P 98 om (x) ay am ae 267 Detailed Explanations of Answers We now integrate the equation ® £, (ER BY- sen orm fev a to get ahi a am axe The solution to the Schrédinger equation is fAe™, x<0 wo={ x>0 thus woe ho x<0 -2w(0)=0 —RY(x), x>0 Upon substitution, we obtain we m 45. (E) Upon solving the Schrédinger equation, one obtains the following eigenfunctions: [/Aet* x<-l/2 W(x) ={ Boosk,x Ixl<1/2 Ae x>t/2 ‘The continuity of w (+ 1/2) gives the condition that AeW"'? = Boos(kyl /2) ‘The continuity of y’ (+ 1/2) gives the further condition that kde a kB sin(k 1/2) Dividing produces the equation tan (k, 1/2) =k, or tan (k, 1/2) =k, /k, Substituting the appropriate wave number values, one obtains tan(/2m(V = E)l | 2h) = V2mE / k/-fIm(V, —E) | k tan./ml?(V, - E)/ 2h? = (E/(V)-E) tan@ = 6,7 /@*-1 Test 3 with — @=4/ml?(V, - E)/2h” and 0) =4/ml"Vy/24* 46. (B) The given potential energy is: $m ax? x>0 VQ)= x<0 ‘The WEB or Wentzel-Knowes Brillouin approximation may be used. This approximon is valid for a slowly changing potential energy. £ peanE+$, n=1,3,5... ff pax= zs Wid ie Li, wait where ——— = me ma?s?) ‘Note that for the case of the full harmonic oscillator, the WKB solution is exact. . (0) “ te ‘Zeeman effect may be explained semi-classically. Consider an electron orbiting in a circular orbit. The centripetal force is, F, = mo2r with B=0 and F = mart 22 = morr with finite B ‘Combining the two equations, one gets m(o-@,)r = teorB/c or = (@+@,) (@-@,) = teBo /me 2a8w = teBo | me Using the approximation that o= a thus bo = £eB /2mc 48. (E) ‘According to the Bohr Theory 1/A = R(L/r-1/ mi) 289 Detailed Explanations of Answers = 1,0977373 x 10°(1 /n?—1/m)/A Where R is the Rydberg constant R = 1,097 x10’ m"* = 1.097 x 10°A+ For the Lyman series, n = 1. For the series upper limit, m=n + 1 = 2, Hence A= 1216, For the Balmer series, n = 2. Thus 2= 65654, For the Paschen series, n = 3. Therefore d= 18760 A. For the Brackett series, n = 4. Hence = 40520 A. For the Pfurd series, n = 5. Thus = 74600 A. Clearly the Paschen series. discovered in 1908 is the answer. 49. (C) In the Thomson atom, the electrons are dispersed throughout a positive nuclear fluid. Applying Gauss’ law: & Integrating over the inner sphere, we obtain E-4nr* =ptn? 1€ Srv? j4nrte, aE mek The electron vibrates in this positively charged fluid so that = (-p* / 4m?) Test 3 @o = ye? / 4negR?m (9.108 x10") = 1.59210" rad/s = 2me / w= 2m(310*)/ (1.59210) = 11844. 50. (A) In the Franck-Hertz experiment, an electron loses most of its kinetic energy in an inelastic collision with an atom. The electrons raise Hg atoms to the 1st excited state 4.9 eV above the ground state. Any monatomic gas can be used in the Franck-Hertz experiment. Of course, the excited state energy will be different for other gases. A Co) In the photoelectric effect , by conservation of energy fy= "frm +o where @ is the work function of the metal. Furthermore, =, mv. ‘Thus the stopping potential V, is directly proportional to the incident light fre- quency v: =hv-6 or Waty-% e In fact, this is one way of determining Planck's constant. 51. (D) 52. (E) ‘The Rutherford scattering differential cross section is do _(ZZ,e7) 4 2-4) =) Z,=1,Z,=79,K=8.8 MeV 2 do _(0)(79)0.44)) 5 .-4( 37? aQ ( (48.8) ) (¥) =1030 fn? x Jom =10.306 291 Detailed Explanations of Answers 53. (D) For a cavity of volume VeP kl=nn or n, = Ik, thus dn, = IIndk,, Also, the same is true for the y and z directions. But dn = dn,dn,dn, = ny Pk However, only the Ist octant of number space is physically meaningful dn = (In dk Finally, photons have two possible polarizations dn= aes and k= 2 @y 2 =Avan Spa an my he) 2, =Vo? 2. Yodo ° 3 we? 34. (B) The black body energy density is ho” hort u(@) = 251 /(e"™ —1) c ‘The functional dependence is that ox? /(e - ho m@)ex* (e-1), x7 We use the first derivative test to find the maximum, Mase et -9-@ Hehe" =0 3x2(e* -1)—x°e* 3e* -3-xe"=0 e*(3-x)=3 By inspection or recollection, £22.82 = he, / AT @ =2nv = 2ncfh =P Digg, T= hel(2.82 ) 292 Test 3 55. (C) ‘The threshold wavelength allows us to determine the work function ¢ of ‘the metal: @ = myahe/ ry = 12,400 / 2300 = 5,39 eV ‘The incident light has energy hy = he 1/2 = 12,400/ 1500 = 827eV Hence, the kinetic energy of the photoelectrons is K=h-6 = 8.27-5.39 =2.88 eV Finally eV, = K= V,=2.88 Volts 56. (A) In the Compton effect, photons scatter from electrons. yre a y‘te. Initial The given wavelength is: 0 = 150A then AA = 2A, sin’ #/, is the Compton shift where the Compton Wavelength A, = hi] m= (6.626 x 1077) / 9.109 x 10%) (3 x 10") = 2.43 x 10" cm = 0.0243 A thus AA = 2(.0243) sin? 90°/ 2 =.0243 A and NM = ht dd= 15243 A Finally the electron kinetic energy is K = he | h—he/ W = 12,400 (1/1.50 ~ 1/1.5243) = 131.8eV Detailed Explanations of Answers 57. (E) ‘The given potential energy is ~ x<0 veon| a ibe E = p/2m+cx Using the uncertainty principle: dp Ax =h or pak pah/x E = (h/x}/Im+cx = / Ime + cx Now we look for the minimum energy value: dEfdx = /me+c=0 =x, = @/cm)? E, = (8 / 2m) (cm/*) ” + c(t / cm)? = (he/2V2my"” + (h°c2/m)'” 58. (A) ‘The transmission probability is Tee# where k= (2m/h*)(V, - E) Note that V, = mgh = (2) (980) (20) = 39,200 erg. Hence {(2(2)/ 4.054 x10}? (39,200 -10, 000) 3.24 x10" Finally Tae 20200) =e ox10% = 10715910 hope 21008" which is very, very small. 59. (B) (A) The Hamiltonian and the angular momentum operators all commute. 294 Test 3 (B) The raising operator is L, = L,+il, (LL) = +i, LJ= LL) +i, 0) = ~ibL,+ilihL J =—bL, (C) Thisis true since (L*, L,] = 0. Hence PLY, =LLY, =ll+) L,Y, ) L,andL, do not commute, ©) The three body problem has been solved for this case. In fact, the solution is given by the Slater determinant. 60. (D) The transmission probability T=e™ where K= ae a) 5x10" m, m, =9.10x107" kh, leV, h=1.054x10™ J-s Tw eH) (I0) 4 519-5 61. (E) ‘The number counting is still the same relativistically v (Qn) and g =2 for electrons. Integrating, Vv 2 ner, ( anitae ny dn=g @k then kp =Bn*p)!, Now = E* =p? +m”, hence E, tk = (3n7 0.01)" (197.35) =131.54 MeV/c Thus, = 131.547 40.5117 =131.54 MeV is the Fermi energy. Detailed Explanations of Answers 62. (A) ‘The Schrésdinger equation is for the two particle system is ; (-Zorwowa =a BH) For Vv, = 0, W, = (hk,)?/ 2m + (ok)? / 2m = (f/ 2m) (3 + 3) n/a? = 3h x? / ma? ‘Where we have used the fact that k, = nla (nxtny +nz)andn,=n,=n,=1 for the ground state. (Similarly for k,) W, = = a) JV, ® (7, -1,) sin? (nia x,) ... sin? (nla z,)d'r,, Pr, = lay V, J sin* C7, x, sin C/,y,) sin CY, 2,) 8 r, where we have used the fact that Vo = C/,)’ sin (*/,x,) sin ('/,x,) sin (‘/,y,) sin (/,y,) sin ("/,2,) sin (7/,2,) Thus W, = C/,)*V, U sin'(7/, x) dx, = CLYV, Ch, 81, = Cl) Vo Finally £, = W,+W,= 3h? n*/mo?+(/,) V,, 63. (B) ‘The s shell has /= 0 and the p shell has /= 1. Hence m,=—1, 0, 1. The possible states taking each electron separately are then -tlaettted otfltitiuty 8 states 1TLTtT¢d¢et ‘Where the first electron is on the first line, the second electron on the second line, and the third electron on the third line. Now we count states with two electrons in one state and the other electron separate. -P----TINNT ITN Tr o TUT - - - - NTT L 12states INNTINNTL--- - Test3 Hence, there are a total of 20 states, as one expects from the binomial coefficient 6). OL 3) 313 64. (C) “The given potential energy is, V(r) =e? /r+k "9, V(r*) =-3.60eV e =" W ppp kerr wel! 0 Na cl e? =1.44M,V — fn=14.4eV-A Plugging into the two equations, we get 14.4 /(2.5)? -k/ me"? Bechet =-3.60 Adding we get 2.30 r, - 5.76 = — 3.60 and sor, ‘Substituting in the second equation, 5.16 + ket = — 3.6 thus =k = 30.94 eV. 65. (A) ‘The Lennard-Jones potential is a A_8B vn=4-> OnE m We wish to find the minimum. Syn Pao => m=%2A7B Using a Taylor expansion, we have oa VK =V(m += duel _ + +4 a Such =o =r) 428 Vena te et Ct Ben 0 ‘The constant may be eliminated by redefining the energy zero. vena dne =n) 297 Detailed Explanations of Answers ‘The angular frequency and reduced mass are vk/p and p=m/2. 136A 42B ee w=, [3124 848 rm mang 66. (A) We are given that m= 60kg and v,=1Hz. According to Newton's 2nd law F = —ke— bx’ =m” so @=2/m. ‘The differential equation is then = Px’+a2x=0, where = Vk/m and B=b/m The solution is x= ce cos wt thus = 2, SB=in2 b = Bm="/,min2 = 0.83 kg/sec Also 8 = Jo,” ~B7/4 = -V4n? -.139°/4 = 6.283 rad/s 67. (0) Xe From Newton's 2nd law, we obtain two equations Test 3 ma amp Seem) xy = mg" + k(x, —%) Assuming sinusoidal scion, we set x) =x, and x, =x, ‘The coupled equations then become: ‘meox, - mg Aba ktny -x)=0 marx, mg 2 Kx,—%)=0 or (@*= gi) (e,-x,) - 2kim @,- x) =0 Combining the equations there are two solutions, the symmetric mode o,=Jg/t and the antisymmetric mode o_ = g/1+2k/m 68. (A) ‘The refractive index is given by = Jf1- 0," ‘The wave number is on ka 2=™ since n=c/v, » @ Det =o fl-0,7 /0 © nt ‘The phase velocity is easy | ima, TeF = ‘The group velocity requires differentiation d(o “fe levrerre*) 1 @ 12, “fs f-a,'/a? +2d-@,"/0*y nee] =oiira+4 N2)=c/V2 Detailed Explanations of Answers 6. (C) The observable is facdxly and the standard error propagation formula is 70. (D) By similar triangles Fro ut Ey cht a auArt cht” thus Ey =8, SE ¢ since r= ct or t= ric, But Fu 4ne,7 is just the radial Coulomb field. Hence y= Sh, = gen A Amegre? Ane grc™ Also BarxE, /c. Hence ISi=|ExBI/ Ho = q’a* sin? @/16n*egr7c> is the magnitude of the Poynting flux. Test 3 71. (B) ‘The scattering cross section is 6, =o,(. 8x e were 0, = 88 ( is the Thomson cross section 6, = 6.65 x 10-* mi, ‘The reduction in intensity is F = 1-e?r = 1—exp (— (1.68 x 10%) (6.65 x 10°) (750/4500)') = 0086 = 86% 72. (E) In the rest frame of the charge E = q/4ne,r*, In the lab frame where the charge is moving va 1-6" Be =? sin? 6’)? 4negr” 1-p* ye wales apie to =90 We are given that E” = 2E, thus 1 2, —a=4 1-p? of = (=p)? - pefinte 73. (A) ‘The Maxwell Boltzmann probability density for speed is Pv) = neve met a. -mtnat _y2 2m -mtinety dv Arc(2ve Or e )=0 wet iT Thus v= 2k /m is the most probable speed. 301 Detailed Explanations of Answers 74. (A) ‘The typical magnitude of pis p= mv= Vink Using the most probable speed for v. This corresponds to h Boos p Vamkr The classical description is valid for s >> A where s is the typical separation be- tween particles. Now N 1 prpaeey takingN=1 and V=s'. Hence ua 0)" >> yr p ‘mk hp”? < thus Ff =—A*y £0 x— ALpyx’’ Ye ‘Using Newton's Second Law Po PoLVo = fypPeA ny, [A Po Vol “VV. x’ +Ay x=0 Test 3 76. (E) The field in the plasma is e ‘The differential equation is then 2 74 BE 20 x 1,602 x10" /2n 19.109 x10-*" 8.854 x 10 =8.98 VN Hz 7. (©) The given wave form is nef 7 OSES MO=) g -urcoct The square wave is an odd function = A, = 0 Ba= Eff sin(m) dx, x=0r ep, FI, gf Psinimar) dt rr G2J, asinomary de 2a 4a 4a FEU —cosmny= B, 0, 32 Hence _4a fie $0 = SHsin(ar) + FsinBer) +...) 78. (C) ‘The energy of interaction is Ea-p-B = £,=-p,B, E=pB Now we must find the probabilities eM = CeO” B AAT 303 Detailed Explanations of Answers p.=Ce = Ce Mok Py tp_=1 => C=1/ (eM? +e?) = Pyle + (Ho) = (CPA? — FOP), / (EPO? +00) = Mo tanh(HpB / kT’) ‘The magnetiziation is m=N

= Nyy tanh(tyB / kT) 79. (A) We are asked to evaluate B aae Rewrite the integral as =n, nykh Hence Test 3 5 an +8RT ‘Thus each component acts as an ideal gas, but the mixture does not! Ri. (A) ‘The coefficient of cubical expansion 1 (®) __ 1 @PAT), Boy \ar),-"v @Pav), ‘Use Van Der Waals equation of state (P+ ZV -b)= RT thea (22) =-R_ (2h) -_=AT 20 45 (Fe - 1 R/V-b)_ RV 8) Hence B= Teej yp +2a/V RIV -2a(V -BF 82. (B) ‘The mean free path is 1 he Vanna ‘Where the factor -/2 comes from the motion of the molecules. Now La=Vwn for a random walk. So N=LIN = L2ntr? 16a = 320 nal = 32x? (2.69 x 10) (10-*cem)* (10*? = 2.28 x 10" = 10" where we have used the fact that n= 602X109 69x 10" fec 22.4 x 10° cm? 83. (C) We are given that 2 E=2_ =x 2m by the equipartition theorem, Also a wt =-< In a, fuofe since y= Bx‘, y=BM"x and dy=Bl ax = gee Lemay Cane! (a4) Hence =terstar=2er 2 a4 84. (0) ‘The gas is thermally insulated =dQ=0 and hence the expansion is adiabatic. We-[ pava-f WV-' dV since pV’ =k k ‘o vol = y+ WV, 1 = yaa PIs — Po¥o) q W@W, - Pale) 85. (E) k k Quantum mechanically, the energies are Test 3 E,=n, be, combine to give total energy ExE,+E,+E, Heuve the partition function is Za Le = Le Le Ee AP 1 1 er ser Ser -e =e ‘This gives rise to a sum of Planck distributions with average energy as a b c SEP=bOo| rand yt hao 1 * Shoot 7 86. (E) For the hydrogen atom E,=-1360V-45 ‘The K,, x-ray energy has energy A 14 16ev(4-4) 10.2eV ‘The initial energy of an -shell electron is E=- 136 eV 4=-3.4eV 2 After absorbing the photon and escaping, the kinetic energy of the electron is K=102-34=68eV 87. (D) This is a mode counting problem. 2 dn=dn,dn, (4 dk,dk, But since only the first quadrant in number space is physically meaningful dn= (sh) amk at 307 Detailed Explanations of Answers Finally the photon has two possible polarizations so that A a5 k dk 88. (C) Tn the Compton effect AM=N-A=2A, sin? 2 Foro=n N= A+ 2h! m,c High incident photon energy E = hc/\ means small wavelength 2. Nevertheless ved and B= AEs dmc? 89. (B) In the Thomson experiment, the electric force balances the magnetic force cE = evB. ‘Thus =v = FIR. In the absence of B, the deflection is Hence £201.25 x107(50/1.5x107) / (5x10 (1.2104)? m = 2.3110" coul/kg 90. (A) fa+ GPb is the reaction, We are given that K = 15 MeV. . ae Pb a G o z e Test 3 Kes my? = 1h. = Be 2 ror r=82e" / K =82(1.44)/15 = 7.87 fim In a head on collision, the KE is transformed into PE at the distance of closest approach. 91. (C) Vpn] ae f gf dey CR" ‘Where the integrals are all subject to the constraint that afta tty? Rt where R is the radius of the n-dimensional sphere. Now consider a different inte- gral Jey J dryer?) 2(f are fave ™ an? J nc,Rt te Par = x" Make a substitution R? = one FO ee des wi 2e(t\en? «(2 i) x" 2 roi(3) 92. (D) Q(E) can be thought of as the number of ways of putting M indistinguish- able balls (the quanta) among N — 1 partitions along a line. For example, [-] Thus C, ny=2 Hence M+N-1) (M4N)I a-( M )- Sam ‘The entropy is Detalled Explanations of Answers $= kina = kin(M+N)!-InM!-InN! = (M+N)In(+N)-MInM-NInN Using Sterling’s approximation 93. (A) ‘The polarization is P = Ner where r is the vector from negative to positive ion. ‘The positive ions tend to be found at + a/2 with respect to the negative ions (by sym- metry). The positive ion can thus have en- ergy E,=eEal2 ‘Thus the polarization is NeZ eI _Ne e +e = (Ne a/2)tanh(eEa / 2kT) 94. (B) According to Newton's 2nd law F=cE =m’ with Solution 1leE xeif P= is the probability that a particle after surviving without collisions for a time ¢ suffers one collision by time ¢ + dt, The mean distance is exe fhtielena -12 TQ), 2m (A/T) m 9s. (E) ‘We must solve Laplace's equation V7o=0. Use separation of variables O=XYZ 310 Test 3 X=a, sin(o,), =< ‘= b, si at ¥=6,sin®,y), B.=— Zack, etm, Yom 2 At HP To get the potential of the center, we can use symmetry. If all six sides had = Vz, then 9, = V,, The potential is the superposition of contributions from three pairs of opposing sides. With 4 = V, for just one pair, 6. =, V,, 96. (0) ‘The Boltzmann equation is FP Hee Om Or Op =I! fr- F hyavad’p,’ ‘The scattering process is \7 ON, pL masa? pO = 2 fap mp = 22 Sp semalaey Bit em as pp PF m= fa pfp=2 Jans 3p to 5, 0) a Jape ZL m=lap er, p= niol =0 PFS ey, ifm) = Fi fm Likewise, the collision integral contributes zero since the net mass does not change in the collision between two particles. Hence we get the conservation of mass equation. 20, Apu). x or” ant Detailed Explanations of Answers 97. (B) ‘The general potential is W880 At! + Fy (0.0) i is Y(R, 6, ) =B sin 6 sin ¢. Now and ¥,,=sinde*, Hence Prsinesing rR ‘The charge density is =e,(2¥| -2¥] ° oo in ar four 2 10(Zsinosing + 2% snesing) 3862 sinosing 98. (A) ‘The potential energy is transformed into kinetic energy dmv? =a¥y ‘The centripetal force is the magnetic force yp v=qPR Ro Tm Ley? /2¥, =(gBr/m)? /2¥, ™ q/m=2V, | BPR* R=2Vym/B'q '2(4000)(10.0129)(1.66 x10) / .0576m. =5.76cm a2 Test 3 99. (B) ‘The energy splitting is AE =2g4, 81 where g is the Lande g factor and [= e* / 2m, is the nuclear magneton. Also J = 'j,i8 the intrinsic proton spin. Hence AE = hv, =2g 4, BI where v, is the resonant Larmor frequency U, = 2ghyBI/h (5.56(0.5050 x10"*)(0.6642\(1/ 2) (6.626 x10") =2.81x10" Hz =28.1MHz 100. (C) Radiation damage is caused by the ionization and excitation of charged particles in the body. 1 rem causes no damage 10 rem causes detectable blood changes 100 rem causes injury 400 rem results in 50% deaths in 30 days 100,000 rem results in quick death 313 The Graduate Record Examination in PHYSICS Test 4 GRE Physics THE GRADUATE RECORD EXAMINATION IN PHYSICS TEST 4— ANSWER SHEET ®®OO® 0. DOHOODH | DOOO®D | DDOO®D 4. OOOOO | DDOOO 1 @®OO® 4. ©@DOO® 8. DOHOOO 2 @®®OO®D 5. OOOO®O 9. DOOOO 3 ®©OO®D 6. DHOO® 0. ©DDOO® 4 ®@©OOO®O 37. DOOODO 1. DOOOO 5 ®©OOO 38. DHODO® 2. @®OGOO 6 DDOO®D 9. DOOOO B. OBOOG®D 1 8. 9 14, 1S. ®®DOO®D 2. ®@OOOO 6. ODOOOO 0. ODHOO® B. DOOO® Tl. B©OOOD i. O©OOO 4. D©OOG® B ®®OOO 2. 80. 81, 82. 2. ODOO® 4. @OOOD®D . DOOOO 3. DOOOO 4. DOOOO . DOHOOGO 4. @OOO®O 4. DOOOO | ODOOOO 5. DOHOODO 8. DOOODO . DOOOO 6 ODOHOOOD 2 @®OOO 3. DOOOO 1. @OHOO®D 0. DDHOOO 4 @ODOOO B. O©@®DOO® 5. DOHOO® 5. DBOHOO®D 2 ODOOD® 2 @O®OOO 6 DOOOO 0 DDOO®D 3. ODDOO®D 8. HOOOOG 1. ©®HOOO 4. ®@®OO® 8. DOOO® 2 ODOOOO 5S. ODOO® 2% DOOO® B. DDOOD® 6. ODDOO® %%. DDBOO®O 4. DOOO® 57. DOOOO 1. QOOO®D 3. DDOOO 8. DDHOO® 2 OBDOOD® 6. DOHOO® 9. POOOD 3. DDO@O®D 72. @©OO® 0. DOOO® 4. @D®OD® Be @®®OOO® 6. OOHOODO® 8. OOHOOO 2d. DDOOD® 2 ®BOOOO 6. O®DOO® 0. BOOO® 8. DHOO® 7. QOOOO 31. ODHOOO® 4 @®OOO 8B. ODOO® 32. DDOO® 5. DOHOOO %. DOOOO 3. D®OO® 6. DOOGO 10. ©DOOO 4. QHOGO 316 GRE PHYSICS TEST 4 TIME: 170 Minutes 100 Questions DIRECTIONS: Each of the questions or incomplete statements below Is followed by five answer choices or completions. Choose the best answer to each question. 1. Consider the motion of a relativistic particle of mass m, momentum p, and energy E. Find the group velocity. (A) vepet/E ®) »,=E/p ®) »,-E/p ©) v,=p/m 2, Bach of the hydrogen atom quantum mechanical wave functions has a characteristic symmetry. In the below 3D picture where we have plotted WY" (y= 0), what is the quantum state? 317 GRE Physics 318 A) Veo D) Wyo B) Vy ©) Veo ©) Vo Which of the following is NOT a true statement about quantum physics? (A) the wave function is always a real quantity (B) the wave function represents the complete physical state (C) vy. W, and y” are finite, single-valued, and continuous () for every observable, there is a Quantum Mechanical operator ) inone dimension that" yey dx = 1is required ‘The first excited state of the one dimensional harmonic oscillator has eigenfunction o(x) =Nxe 8712, Find N. (A) (o?/n)/* @) (@/nx)'? ®) a/x © Sree © 20/Vr Which of the following would be a laboratory technique to measure tem- perature in the 2000-3000 K range using the Saha equation? (A) the mercury thermometer (measure a liquid height) (B) the standard thermocouple (measure an EMF) (C) the resistance thermometer (measure resistance) (D) the ionization thermometer (measure the degree of ionization) (©) _ the optical pyrometer (match blackbody spectra) Test 4 ‘The dispersion relationship for deep water waves is given by w? = gk + ak where g and a are constants. Find the phase velocity in terms of A. (A) VaN2n+2nar @) V2nah B) SeN2n © VEnnrnan ©) VSgn2n+2ar An important part of experimental physics involves the use of high vacuum. technology. Which of the following is an incorrect statement? (A) Mechanical pumps pump down to about 10° torr. (B) Ton pumps are generally useful as roughing pumps. (C) Molecular diffusion pumps take the system to 10” torr. (D) Ion pumps can evacuate a chamber to 10? torr. (E) Several different types of pumps are usually needed to get a high vacuum, ‘The potential energy of a particle moving in one dimension is given by U @) = 1/2 be*+1/4 bx’, Determine the force. (A) heb? @) -1/6 be 1/20 bx? B) e+e © exe (C) 1/6 ke +1/20 bx A particular curve connected from points P to Q and revolved about the x- ‘axis generates the surface of minimum surface energy. What is the name of the surface of revolution? (A) conic section (B) catenoid (C) portion of a sphere 319 GRE Physics Ml. 12, 13, @) cycloid ©) geodesic A pendulum of length /is attached to the roof of an elevator near the surface of the Earth. ‘The elevator moves upward with acceleration a = 1/2 g. Deter- mine the linear frequency of the pendulum’s vibration. A) 1/2nq/3g/21 B) 1/2n4/2g7/31 ©) 1/2nJ/git @) 1/2nq/g/2t €) 1/2nq/2g/l A room of dimensions h = 2.5 m by w = 5.0 m by !=5.0 m contains 10” air molecules. If all of the molecules in the room statistically conglomerated in a small comer of dimensions h = 2.5 cm by w= 5.0.cm by [= 5.0m, then a person in the room would be unable to breathe, Calculate the logarithm og,, of the probability that this will happen, (A) logp=0 @) logp=-6 (B) logp=-27 ©) logp=-162 (C) logp=-6x 10" ‘An object is projected upward near the surface of the Earth, but also subject to a resistive force -bv. Determine the time taken to reach the maximum height. Let y= b/m (for simplicity), A) w/s @) In(l+v,y /g) (B) 22,/¢ ©) G/yin-v,y/g) ©) G/pind +yy/g) ‘What is the gravitational field of an infinite line mass of linear mass den- sity 42 Test 4 15, 16. ' a (A) -AG/)r @) -AG/r)r B) -2aG/r)r © -2aG/r)r © @ginr A circular annulus a < r < 6 of uniform mass density ¢ is situated with center at the origin in the yz plane. Determine the gravitational potential at distance x. (Ay -2n0GV 8 + x? @) -2n0c_Ve'+e - Vere] (C) +0 [ba] /x (D) no [b*-a"]/x ® +2noclVo'+x - Vere) A stunt plane flies a loop-the-loop circle at uniform speed v. The pilot ex- periences an apparent weight at the bottom that is twice his apparent weight at the top of the circular path. What is the radius of the path? A) Vig B®) wig © wig @) 47/8 ©) sv/g In an Atwood's machine where the one hanging mass is four times the other, find the acceleration. GRE Physics 17. 18, 19. (A) 9/2 (B) 28/3 ©) 39/5 @) 38/4 (©) 4g/5 Which of the following is not a true statement about nucleons? (A) protons and neutrons are fermions (B) even Z even N nuclei have zero total angular momentum (C) the total angular momentum is integral for nuclei with even A (D) protons and neutrons have integer spin (&) _ the total angular momentum is half-integral for nuclei with odd A ‘The Roche limit for the earth-moon system is the distance at which the tidal action of the earth would start to rip the moon apart. Let a = earth-moon distance (assumed variable), r = radius of moon, M = earth’s mass, and m = moon's mass. Find the Roche limit a. (A) @m/M)r (B) GMI m)"r R © GMI mR a D) Bm/m)"R ©) GMI m)"r M Many nuclei either in the ground state or in an excited state assume an ellipsoidal shape. Let Z be the number of protons in the nucleus, N the number of neutrons, e the eccentricity, a the semi-major axis, and b the semi- minor axis of the ellipse. What is the quadrupole moment Q? Test 4 20. 21. (A) 1/5Zea* @®) WsZeb © 3/5Nea ©) %5Zea €) 3/SZeb? ‘The meson theory of nuclear forces suggests a nucleon-nucleon potential of the form Ue) =Vge hI Vye Ie. Determine the form of the repulsive part of the force, the so-called hard core, u A) (Ve (rk +1 [r]-(Vse™" [rly +117] B) Vgkge |r ©) Vakge /r D) (Vee Mr) Uke 1/7) ©) (Wye Pr) kg 1/7 Use the nuclear shell mode! to determine the ground state spin of © Zn. @ 1/2 © 1 @) 3/2 © 5/2 © 0 GRE Physics 23. ‘Which of the following is a true statement about the elementary particles? (A) the photon is a stable particle with non-zero mass and spin one (B) the electron is a stable particle with mass 511 MeV/c? and spin 3/2 (©) the proton is a stable particle with mass 1836 MeV / cand spin 1 /2 (D) the pion hass mass 134.96 MeV / c?, lifetime longer than that of the pion, spin zero, and strangeness one. Hadrons consist of baryons and mesons and their structure is investigated using quantum chromo-dynamics or QCD. Which of the following is NOT a correct quark assignment? (A) p=uud (B) n=udd © mau ) K=%s ® Jak Consider the elastic scattering of two identical atoms. Let the scattering angles in the lab frame be (0,,.4,,) and those in the center of mass (cm) frame be (0,6). Ifthe differential cross section is isotropic is the CM with value 0 4n, then what is the lab cross section? A) of4n B) of2n > - botrame © o/8n — <+— CMirame @) 9,cos 6,,/ 2 © 0,c0s0,,/x Much of radioactive dating is based on the nuclear reaction MOMN e+, Test 4 21. which has a half-life of 5760 years. What is the mean life for this reaction? (A) 11,520 years (B) 5760 years (B) 7985 years B) 3993 years (C) 8310 years ‘The laboratory operation of a LASER is related to the atomic transition problem. Let £, - E, = h @ for two atomic states, u(«) be the radiation den- sity, N, be the number of atoms in state 1, and N, that in state 2. B,, is the Einstein coefficient for absorption, B,, that transition probability for emis- sion, and A,, the spontaneous emission coefficient. Find N, / N, for thermal equilibrium. (A) B,,u@)/A,, ®) A,u(@)/By, © Bua) /B,, ©) Buf tA, + By) ©) B,,u(@)/ TA, + B,,u(@)] Which of the following is not a true statement about the Raman effect? Let to be the incident light energy. (A) a quantum of monochromatic light is scattered inelastically (B) energy hoo’ is exchanged with a molecule (C) the frequency of the scattered radiation is @” = @t oF (D) Raman scattering occurs as a result of the induced dipole moment () a quantum of monochromatic light is scattered elastically Find the magnetic field due to the finite current carrying wire pictured at point P. (See figure on following page.) A) byl /2nr ®) Bl /4nr GRE Physics 29. 30. 31. (©) Bgl / Anz [cos ©— cos @'] ) HY/ 4nr [cos 8 + cos ©) wd / 2nr [cos 0-cos 8’) Figure out the total electric potential energy of a single spherical object of uniform charge density p, total charge Q, and radius R. Let k ='1/4 ne, as usual. (A) 0 B) MIR C) 12kQUR ©) 3/5kQ7R QT ©) We sR A plane wave solution of Maxwell's equations in free space is E=yE, cost -kx+0)+z E, cox (@t—kx +B). Let 5 = B - a be the phase difference. Under what conditions do we get elliptic polarization? (Ay b=4n/2 (D) B=4x/2andE,=E, (B) 5=0 © b=in (©) b=and£,=£, A toroidal substance of inner radius a, outer radius 6, and magnetic perme- ability 41 is wrapped with N tums in which current / flows. Find the mag- netic field B midway from a to b. Test 4 32. 33. 34, CS) ®) © @) ® Npy1/ (a+b) Nyy f/ 7 Nul/n(a+b) Npllna NI/n (a+b) ‘Monochromatic light wave of wavelength 2. are incident on a single slit of width d and observed on a screen a distance ! away and a height y above the slit optical axis. If the distance y is half way to the first minimum, then what is the intensity 1(y) / 1,2 @ @) © @) ® 1/2 lle 2sn Afr light <_—__———_ Ve sit screen A point charge of magnitude q is situated a distance h above an infinite conducting xy plane, as shown. What is the charge density o on top of the plane as a function of the cylindrical radius s? “ @® © ©) ® gh [2 ns? + Wye? -a/@+h) +9q/ (sth) stgh | 4m (s+ f?) 9"? zero since the plane is conducting ‘An infinite conducting xy plane is maintained at zero voltage everywhere except within a circular region s < r where the voltage is V,. Given that the voltage along the z-axis is GRE Physics 35. 36. 37. oe FEE fo drag s(t 4, determine the electric field along this axis. (A) Ves (+2) B) verte) © Vir x ©) Vv/z ® vez In the fundamentals of heat transfer the energy flux j, the thermal conduc- tivity 6, the specific heat c,, the temperature T and the time ¢ are important variables. In one dimension, what is Fourier’s law for heat transfer? Let p be the material density. (A) j=-oaT/ax @) W/de=o/c,FT/ ae B) aT /a=o/c,aT/a (E) a /da=o/pHT/a* © je+oor/ax ‘The magnetic vector potential can be used to derive the magnetic dipole m field. What is the resultant field at distance r? z (A) py (3 (mr) rm) /4nr? ®) 4, (3m- rr—mj/4nr (© py (2m-rr-mj/4nr ©) 4, (3m-rr-m)/4n7 ©) pm/4nr* x A particle is constrained to move on the surface of a sphere of radius R near the surface of the earth. What is the Lagrangian in cylindrical coordi- nates (0, 8, 2)? 38. 39. @ @®) © @) ® Test 4 x 1/2 m(p? + p°0? +2) — mgz 1/2 m (p+ p0%) —mgz 1/2. m (p7 +27) — mez 1/2 m (p90? + 2) — maz 1/2. m (p? + p07 sin? © + 2) — mgz A grounded conducting sphere is placed in a uniform electric field E = E,z. Determine the electric potential ®(r, @) given that @(r, 0) = [Ar + Bir?] cos 8. “ ®) © @) ® —E,rcos @ Ey [1 = (alt) cos @ — Ey (aft? cos @ —Eyr (1 + (ale)?} cos © + Ey (alt) cos @ What is the differential statement of Poynting’s theorem? Let $ be the Poynting vector, u be the field energy density, and j be the current density. (A) @) © VS 4j-E-0 @) V-S +du/a4j-F+IR=0 V-S +du/a+j-E=0 ©) VS =0 V-S +du/ar=0 GRE Physics 40. 41, 42, A woman wishes to resolve objects one meter apart at a distance of 10,000 m. Assuming light of wavelength 6000 A, what is the minimum diameter circular lens needed? (A) 29cm @) 03cm (@) 14cm ©) 01cm ©) 0.7m Light is incident on a prism of apex angle o. = 60° at angle of incidence ©, = 30°. The prism is made of a substance of refractive index n, = 1.5 and is surrounded by a near vacuum, Determine the net angle of deviation of the light due to passage through the prism. (A) 30° (B) 60° ©) 2° ©) 4” ©) 70° Consider the general thin lens problem where the object sits in medium 7, the convex lens is made of material n,, and the image is found in medium n, If the curvature radii are R, and R, as shown, then find the secondary fo- cal length f” mg! Un, 1g) /R, + (n,-1,) Ry) B) f= n/n, a/R, + 4,8) © fanaa) /R, + 1-0) / RD @D) f= ny) Lorn) R, + (0, -1,)/ Ry) ©) f'=A4/R, +R) Test 4 43. 45. 0 Determine the logic statement for the CMOS gate shown, where Q, and Q, are n-channel MOSFET and A Q, and Q, are p-channel MOSFET. Assume positive logic. (A) NAND + Vee (B) AND () OR a] LILHIU LU @) XOR (©) NOR A TTL logic gate is designed to be a NAND gate using positive logic. ‘What is the logical statement assuming negative logic? (A) NAND. (@) OR (B) NOR © XOR (©) AND For the below circuit, determine the waveform which appears at point A. Re Ry ccc * RRS A (A) sine wave of frequency w= 1/3 (RC) (B) sine wave of frequency 1/-/6 (RC) (C) square wave of frequency 1/RC (D) square wave of frequency 1/-/6 (RC) (G) triangle wave of frequency 1/RC 331 GRE Physics 46. What is the basic function of the below OP AMP circuit? 47. @) ®) (c) © ® vi Vo = acts as a low impedance buffer with unity gain, acts as a high impedance b uffer with gain equal to the open loop gain of the OP AMP, acts as a low impedance buffer with gain equal to the open loop gain of the OP AMP, acts as a low impedance buffer with gain equal to the closed loop gain of the OP AMP. acts as a high impedance buffer with unity gain, For the shown circuit, what best describes the output voltage compared to the input voltage at very high frequencies? A) 6) © ©) ®) 10ka pe aH 10ka V,=V, and leads by 90°. bs V,= 1/2 V,and lags by 90°. V, << V,and is in phase, V,= 1/2 V, and is in phase, V, << V, and lags by 90°, Test 4 49. 50. AA) \@n+IR Which of the following is NOT a correct statement about the phenomenon of optical activity? (A) _ the vibration plane of light undergoes rotation when passed through a substance like turpentine. (B) the electric field of the incident plane wave rotates about the optic axis (C) quartz is only dextro-rotatory () a substance that causes clockwise rotation (looking in the source direction) is dextro-type (E) a substance that causes counter-clockwise rotation is levo-type Consider a plane transmission diffraction grating. Let d be the distance be- tween ruled lines, m the order number, and @ the observation angle. Find the angular dispersion d0/dd. for incident light of wavelength A. (A) sin@/A @) tan0/r (B) cos6/A (©) sec/r (C) coto/r Newton's rings are observed with a plano-convex lens resting on a plane glass surface. If R is the lens radius or curvature, m is the order number, and 1 is the incident light wavelength, then find the radii of dark interference rings. B) Yond) /R ©) Var ©) marr ©) Yom+1/29R GRE Physics 51. Determine the speed of the photoelectrons ejected from a metal surface. ‘The threshold wavelength is 2638 A and the wavelength of incident light is 1600 A. (A) 52x 10° m/s (D) 1.66 x 10° m/s (B) 2.6x10' m/s ©) 1.04% 10 m/s (©) 2.08 x 10 m/s 52. Electromagnetic radiation of wavelength 6.20 A is incident on a substance and back-scattered at an angle of 180°. Determine the Compton energy shift of the EM waves. (A) 31.0 Y . @®) 155 Y Ae e <<“. — (C) 2.0kev io) ©) 40K ® © ©) 10keVv 53. A particle of mass m moves in one dimension subject only to a resistive force F, = — by. Let y= b/m = 2.0 5" and the initial speed be 100 m/s. De- termine the distance moved at f= 2.5 s. (A) 500m (8) 250m Fr © 125m —_— @) 49.7m © 248m 54. Consider the motion of a rocket in free space. If the rocket starts with initial velocity 0.5 km/s and its mass decreases by a factor of two due to exhaust emitted with speed 1.0 km/s, then find the final velocity of the rocket. Test4 55. 56. 37. @) (B) © @) ® 1.5 km/s 0.5 km/s 0.6 kms ~———_ ¢— 1.7 kms exhaust rocket 1.2 km/s Obtain the expression for the acceleration of a mass in cylindrical coordi- nates. (Hint, find r then use to get v in cylindrical coordinates.) (a) @®) © @) ®) The position of a particle is given by P = (0% + p0%) p+ (98 -2p°0)6 +22 0° + po" b +23 (8%) p + (98 -2p°0)6 +22 (p’—p0%) 6 + (98 +2p’0)6 +272 0, 1.0) m in standard Cartesian coordinates. What is the position of this particle with respect to a frame rotated by 30°? C8) @) © @) ® (1.00, 1,00) (1.37, 0.37) (0.37, 1.37) (1.37, 1.37) (0.37,0.37) x Stirling's approximation for N! may be found for large N by using the ‘gamma function, What is the approximation to second order? (a) Wiley @) GRE Physics 58. 59. © Wie" Vin ©) (Wie) V2RN UT -1/12N) © (N/e)" V2nN ‘The microcanonical ensemble theory value for the cumulative number of states (ideal gas) is wna (2B )icenmy ‘Use this information to determine the entropy, i.e., the Sackor-Tetrode equation. Let g =m / 2 mh and T = 2E/ 3N. T(e)= V™ P?% where P= l2mE . (A) S=Nkin [(@7)*V/.N] +5 2NE (B) S=NkIn [ (gT)V/N} +3 2NE (C) S=3/2NkinT+NkinV +5/2Nk (D) S=3/2NkInT+Nkin V+3 /2Nk ©) S=32NkInT-NkInV/N+5 [2Nk Use your knowledge of nuclear and particle physics to determine which elementary ctoss section the below curve represents. 50 6 (mb) ° 18 48 Boev (A) the pp total cross section (B) _ the nn elastic cross section (©) the nn inelastic cross section @) the direct pn cross section ©) the mn cross section Test 4 60. Determine the threshold kinetic energy to produce proton-antiproton pairs in positron-electron collisions. The positron KE is T, and the target electrons are at rest. 61. 62. (A) @) © @) ® 1.876 GeV 0.938 GeV 1,72 TeV oe o 3.44 TeV 0.86 TeV Obtain the correct classical Lagrangian for a particle subject to an electric field E due to potential @ and a magnetic field due to vector potential A such that B= Vx A. (A) @®) © @) ® L= 12 m+ qo-qa-¥ L=12mv-JqE+vxB): dr L= 12 m—ao+qh-¥ L=12m+|qE-dr L=1mvi+qo+qa-v Find the distance of closest approach for the elastic nuclear reaction 7 Lit % Pb. Assume that only the Coulomb force is important. The Li nucleus is accel- erated to a kinetic energy of 50.0 MeV. @ ®) © @) ® 1.12 fm 2.24 fm 3.54 fm —_—_—> - 7.08 fm beam target 8.20 fm GRE Physics 63. 65. ‘A charge q is distributed throughout a sphere of radius R with uniform charge density p. Given that the potential is $= —pr/ 6¢, + Air +B forr >r. (A) Re[l/r+1/d]/xo @) R=[l/r-1/d\/no ®) R=[2/r+2/d\/xo ©) R=(2/r-2/d)/xo (C) R=d/nor ‘The nature of the gravitational field and the electric field are alike in that both are inverse square, Use this analogy to develop a differential Gauss’ law for gravitation. Let p be the mass/volume. (A) Vg=-4nGp/e, @) Ve=-p/G () Vg~- Gp ©) Vg=-4nGp (©) Vg=-4x9 Tost 4 The first variation ope Lice tev eax is used to determine the Lagrange equation, What is the second variation of the integrand of the action integral A= J? f@xx) dt? 1 oF sy? OF 4 Of gy (a) Gy 80" 42 gra Bbw t Oe y a (74 2f xy? @) Fe B+ Or) 2 2 © of (82)742 LS 5x80" ox OxOx" af 1 OF ges? ©) 2 spe Bxbx'+ apt Ox) ® a (8x) ‘What is the fundamental physics hasis of Snell's law? (A) _ the first postulate of special relativity (B) the Pauli principle (C) the uncertainty principle (D) Newton’s first law (B) Fermat's principle of least time Determine the two dimensional Lorentz transformation matrix K + K’ us- ing the rapidity variable y = ip. cay {shy = tiny } isinhy cosh y (B) (coshy isinhy isinhy cosh y GRE Physics 70. n. 72. (© (coshy — isinhy (or, coshy ) ) {coshy —isinhy (2rmay cosh y ) ©) (coshy sinhy (s ) Discover the first correction term for the classical kinetic energy as relativ- istic effects become important. (A) 12 my ©) 3/8 mp (B) 3/4 map ©) 12 mepre (© Vamp Consider the standard two body nuclear reaction “NV (a, p)"O and deter- mine the minimum kinetic energy needed (in the center of mass frame) for the reaction to occur. Given: .0078, m, = 4.0026, m, = 14.0031, and m, = 16.9991 all in amu. (A) 0.0 Mev @) 06MeV (B) 1.1Mev ©) 1.6Mev (C) 2.2MeV An ideal system of N spins each of magnetic moment p, is under considera- tion, Each spin can either point up or down only, where P(T) = p and P(L) = q= 1 —p. Find the variance of the mean magnetic moment. (A) Npqu,? ©) 6Npqn,? (B) pq? ©) 8Npqu2 © 4Npq,? Cosmic ray events are detected with a Geiger counter. The everits occur randomly in time, but with a well defined mean rate r = 1 Hz = 1 evenv/s such that P[1 event occurs in (t,t + dt) ] = rdt. What is the probability of recording 5 counts with the Geiger counter. Test 4 B. 14. 75. (A) .009 @) .047 @) .019 © 057 ©) 038 ‘The second law of thermodynamics is intimately connected with the trans- fer of heat and the operation of machines. Which of the following is NOT a correct statement in light of this law? (A) tis not possible to only transform heat into work extracted from a uniform temperature source. (B) _Itis impossible to construct a perpetuum mobile of the second kind (C) _Itis impossible to only transfer heat from a body at high temperature to one at lower temperature. (D) If heat flows by conduction from body A to body B, then it is impos- sible to only transfer heat from body B to body A. ©) _Itis not possible to only transform work into heat where the body is at a uniform temperature. In a centripetal motion laboratory experiment, g is computed from spinning a bob of mass m = 50 gr in a circle of radius r = 20 + 1 cm. The rotational period is measured to be T = .638 + .016 s. The body stretches a spring a distance equivalent to the force of a weight Mg where M = 100 gr. If the working equation is g,,, = 4mr/MT?, then find the uncertainty Ag. (A) 24emjs? (B) 39cmis* © 49ems* m ©) 6emis © s9emys* ~~ For one mole of ideal gas and the Carnot cycle pictured on the following Page, find Q,,—Q-- oat GRE Physics 16. 7. Qa v (A) RT, InV,V,-RT,InVIV,, @) RT, InVV,—RT,In VV, ©) RT, In VIV,-RT_ In VV ©) RIt,-T3 ©) RT, nV V,—RT,InVIV, Use the Boltzmann factor to study the thermodynamics of N independent particles of a spin system in a magnetic field where the energies are E, = Fp. Find the total average energy at temperature T = 1/Bk. (A) -Nu.B coth (By,B) 8 (B) + NpBcoth By,B) © -Ni,B tanh (By) | | ) —Nu,B sinh (Bu,B) ©) + Ny B tanh BB) Evaluate the microcanonical ensemble theory density of states for a harmonic oscilla- tor with hamiltonian p?/2m + ‘ee, The phase space plot is shown below. Let @ = V/k/m (A) Efro ®) Iho Test 4 78. 80. © Efo ©) VE /ho © Eto Identify the below phase space orbit (A) Lissajous figure (B) damped motion. (C) simple harmonic motion (D) gravitational motion (g = constant) (©) _ logistic difference equation Consider the spectroscopy of the hydrogen atom in Bohr theory. Determine the upper limit for the Brackett series. Given that R = 109,677.6 cm". (A) 1216A @) 1880nm @®) 65634 ©) 7450nm (©) 4050om By looking at an empty glass along the ray path shown (angle = @ ), one sees the lower left hand corner. Now when the glass is filled with a clear liquid of refractive index n = 1.3, one sees the middle of the bottom of the glass again looking along angle 8. Given the width is 5.0 cm, find the height y. (A) 2.73 cm B) 5.46cm © 135cm ©) 4.08cm ©) 633cm GRE Physics 81. 82. 83. A circular annulus of inner radius a and outer radius b is centered at the origin in the yz plane. The an- ulus is filled with positive charge of density 6 = charge/area, Deter- mine the electric field along the x- axis. “ ®) © © ® B=2nate[1/ Va +a? 1/1? +67 |x E=2nokex/Vx" +a? B= 2nokex/Vx? +6? E=mo[b?-a?}x/x? B=-2nokx[1/-Vx +a" ~1/Vx? +07] x ‘The surface temperature of a blackbody such as the sun or any star can be found from plotting the intensity u(A) versus the wavelength A. Generally, uQ) 0: 1/[%(e - 1)] where x = hcB/A. Find the equation for x which one could use to find the temperature. “« @) © @) ® e(S-x)a5 e4-x)=4 e(3-x)=3 uf) eQ-x)=2 e(i-x)=1 A ball bounces elastically in the vertical y direction. Calculate the energy levels using Bohr-Summerfeld quantization. (A) ®) © nghm 5 | e [ong am]? (ong atm)" Test 4 85. ©) [9n?x*g*h’m/s)° ) [9n*ntg*h?m/8]}” A metal ball is dropped into a deep well with water on the very hattom. The time taken between dropping the ball from rest to hearing it splash the water is 6.83 s. Calculate the depth of the well assuming c = 330 m/s. (A) 229m (B) 219m © 201m @) 191m ©) 181m Consider a quantum mechanical problem where the eigenfunction is the spherical harmonic Y,, with = 1 and m,= 0, i., ¥,, (8, )=N cos 0. Find the normalization constant N. A V3/65 ©) Via" @ Vik © Vian © V3" ‘What is the degeneracy of the energy for a Hydrogen like atom with princi- pal quantum number n and orbital quantum number /? An @) © wv ©) «+n ; v © w+ GRE Physics a7. 88. 89, Study the ballistic pendulum problem where a bullet of mass m_ becomes embedded in a block of mass 1000 g. Given that the initial veloc- ity of the bullet is 20,000 cm/s, determine the height the ballistic pendulum rises. (A) 5.05 em LLL. (B) 10.1¢m (©) 15.15em @) 202m ce> () 25.25em Determine the speed of the Lorentz transformation in the x-direction for which the velocity in frame K of a particle is u=(/V2, ciV2) and the velocity in frame K’ is seen as =(-c/V2, ciV2) A) 14xl0'ms y (B) 28x10" m/s (©) 7.0x10" m/s @) 35x10 m/s ©) 12x10’ ms x « In the quantum theory approach to the hydrogen-like atom for = 0 using the Schrddinger equation, find the energy eigenvalue for the ground state radial wavefunction Ro(r)=Ne~™!®, (A) -PZpe2 @) FZ pet/2e (B) -Zet/Yph? ©) FZe!nh (©) -#Zpet/2be Test 4 90. 91. 92. 93. Two events occur in the space-time continuum. Event A has coordinates (1m, 2m, 3m, 0s) and event B occurs at (2m, 3m, 4m, 1/c) where the 4th coordinate gives the time in seconds. Calculate the proper distance between these two events. (A) Im wo) V2 m 8) 2m ® V5m © Vim Use the Fermi gas model for electrons in a metal to determine the Fermi momentum k, = p,/t for electrons in a metal of density .971 g/cm? and mo- lar mass 22.99 g/mol, (A) 0.11 A+ @) 022A+ (© 04s A+ @) 091 A+ © 18244 ‘The complete wavefunction for a particular state of a hydrogen-like atom is V(r, 8,6) = Ne Ogintoe?, Determine the eigenvalue of the angular momentum operator L,, A @) 4h @) 2 ® * (©) 3h Study the coupled harmonic oscillator problem pictured below. Find the anti-symmetric mode frequency. pinning in 47 GRE Physics 94, 95. 96. (A) Viim (D) Vn +2/m ®) VKim ® (k+0/m © V+ 20m ‘Determine the electric current due to two electrons in a 1s* quantum state orbiting a central nucleus at distance 1.0 A in a circular orbit. (A) .0004A () .0016A (B) .0008A ©) .0020A © 012A Evaluate the circuit shown below to determine the anti-symmetric mode frequency. Let k= 1/LC, x, = I/Ly, and x, = B/L. (A) I I ve to L 2 (B) “ ler 2k,) I © SkriK +I) ¢ 5 | ©) Jarry ® Jerqiare F. London and H. London in 1935 explained a superconductor as a single wave function describing a coherent collective state. Use the London equa- tion j = — A/j1,A,? to determine the equation for the B field as a function of distance into the superconductor. Hence explain the Meissner effect. (A) B=Oinside B B ®) B=Bye (© B=B,(1-xA,) () B=B, inside ©) B=B Se) T>Te T ‘The phase velocity is k = [8h 4 282 an” using the definition of wave number k = 2n/A. 7. (B) The use of vacuum techniques is essential in many laboratory experiments. Mechanical pumps pump down to about 10“ torr (about the same as 10° mm Hg). Molecular diffusion pumps can then take the system to 10” torr, Final pumps are used to get to 10° torr. Liquid nitrogen cold traps are used to condense Volatile vapors in the system. The pressure in free space is much better than any of these values, about 10~* torr, Some mechanical pumps are fwo stage in design. Ton pumps are generally of the cold cathode or hot filament type. * K ne dimensional non-linear oscillator is usually developed as an approxi- mation to a general potential function U(x) = UO) + U'(O)x Vy UO +... using a Taylor series. In the given problem UG) ="), be +), bx, Test 4 Constant terms like U(0) do not affect the results. For stable equilibrium in a symmetric potential, the odd terms are zero. The force is F=-VU o peage in 1-D = ake bx? 9. (B) ‘This is basically the soap film problem, A film has a certain surface energy per unit area 7. The total energy is then just y * area, Hence minimizing the en- ergy is tantamount to minimizing the action integral A= 2ny ds =J2myafi ty? ae. fayity? =O.) the 2nd form of Euler’s equation is needed. Since of od . E-Elore After a little work, we get z yeceosh( 10. (A) This is the standard pendulum problem, but in an effective local gravi- tational field getBtye a3 =e. By Newton's 2nd law for rotational mo- tion r= lo —mg,1sin0= 10" mp0’ thus 97+ sino=0 is the equation of motion. For @ << 1, a Taylor expansion gives sin @ = @. Detailed Explanations of Answers is the angular frequency. Also aMoot [38 On le is the linear frequency. In other words, the problem may be solved by substituting 8, for g. 11. (C) The room volume is V = Iwh=(2.5 m) (5m) (5m) = 025m The smaller volume is Vy = (.025 m) (.05 m) (.05 m) = 6.25 x 10m’. The probability parameter is p= oe 10°. ‘The situation under consideration is binomial in nature p(n) = (’) p'(- py”. The desired probability is povy= ep" ap) ” ~6.10 =(10"y or log,, p(N) =- 6 x 10”, 12. (C) For gravitational 1-D motion with a resistive force the equation of motion is mx” = — mg bx” G+), ym bim. Integrate fo apa Btw toget 356 Test 4 dip £+¥ oy. Y 8t% Exponentiate to get gtwa(gtye™ or v=[Een}er-£. Y Y ‘The maximum height occurs when v= 0 so that are Y Y, or ee Y & Note for y<< 1, we get using a Taylor expansion Avot 2 Yo “ys 8 ‘the vacuum value. 13. (E) Gauss’ law for gravitation is 1 ——_—__+ V-g=-4nGp where G = the universal constant of r gravitation. Applying the divergence theorem, we get fy-da=-4nGm,,. For an infinite line mass, the mass density is 1 = m/l. Use a Gaussian cylinder for integration to get $g-da=—4nGN Detailed Explanations of Answers ‘The basic equation to find the gravitational potential is an o=-cf = =-Gff 2masds r using the definition of mass density o #0 that dm =2nos ds using the theorem of Pythagoras. Integrating, we obtain =-2n0GV 5s? +x? =o Fae? A Note that one could also find g since =-Von- Bx. N 15. (C) At the top of the circular path IF=W-N=F, | Fe The pilot's apparent weight is w 2 Ny =mg— 7 N At the bottom of the foop t IF =N-W=F, Fe and the pilot’s apparent weight is 2 Ny=mg+ w r ‘We want 16. (C) This is the standard Atwood’s ma- chine problem with m, > m,. The two me m 358 Test 4 free body diagrams shown here. T By Newton’s Second law T mg -T = m,a and T-m,g =m,a af Solving the Second equation T=ma+mg and substituting in the first: m9 m9 mg—ma-mg=ma (m, —m,) g = (m, + m,a a= (m,—m,) g/(m, +m,) For m, = 4m and m, = m, we obtain. oan 17. (D) This question concems some of the basic properties of nucleons and nuclei. Protons and neutrons are nucleons. They are also fermions since they have spin s ='!/Jx. Their orbital angular momentum is integral /= 0, 1,2, ... The total angular ‘momentum of collections of nucleons in nuclet is JeLs, int and is @ integral for even A nuclei (i) half integral for odd A nuclei and (iii) zero for even z, even N nuclei. 18, The Roche limit is the Earth- moon centers distance at which the tidal action of the Earth would rip the ‘moon apart. Consider the moon as be- ing composed of two halves. Then the attractive force is mm ip Feo oor = Gn /4r? and the disruptive tidal force is 359 1 wee lw - an Taylor expanding = Hie fia (1-29)] = 2GMm1/a® Set F, = F, to get the Roche limit distance Grit | 47? = 2 GMmr{a* @ = 8Mr|/m thus =a =(8M/m)"" r= 16,000 km. 19. (D) ‘The electric quadrupole moment of a charge distribution is o=22(a'-6) where Z = the number of protons in the nucleus, @-= the nuclear semi-major axis, and Clearly this factor is a measure of how elliptical the nuclear charge distribution is, The eccentricity is Vat =b fas &a (aba? Q= 2 Zea. For a circle ¢ = 0. The deuteron, for example, has Q = .003 bam. = the ellipse semi-minor axis. ‘Thus 20. (D) ‘The Tukawa potential is based in the meson theory of nuclear forces. The relativistic wave equation 2 yet 2) - (° we % Fa) on0 may be separated by O(r,1) = (reo (=p) o=0 for a virtual particle. The radial solutions are $ ~ e™ / r. The Tukawa potential has the same form 360 Test 4 U(r) =V ge [r-Vye™ |r ‘The repulsive part is Vye |r with force ea) 21. (E) It is desired to use the nuclear shell model to find the © Zn spin. The proton configuration is » (1s, IP ya)* Pa)? (1d, (28,7 (dya)* fa)" (Py)? and the neutron configuration is (18)? «++ 2Pyg) (tga)? One looks for unpaired nucleons to determine j. Only one If,, neutron is un- paired, Thus j= 5/2. 22. (E) ‘The photon is a stable particle with zero mass, infinite lifetime, and spin j = 1. The electron is a stable particle with mass 0.511 MeV/c?, infinite lifetime, and spin j = '/,, The proton is a stable particle with mass 938.28 MeV/c, lifetime greater than 107 years (perhaps infinite), and spin 1/2. The pion n° has mass 134.96 MeV/c?, half-life r=—8 x 10°” s, spin 0, and commonly decays to yy. The kaon K* has mass 493,67 MeV/c, lifetime 10 s, spin zero, strangeness S = 1, and commonly decays to 1"D. 23. (C) Hadrons are built of quarks whereas leptons are fundamental particles. Quarks have spin j= 1/2 and baryon number B = 1/3, Antiquarks have the same spin, but opposite baryon number, electric charge, and isospin. u, d, s, c, b and t are the flavors of quarks: up, down, strange, charm, bottom, and top. Some common hadron configurations are peuud ‘peuud n=udd n=udd = A*=uuu weud weud = Jace Kr=us K=us Detailed Explanations of Answers 24. (E) In the lab frame the atomic scattering looks like Ae ee LL whereas in the CM reference frame, one sees \ A where @ = 20,,, and = $,, (not shown). Particle number is conserved go that (0 ,,) dQ,, = 0(0)dQ " 7 " (0 )2nsin®, d0, =z 2nsind do = 6,2sin @,cos @ 240 /2 5(0,) = 0,005 0 /r. ‘The differential cross section is thus not isotropic in the lab! 25. (C) In radioactive decay N=Ne™ follows from the assumption that the decay is a random process where the prob- ability of one decay is Adt. (This means dN = — AN dt.) The half-life is related to the decay constant Ny /2= Noe? => fy, =10(2)/2, as is the mean life Test 4 26. (E) Since the discovery of lasers in 1955, this laboratory technology has found diverse uses in the cutting and welding of metals, the research study of nuclear fusion, dermatology, and even art/music/entertainment. A small CO, laser can ra- diate at 10° times the solar intensity. The basic idea is that of optical pumping or to produce a population inversion. Suppose the atoms have 2 quantum states £, and E,, Then hw = E, — E,, Also, according to Einstein dN, —L=N,B,.u(@) n= NiBiaH(@) a Aa + NaByu(@) At equilibrium N’, = N’, so that Big = NyAay +N, Ba thus a = Byu(@) | (Any + Bye). h 27. (E) In the Raman effect, an incident beam of monochromatic light of frequency © induces a dipole moment in a molecule. This inelastic interaction results in scattered radiation of frequency @ =0t 0 depending on whether hav’ of energy is given to or taken from the molecule, The electric field of the light interacts with the molecule. The incident light can be of any frequency whereas in fluorescence, the incident photon must be at the proper molecular absorptive frequency. 28. (C) One can use the Biot-Savart law aB = 42 jaxs/s? 4n a Hol desin® | 4a s* and plug in the trigonometric facts ‘csc® and cot @= —* sothat dx =resc?@ d0 toget Detailed Explanations of Answers Integrating, one obtains Bol oe Bol = apr), sine do F cose}, Bel = Far (cos@ — cos 6°) 29. (D) Consider the electric potential energy between the spherical shell of differ- ential charge dq = 4nx'drp and the central charge a=fmrp. ‘The differential potential energy is dU = kadq |r = l6n*kp*r'dr /3 ‘The spherical charge distribution is total electric potential energy is then v= 16n7kp* i Ya 3 fo 16 12, 275 = 18 ntepte’ is * *P 30. (A) ‘Maxwell's equations in free space are = oB V-E=0 VxE=-5 V-B=0 VxB= nye and they yield a wave equation “om e() ee where Hob Test 4 ‘A plane wave solution is Ey, cos (ot— kx + 0) +2, cos (ct— ka + B) and Fa conte be +42 cont ke +0) since c= wk, Now ifm B—a, then a (2) +[ EL) -corereateen oy or where mt — kx + 0. Clearly for 5 = 4n/2, we get elliptic polarization 2 Ee) 31. (C) Ampere’s law in medium is VxH=j, By Stokes Theorem § Hedr=f VxH-da ‘Thus $H-dr=fj-da=!, Integrate around a circular loop to get 2nrH = NI Solve for H=NT/{2n(a+b)/2) since r= 24> =NI/n(a+b) Thus B= pH = Nhl /n(a+b) since the magnetic field B is related to the field H. Note that in the vacuum case By =Nitol /n(a+b). 32, (0) ‘The Fraunhofer diffraction pattern results from monochromatic light waves incident on a single slit. The condition for destructive interference is, dsin@=nd Detailed Explanations of Answers and y= Xt gives the first minimum. The intensity pattern is ra )(G6) i ad Finally 1 _ 4 z = 33. (A) The electric field is that of the given charge and an image charge of equal and opposite magnitude. Hence z+ E, = tq] —2=2 > —_2 tt Gey" ‘The charge density is [fos ds fa =~ anf s dois? + ny? =ahists NT ast thy | =-q -qe ‘The infinite sheet has charge q on top, as it must. 34. (B) The given voltage boundary condition is { Vy sr Test 4 for z = 0 with along the z-axis, sds {tthe f(Z)= 2p vals +24)" taking ["do=27 -v:(1-— wonre( sat) =¥[ 1-2 (a) Now we calculate the electric field. a(_ ve 2) by the definition of the electric field E Ov? +. V 6. Differentiate to get ay ) F 2(2(r? +27)" E,=Vo a dipole field. 35. (A) Let j be the energy flux or energy per unit area per unit time, The energy flow is in the x-direction and the temperature decreases in that direction. This is just the Oth law of thermodynamics: heat flows from high to low temperature, Fourier’s law then says or oor where © is the material's thermal conductivity. Consideration of energy gain gives i a Per or ax’ where c, is the specific heat. Using Fourier’s law. we obtain Wo OT ar Bey axe 367 Detailed Explanations of Answers 36. (A) ‘The magnetic vector potential is Antes jarjrates and rire) where jis the current density. By Stoke's theorem Holt as x¥(2) an) Ox, ol da x Bolt da an ORE since dm = Ida for a small current loop. Ho r wal abomxt z Finally anor B=VxA =Hevx(H3t) an 7 7 = Bt [sem-ry—m] which is the standard form for any dipole field. 37. (A) ‘This problem is best treated in cylindrical coordinates, as suggested. There x+y =p? and -x=pcos®,y=p sin®, and z =z. ‘The potential energy is U = mgz taking U=0 at z= 0. Also, ve p+ pote 2? in cylindrical coordinates. Hence the Lagrangian is L = T-U='),mv*—mgz = Yi m(p? + p07 +2) — mez. 38, (B) ‘The problem of a grounded conducting sphere in a uniform electric field has potential © which satisfies Laplace's equation: Vi@=0fora B=-Aa®=a7E, Hence the electric potential is aE Ha Eyr(1-(#))eoso (7,0)=|- Ey + *. ceasider a eyiten of charges and an B fed. Then slce power =F -¥ Scie where F = gE has been used =| pv-Ed’r passing to the continuous case JEd’ref wer where is the mechanical power density and j = pv is the current density. Let be the field energy density and § = E x H the Poynting vector. Then ¥-8+244 5-2-0 is the differential statement of Poynting’s theorem, In integral form, we have § s-da+f Rare jE@r=0 which is just the conservation of energy. Detailed Explanations of Answers 40. (C) ‘The minimum angle of resolution for a circular lens is d LI. The relationship d= 10, follows from the radian definition of angle. We are given d = Amand! ~ 10,000 m. Thus 0, = 10*rad and using 2 = 6000 A D = 1224/0, = 1.22 x 6000 x 10"°/ 10+ —_)\ ¢ = 73x10°m 7 = 0.73em 41. (D) ‘We are given 0 = 60° and @,, = 30°, By Snell’s law sin @,= 1, sin ®, (1) sin 30° = 1.5 sin @, => @, = 19.47° By geometry = 0, +0, => 0, = 60 ~ 19.47 = 40.53°, Again apply Snell’s law to get 1.5 sin 40.53°= 1 sin @ => @, = 77.10°. Now the total deviation angle is 8 = (0, -9,)+(@,-9,) 0, +8,-a = 30+77.1—60=47.1°. Note that the near normal incidence prism formula cannot be used here. ny 42, (B) R This is the general thin lens problem. Apply- ing the optics equation for going from one medium to another a we get 370 Test 4 OR Ry a - 7, * oF Gra gIR, +a, = mR, 43, (E) i-channel MOSFETs conduct for positive gate to source voltage. p-chan- nel MOSFET’s conduct for negative gate to source voltage. The gate may be understood by constructing a truth table. Considering all possible combinations of input, we have: A B 9 9 2 output 0 0 off on on off 1 0 1 off on off on 0 1 o on off on off 0 1 1 on off off on 0 ‘The output is seen to be the same as the definition of A NOR B, 44, (B) Positive logic means that the high voltage state is 1 whereas the low volt- age state represents 0. The assignments are reversed for negative logic. The voltage truth table for NAND is A B output lo lo bi lo bi bi hi lo hi hi hi lo For negative logic, this becomes A B output 1 1 0 1 oO oO 0 1 0 0 0 1 which is A NOR B. 371 Detailed Explanations of Answers 45. (B) ‘The circuit is a phase shift oscillator, At a frequency of 1 = Re ong RC each RC section produces roughly a 60° phase shift. Hence the three RC sections yield a 180° phase shift relative to the output at A. When this is put back into the inverting input, positive feedback and oscillations are achieved. 46. (E) This is one of the standard OP AMP circuits. It is the voltage follower configuration. The cifcuit has unity gain and a high input impedance. 47. (0) At high frequency the inductive reactance X,=aL is very much greater than the 10 kQ resistance. Thus most of the current then g0¢s through that resistor: R Cc na Y L Also at high frequency the capacitive reactance X,=1/0C acts like a short. The equivalent circuit is a voltage divider. 48. (C) Arago first discovered optical activity in 1811 in France, The plane of vibration of light undergoes a continuous rotation when passed through a number of substances (quartz, turpentine, benzil, etc.). The E field of the incident linear plane wave rotates about the optic axis. With respect to the source direction, a substance that causes clockwise rotation is dextro type or right handed. Levo type means left handed. Crystal quartz can be either dextro or levo depending on the crystallography. 372 Test 4 49. (D) For a plane transmission diffraction grating the condition for maxima (con- structive interference) is dsin@ =m where m = 0, 1, 2, 3, ... gives the order of the spectrum. Note that for m = 0, all wavelengths are indistinguishable since @ = 0 for each of them. Differentiating, we get . 40m dc0s0 d0=mdd or $2.2 om Since m /d = sin 0/2, the angular dispersion is do_1 an me 50. (C) ‘The radii of the dark interference rings of various orders is given by raVaak where m is the order of the spectrum, 2. is the wavelength, and R is the radius of curvature of the plano-convex lens. This result follows from the thin film de- structive interference condition. 2nd=md where n= 1 for air = d=m\2 and geometry R=QR-dp+r P=2Rd-f=2Rd Thus r= V2Rd =VmiR si. (E) Note that Ag is the substance. We find its work function 9 = i,=hc/A, = 12,400 eV.A / 2638 A= 4.70 eV The energy of the incident light is E = hy=he/ X= 12,400 / 1600 =7.75 eV ‘Thus the kinetic energy of the photoelectrons is T= hv- $= 7.15 -4.10 = 3.05 eV Finally 373 Detailed Explanations of Answers my? => v= {(3.05) (2)/ (S11, 000) = 1.04108 m/s 52. (B) ‘This is a standard Compton scattering problem We are given that: 1.=6.20A. Hence whe _124keV-A 62. The Compton Shift is E =2.0keV =180°=> AN =2A, sin 3 = 2(.0242) (1) =.484 A thus the photon wavelength and energy in the final state is Nad+Ah= 6.24844, Bra HE =1.985keV ‘The Compton energy shift is then AE = E-E’=.015keV =15.5eV. 53. (D) Apply Newton's Second Law to get my =— bv and integrate: £ wae | te, tate 2: hy mo ym wrvenetaS on * dt Integrate again to get 374 Test 4 xelqcet) Y = 40.7 m substituting the given information 2.58, Y=25", vo =100m/s 54, (E) This is a rocket problem. By the conservation of momentum mdV = —V dm Integrating, we obtain = dm Ay” In a= oh, wv my V=Vy+V Inge = 0541002 =1.2km/s 55. (D) In cylindrical coordinates re=pp +f and y= ap'p4pe'be2'2 where B06 is used. Now by differentiating, one gets an Lap pip ebip eds p er bipe Bass then use #9 dt and collect terms a=(p"—p0%) 6+ (pO +290) O42°2 to get the desired answer. 56. (B) ‘The given vector is rexty =x,+3, ‘The rotational transformation is 375 Detailed Explanations of Answers v= 137x’,+ 37 x= (137, 37) m which may be confirmed geometrically using cos 15° and sin 15°. 57. (E) Stirling's approximation to second order is desired. Use the definition of the F function: MaTN+D=[ etMdr= fdr f(t)=Nint-t Now we wish to do a Taylor expansion f-X-izosten maximum _N t| eu k=N ON $= FO) FYE NDZ FONE 58. (A) In microcanonical ensemble theory, the cumulative number of states for an ideal gas is Tte)= own Wann)” V¥ (ame) ‘The factors of 376 Test 4 1" ImE)” 13N 12 ‘come from the volume of a 3N dimensional sphere in momentum space. Using the 1st order Stirling approximation, we have mol me 8)" ‘The entropy is S=kiar +iNk = went ( 2 N\2nh? 3, which is the Sackor-Tetrode equation: S=Nkin(gr)"? $45 Nk m_2 eye 5 59. (A) The pp total cross section is high at low energy, has a 50 minimum of about 24 mb, and is about 40 mb at high energies. @ (mb) 1.8 &cev 60. (0) The total rest and kinetic energy of the electron-positron pair must be sufficient to account for the rest energy of the resultant proton antiproton pair. The particle reaction is ete + ptp VS = Eg = (on, + ty) + Tish 2x9,38=(2x.511)’ +27, (511) T, =3.44x10° MeV =3440 GeV =3.44TeV 377 Detailed Explanations of Answers 61. (C) ‘The given Lagrangian L = T ~Uis LaLmw -aptga-v. ‘The Lagrange/Euler equation is F=qE+vxB) which is the Lorentz. force law. 62. (D) ‘The given nuclear reaction is is Lit 7% Pb ‘The closest point will be where the kinetic energy of the Li is converted entirely to potential energy. To hm? = Lite. 182)e* _ 2460" r r r r= 246e" /T = 246(1.44 MeV- fm) / (S0MeV) =7.08 fm In the head on collision, we thus see a transformation of KE into PE. 63. (E) Gauss’ law and the definition of electric field give -2 rer V-E=P , B=-vo2VE= on ° 0 r>R In Spherical coordinates 21a a eres 378 Test 4 We are given that $=-pr/6e, + Ar+B,r R, we guess by symmetry @ = C/r + D. The boundary conditions then ive . or > +) -0= D=0, (70) finite = A=0 (r= R) continuous => B ~ pR*/ 6e, = C/R Gauss’ law in integral form gives © gnpt2Loc2—L B-da=-S nk? = §B-da=— 4nk 2 Thus g=—2_4PR “4neR "Gey =P (Rt —p2) 44 _ and se, * + Trek afk (r -4) 2e,\° 3 64. (D) 4 2r <_—_—_ yy A matrix approach is appropriate here. CeCe JO) $2, Ry Ra JU i 1, = OR, =1/2 nor,) = 1/2 nor) (1, #0) I, = 0 Ry =1W2KGr,) = 12 nOr) (1, #0} In the same way R,, = R,, = 1/(20d) %) 4 6,)° 2x0) ale we 379 Detailed Explanations of Answers Now use /, = =~, By Ohm’s law .- 9) T 65. (E) Newton's law of universal gravitation is F=- Se, =mg ‘We can write this as a Gauss’ law for gravitation: fu: dam— DM ane? = 4G 4mm, The divergence theorem is $V -gd?r=$g-da and hence V-g$ m? =—4nGm, =>V-g=-4nGp This is analogous to the electric field law V-E=4nkp, with k= 1/4ne,. 66. (A) The action integral is A=? Fe, x°) de ‘The first variation results in A = J 8¢ de where a= Lore Doe gives the usual Euler-Lagrange equation. In a similar manner, the second variation is 2. UF (gy? ar oF 2 of ar 42 ayer 88x Sr (6x) where 8A = | 3% dt and the total variation is 2 AA =08A+5- BAL... 67. (E) Snell’s law follows from Fermat's principle of least time. The action inte- gral 4 is Azf[F Test 4 is just the time f to go from P to Q. azty he hy ale 4 ye 4 Vat+xt +a Pa? The eon cd =O gives "Tone “oR +(d-x) or an, sin @, =n, sin ®,, 68, (A) ‘The usual Lorentz, transformation is Minkowski space (r, ict) is x',) (cosp sing \(x, (P)Cat, cml) Using Euler’s formula ef =cos } +i sing one easily proves, cos =£ feet = cosh(i¢)= cosh y sing =(e* —e"*)/2i=—isinh(y) Thus, the desired result is x';)_(coshy ~isinhy)(x, x',) \isinhy coshy J\x, 69. (D) ‘The total relativistic energy is E=T+mc=me where T is the relativistic kinetic energy Teméey-Nem{ ie 1) Now we use a Taylor series expansion 1 1,43 w14¢ 443? Vine Lt pat gett with x= B?to get 381 Detailed Explanations of Answers rom (ote) 2 a1 mv? +3 mv? Bt zm +3 6 70. (B) The standard two body nuclear reaction is I+T+E+R where / is the incident nucleus, T is the target, E is the emitted particle, and R is the residual nucleus. Here the reaction is +N > p+"O with Q-value Q=m,+ m-m- m, = (4.0026 + 14.0031 - 1.0078- 16.9991) amu 931.502 MeV/amu = -1.1MeVv In the center of mass reference frame, this is the minimum kinetic energy needed for the reaction to occur. n. (€) ‘The ideal system of N spins is an example of the binomial distribution where p =P()andq=1-p=P() ‘The mean magnetic moment for one spin is

= pH, + (1—p) (-B) = (2p - 1p, and the single spin variance is 8 = <-q>)>=q>- q? = Pig + (Pi Gp Di = 4pqp,? Hence for N spins, = NQp-1)p, and 6,?= 4 Npqu2. 72. (C) ‘We are given that pit event occurs in (¢,¢ + df)] =r dt with r = 1 Hz. One must know that the distribution is Poisson with A = rt = 10 982 Test 4 ‘being the expected number of counts in r= 10 s. Thus, the probability of 5 counts is found from Pin) = ent oris (5) = We*/St = west = 038. 73. (E) Answer (A) is the postulate of Lord Kelvin and is a statement of the second law AS20. ‘Answer (B) is simply a restatement of choice (A). Selection (C) is the postulate of Clausius and is a statement of the second law. Answer (D) is a restatement of (©). An easy counterexample for (B) is the heating of a body, no matter what its temperature, by frictional work. Clearly also electric energy can do work and be transformed into heat in a toaster (resistor). 74. (D) For this centripetal motion laboratory experiment g=4n' mr /TM. ‘The standard rule for the propagation of error is as) (a . a= (Har) +(3 #7) 7 a T - (De) Hence the relative error is Ag Ary* TY F-4 +4F) - JC = 07 ‘The experimental g-value is thus 970 + 69 cm/s?. 75. (E) For the two isothermal parts of the cycle AU =0. Hence, by the first law of thermodynamics AQ =AW Detailed Explanations of Answers > o -2e=We- Ji pav Yp RT. Ve Ine iW =~ RT en ve Therefore Ye O.= RTetng= and in the same way we find 24 = RT yn Ze. Thus, the total amount of heat absorbed is 24-2 RT, In Rr. Inve vy, 5 Vy 76. (C) For the simple two level system, there are only two relevant energies E_ = pB and E,=—p,B. ‘The partition function for the system is Zam HHP 4 Mok seo tel = 2cosh(ByyB) ‘The single particle average energy is =-nZ Bp =~2p1 Bsinh(j1gB)/2cosh(By1gB) = —HoB tanh(BpyB) Hence the total or N-particle energy is , = NcE> = —Np, B tanh(B py, B) 77. (B) ‘The Hamiltonian of the system is Hx, p) =p? /2m+ b2/2=E, ‘The cumulative number of states is found in the usual way 1 Te\=55) Q(E-H) dr dp Test 4 msgsel, 4. poVome— met et mE -\1— ke? sig SM Name =F [2 ae “ho Using o=Vk/m and a u-substitution u=sind= Vk/2Ex. Finally, the density of states is a_i dE bo Note that J @ (E - H) dx dp could also bé found from the ellipse area formula nab = m7 Vim = 2RE 0. 78. (C) ‘The phase space orbit is easily identified as that of simple harmonic motion EF =k =m" x= @,2x=0, @, =k/m xx! H052xx’= 0 £(beste) afl yayl aye = hex’ )=0 43 my ‘The constant is the energy Eadmts dee which may be written as (JG) with = A=2E/k and B=\2E/m. ‘Thus the phase space plot is an ellipse. 79. (C) The Brackett series in hydrogen involves transitions from energy level m to energy level n = 4 resulting in the emission of a photon. The wavelength is found 385 Detailed Explanations of Answers 1/X=R(1n? — 1m?) m where R is the Rydberg constant. ‘The upper limit comes from m = 5 1/0= 109,677.6 CYy_—"fhy) which gives \ = 40,500 A = 4050 nm. 80. (A) Applying Snell's law to the second figure, we get n,sin®, = n, sin@, w= 0.3) 28 Wy? +25 Vy +6.25 213 Wys2s jy4625- ‘Multiplying means and extremes and squaring, we get 4y? +25 =1.69y? +42.25 2.31y? =17.25 y=2.73cm 81. (A) The continuous charge distribution approach is: B-4j 4 where dq = oda = 2 nosds 2nosds x =k era Test 4 Using x cos8 =~ r Since by symmetry only the x-component contributes. sds E=2nok[? way ls=a ls=b 1 = 2noke x Tae 1 1 =2n0ha - one a ee 82. (A) The blackbody distribution may be expressed in various ways u(@) deo =u(yar, ko? 1 ware we we ince) @=2mv=2n£, ‘The intensity u(A) has a maximum when du(h) | d= 0 Rearranging terms, we get e” (5 — X) = 5 which has solution x = 4.965 so that Yee T = he 4.97 K. 387 Detailed Explanations of Answers 83. (D) ‘The Bohr-Sommerfeld quantization rule is 1 = ae Jp ayaa. For a ball bounding in one dimension y, the energy is BP E=T+U=L_4+m 2m "8 Tus 2 f°" 2m E=mgy) dy = nh E 4 2 =z n= 32 Vm 2 (1-782) EP ing on 3 E) my oo Simplifying, we get alae” smh ot E,=(9n"g?n?h? m/8)"” 84. (D) ‘The total time is a combir wave to propagate. t= (IiTg thle hcy2/gVh-ct=0 h+149,08Vh -2253.90=0 ‘Completing the square (Wh +74.54)? = 2253.90 + 5556.12 Vit =-74.54 488.37 thus #=191m. tion of the fall time plus the time for the sound 85. () ‘The normalization constant for any spherical harmonic is found from the condition S[rf Q=1. ‘Thus or /=1 and m,=0 ‘ J Noto *da= Nn? jo=0 an teo op 3 =r A a cos @4(cos0)/ ab Test 4 Hence 86. (B) ‘The degeneracy for the H-like atom problem is found from thinking about the quantum numbers which describe a state. n = 1, 2, 3, ... is the principal quan- tum number. J = 0, 1, 2, ...,—1 is the orbital quantum number. m,=—1, - (I~ 1), ... -1,0, 1, ...,/—1 is the magnetic quantum number. Hence the degeneracy g is the number of states that have the same energy E, at g= LQl+) ino mtn =2 D+ T1 me (n-1)(") =e 2 =f 87. (A) An initial, intermediate, and final picture is helpful to solve the problem. “T “T Z a o j—* m+M By momentum conservation: By energy conservation: my=(m+My" Jeneaoy? =(m+M)gh +M yee Pah v=2gh 2 (_m ate) f: 2 =[ 5520-000] 12980) =5.05cm Detailed Explanations of Answers 88. (B) ‘The Lorentz transformation for coordinates is x = ¥Qr-v) » a) By differentiation one finds the velocity transformation u,"= (uy -v)/ (1—uv/c?) ty /y(1—u,v/ ec?) u,/=u, [y-u,v/c?) We are given that 1 1 a-(Gecke) aa we(-he dee} Hence v= Be =(310*) (0.94) =2.8x10* m/s 89. (C) ‘The Schridinger equation for / = 0 is 1 d (.dR), mr? te (2B)+ AP E-U)=0. Now the wavefunction is given as Ry(r)= Ne "° and the potential energy is U(r) = — kZe’/r. Differentiating, one obtains Test 4 Thus the Bohr radius is a= ue? and the energy eigenvalue is EB a-W 2 /2pa t= AZ pet 2b 90. (D) ‘The proper distance squared is as? = SdeMae, oO desde ede—cde. Hence we have Ast= Av + Ay? + ArtCar and since x(A) = (1, 2, 3, 0) and x(B) = (2, 3, 4, '/c) we get 2 At ent +-27 +a-98-(2 0) cad c =3-1 =2 or finally ds=Vim. 91. (D) In the Fermi gas model at zero temperature HL 2g Y= const. ak ~ Qn) Integrate to obtain * Lf Pane ak Qn) ° where the degeneracy g = 2 for electrons. 6.0210 /mol : - 2 where p=dxN,/A=971 g/em'x3> 75 =.0254A7 Thus =k, = 910/A. 391 Detailed Explanations of Answers 92. (B) ‘The angular part of the wave function is a spherical harmonic 1 JIS on gam Yop =¥p = 24[ 45 sin? 0 ea ao and the radial part is en vz zy" a Ra 25 (Bao, ‘The complete wavefunction is Yan (72959) = Rus 7) Foy (89)- Fortunately, L, only operates on the y component ba LWT ayy n aha 724 =2hy Hence the angular momentum eigenvalue is 2h= mp. 93. (C) x, =A.cos (ar + 8) and x, =B cos (wr + 8) ‘The easiest way to find the frequencies is to add and subtract the equations to get my” = —ky, y =X, +%, ma” = ~(k+ 2x) 2, 2=2,—%,. ‘The symmetrical mode frequency is thus 392 Test 4 k o, == ™ and the anti-symmetrical one is @, — (et 2) ]m- 94, (B) ‘Two electrons can form an / = 0 quantum state by the Pauli principle, e.g., a 1s* configuration, The current is then whee t (mr/v) a and the velocity comes from Newton's 2nd law F = ma or = (9 x10") (1.6107)? /(9.1x10™") (1x10), =1.59x10° m/s ‘Therefore the electric current is 1 = (1.6 x 10" (1.59 x 10% / m(1 x 10) = 81x104A 95. (C) By Kirchoff’s current law T=h-l, According to Kirchoff's voltage law: he py py -G=%) cor 0 -h-1, 44 =) p20 c Y 393 Detailed Explanations of Answers Differentiate and set K= ILC, K,=1Ly, %=B/L. Note that x, is dimensionless. Then we find 104m) — KL,” + (E+), 1,20 114) 4,” + E+) 1-1, = 0 Add and subtract the equations letting y =1+handz=1,-1, to obtain y+ hy =0 and atte = 0 VE + 2K, IF 2k,). 96. (B) ‘Maxwell's Fourth equation is VxBapj. ‘The London equation J=-A/BA2 is given, Hence Vxj=-VxA/ py? =-B/ pA, Also Vx(V xB) = V(V-B)-VEB Vxp,j =-V?B since V- B=0 by Gauss’ law. Thus VB = B/A2. One solution is obviously B = 0. However, the desired solution is Baby ‘The magnetic field thus penetrates into the superconductor to a penetration depth, typically 2, = 200 A, 97. (E) ‘The energy density of a photon gas is Usor., ‘Thus the energy is 394 Test 4 E=|Udr=oVl Now use the first law of thermodynamics dE =dQ-dW dQ=dE+dW = dE + pav -ortav +ior‘av 4 ort =4tortav so dQ _4 3 dS =—£=—oT*dv T 3 s=f as =4or'v ‘We have used the fact p = '/, E/V, the relativistic Virial theorem. 98. (C) z ‘The motion of two particles of masses m, = m, = m separated by a distance r is equivalent to that of a reduced mass particle [as shown, The Schridinger equation is =n 2 =Ey, H=— Vv 4U Ay =Ey, 2p +U(r) where U(r) =0 for fixed r In spherical coordinates, we get eet a 20)" rein 0 067 +2HE yao, ‘The standard separation of variables technique using (0,6) = P(@) Q(9) gives ow=zee 1d (joa m? and = 1 4d (ying dP 1-7 |P=0 aaa (0S) [1099 -a] where P=P"() is the associated Legendre function, 99. (A) ‘The two postulates of special relativity are: Detailed Explanations of Answers (1) True physical laws are the same in all inertial reference frames; (2) The speed of light is a constant in vacuo regardless of source and observer relative motion. This was Einstein's 1905 theory. In 1915, he went beyond this with the general theury of relativity for which (B) — (B) in the question are tue statements. 100. (E) This is the usual relativistic rocket problem. The four momentum is P= me = (oyu, mye). v ™ m mg Certainly the space component of the 4-momentum is conserved 0, m, c) = (— my, myc) + (m,y,u, m,y,c) so that Om = myo + mop or mye miu. The rest mass is not conserved since this is an inelastic collision in reverse.

You might also like